You are on page 1of 223

1/17/2021 Mock Analysis

LST Mock 27 2021 (CLAT)

Scorecard (ScoreCard.jsp?sid=aaa2yJ6mqgzXH-yCv3nCxSun Jan 17 23:43:00 IST


2021&qsetId=JAmrn2p8H1A=&qsetName=LST Mock 27 2021 (CLAT))

Accuracy (AccSelectGraph.jsp?sid=aaa2yJ6mqgzXH-yCv3nCxSun Jan 17 23:43:00 IST


2021&qsetId=JAmrn2p8H1A=&qsetName=LST Mock 27 2021 (CLAT))

Qs Analysis (QsAnalysis.jsp?sid=aaa2yJ6mqgzXH-yCv3nCxSun Jan 17 23:43:00 IST


2021&qsetId=JAmrn2p8H1A=&qsetName=LST Mock 27 2021 (CLAT))

Video Attempt (VideoAnalysis.jsp?sid=aaa2yJ6mqgzXH-yCv3nCxSun Jan 17 23:43:00 IST


2021&qsetId=JAmrn2p8H1A=&qsetName=LST Mock 27 2021 (CLAT))

Solutions (Solution.jsp?sid=aaa2yJ6mqgzXH-yCv3nCxSun Jan 17 23:43:00 IST


2021&qsetId=JAmrn2p8H1A=&qsetName=LST Mock 27 2021 (CLAT))

Bookmarks (Bookmarks.jsp?sid=aaa2yJ6mqgzXH-yCv3nCxSun Jan 17 23:43:00 IST


2021&qsetId=JAmrn2p8H1A=&qsetName=LST Mock 27 2021 (CLAT))

English Language

Current Affairs Including General Knowledge

Legal Aptitude

Logical Reasoning

Quantitative Ability

Sec 1

https://www.aspiration.ai/LAW/sis/Solution.jsp?qsetId=JAmrn2p8H1A=&qsetName=LST Mock 27 2021 (CLAT) 1/223


1/17/2021 Mock Analysis

Directions for questions 1 to 30: Each set of questions in this section is based on a single passage. Please
answer each question on the basis of what is stated or implied in the corresponding passage. In some
instances, more than one option may be the answer to the question; in such a case, please choose the option
that most accurately and comprehensively answers the question.

Passage – 1

Two recent incidents of Stan Swamy and Gautam Navlakha show up the state of our prisons today. Both are
undertrial prisoners in the Bhima-Koregaon case. Both had to move the courts for the simplest of necessities.
One, to get a sipper cup and a straw because he is an 80-year-old man with Parkinson's and the other to wrest a
pair of glasses to replace a broken pair. It led to the Bombay High Court into making an observation that
workshops should be conducted for the prison staff to sensitise them.

Why did these matters have to go to the courts? It is, perhaps, an indication of the fact that there is little space
for ground-level staff to make operational decisions. O cials do not want to get into trouble, especially in cases
of political prisoners arrested under national security laws.

The law on provision of basic facilities to prisoners is clear - it lies squarely with custodial authorities. The
Nelson Mandela Rules 2015 issued by the UN and the Model Prison Manual 2016 by the Bureau of Police
Research and Development, Ministry of Home Affairs, have elaborate provisions regarding the care, treatment
and rehabilitation of prisoners. A plethora of Supreme Court and high court judgments reiterate that prisoners
are human beings with basic rights.

Yet, violations are an everyday routine. They get highlighted when there is a mishap with a high-pro le person.
Notorious criminals, too, like Charles Sobhraj and Sunil Batra, who had the gumption to take matters to the
Supreme Court, have played their part in prison reforms over the years.

But standard setting doesn't mean compliance. Our prisons are full of people in compromised states. Around 70
per cent are under trials and more than 75 per cent come from marginalised sections. They know little about the
law. Even if they are aware, they have little recourse to any complaints mechanism and must fall back on an
uneven legal aid system. If at all a prisoner can reach the judge, they are either too busy in their routine work or
too enmeshed in the local culture to act. Overcrowding in individual prisons stands much higher; some prisons
are more than three times crowded than their o cial capacity.

Despite efforts at clearing cases, an acute shortage of judges and court infrastructure ensures an accelerating
accumulation of cases in courts. According to the India Justice report 2019, it takes an average of three years
for the case to traverse the high court and six years in the subordinate courts.

It doesn't have to be this way. There are pathways to accountability and to reform. But since both supervisor and
policy maker know the on-ground constraints and the biases within the culture, much is excused or pushed [1]
the carpet. Reform attempts like the constitution of Undertrial Review Committees mandated by the Supreme
Court work patchily. In most prisons, the Board of Visitors meant to function as an oversight mechanism are not
even constituted. The policy rhetoric wants prisons to be places of reform and rehabilitation. Their neglect by
the executive and oversight bodies ensures they act as warehouses for the poor and the marginalised.
Prolongation of these socalled criminal cases is unconscionable, as is forcing vulnerable people into remaining
in hotspots of increased infection and fatal risk.

https://www.aspiration.ai/LAW/sis/Solution.jsp?qsetId=JAmrn2p8H1A=&qsetName=LST Mock 27 2021 (CLAT) 2/223


1/17/2021 Mock Analysis

Q.1 [30465398]
In the light of the passage, it can be inferred that:

a The UN is concerned only about political prisoners.

b The UN is not apathetic about the condition of prisoners.

c The UN is not concerned about the condition of prisoners.

d The UN is concerned only about Prisoners of war.

Solution:
 Answer key/Solution
Correct Answer : b
Your Answer : a
Refer to the sentence, "The Nelson Mandela Rules 2015 issued by the UN and ...."
It clearly shows how the UN is concerned about the condition of prisoners around the world. Option (c) is
exactly the opposite of what the passage states.
FeedBack Bookmark

https://www.aspiration.ai/LAW/sis/Solution.jsp?qsetId=JAmrn2p8H1A=&qsetName=LST Mock 27 2021 (CLAT) 3/223


1/17/2021 Mock Analysis

Directions for questions 1 to 30: Each set of questions in this section is based on a single passage. Please
answer each question on the basis of what is stated or implied in the corresponding passage. In some
instances, more than one option may be the answer to the question; in such a case, please choose the option
that most accurately and comprehensively answers the question.

Passage – 1

Two recent incidents of Stan Swamy and Gautam Navlakha show up the state of our prisons today. Both are
undertrial prisoners in the Bhima-Koregaon case. Both had to move the courts for the simplest of necessities.
One, to get a sipper cup and a straw because he is an 80-year-old man with Parkinson's and the other to wrest a
pair of glasses to replace a broken pair. It led to the Bombay High Court into making an observation that
workshops should be conducted for the prison staff to sensitise them.

Why did these matters have to go to the courts? It is, perhaps, an indication of the fact that there is little space
for ground-level staff to make operational decisions. O cials do not want to get into trouble, especially in cases
of political prisoners arrested under national security laws.

The law on provision of basic facilities to prisoners is clear - it lies squarely with custodial authorities. The
Nelson Mandela Rules 2015 issued by the UN and the Model Prison Manual 2016 by the Bureau of Police
Research and Development, Ministry of Home Affairs, have elaborate provisions regarding the care, treatment
and rehabilitation of prisoners. A plethora of Supreme Court and high court judgments reiterate that prisoners
are human beings with basic rights.

Yet, violations are an everyday routine. They get highlighted when there is a mishap with a high-pro le person.
Notorious criminals, too, like Charles Sobhraj and Sunil Batra, who had the gumption to take matters to the
Supreme Court, have played their part in prison reforms over the years.

But standard setting doesn't mean compliance. Our prisons are full of people in compromised states. Around 70
per cent are under trials and more than 75 per cent come from marginalised sections. They know little about the
law. Even if they are aware, they have little recourse to any complaints mechanism and must fall back on an
uneven legal aid system. If at all a prisoner can reach the judge, they are either too busy in their routine work or
too enmeshed in the local culture to act. Overcrowding in individual prisons stands much higher; some prisons
are more than three times crowded than their o cial capacity.

Despite efforts at clearing cases, an acute shortage of judges and court infrastructure ensures an accelerating
accumulation of cases in courts. According to the India Justice report 2019, it takes an average of three years
for the case to traverse the high court and six years in the subordinate courts.

It doesn't have to be this way. There are pathways to accountability and to reform. But since both supervisor and
policy maker know the on-ground constraints and the biases within the culture, much is excused or pushed [1]
the carpet. Reform attempts like the constitution of Undertrial Review Committees mandated by the Supreme
Court work patchily. In most prisons, the Board of Visitors meant to function as an oversight mechanism are not
even constituted. The policy rhetoric wants prisons to be places of reform and rehabilitation. Their neglect by
the executive and oversight bodies ensures they act as warehouses for the poor and the marginalised.
Prolongation of these socalled criminal cases is unconscionable, as is forcing vulnerable people into remaining
in hotspots of increased infection and fatal risk.

https://www.aspiration.ai/LAW/sis/Solution.jsp?qsetId=JAmrn2p8H1A=&qsetName=LST Mock 27 2021 (CLAT) 4/223


1/17/2021 Mock Analysis

Q.2 [30465398]
The author of the passage is most likely to agree with which of the following statements:

a The elementary rights of prisoners are important.

b Only political prisoners are entitled to human rights.

c Terrorists who are arrested and imprisoned cannot stake claim to rights.

d The condition of Indian prisons is one of the worst in the world.

Solution:
 Answer key/Solution
Correct Answer : a
Your Answer : a
Refer to the sentence, "A plethora of Supreme Court and high court judgments
reiterate that prisoners are human beings with basic rights." Therefore, option (a) is correct. Option (d) is
wrong because in the passage, the author does not mention anything about the condition of world prisons.

FeedBack Bookmark

https://www.aspiration.ai/LAW/sis/Solution.jsp?qsetId=JAmrn2p8H1A=&qsetName=LST Mock 27 2021 (CLAT) 5/223


1/17/2021 Mock Analysis

Directions for questions 1 to 30: Each set of questions in this section is based on a single passage. Please
answer each question on the basis of what is stated or implied in the corresponding passage. In some
instances, more than one option may be the answer to the question; in such a case, please choose the option
that most accurately and comprehensively answers the question.

Passage – 1

Two recent incidents of Stan Swamy and Gautam Navlakha show up the state of our prisons today. Both are
undertrial prisoners in the Bhima-Koregaon case. Both had to move the courts for the simplest of necessities.
One, to get a sipper cup and a straw because he is an 80-year-old man with Parkinson's and the other to wrest a
pair of glasses to replace a broken pair. It led to the Bombay High Court into making an observation that
workshops should be conducted for the prison staff to sensitise them.

Why did these matters have to go to the courts? It is, perhaps, an indication of the fact that there is little space
for ground-level staff to make operational decisions. O cials do not want to get into trouble, especially in cases
of political prisoners arrested under national security laws.

The law on provision of basic facilities to prisoners is clear - it lies squarely with custodial authorities. The
Nelson Mandela Rules 2015 issued by the UN and the Model Prison Manual 2016 by the Bureau of Police
Research and Development, Ministry of Home Affairs, have elaborate provisions regarding the care, treatment
and rehabilitation of prisoners. A plethora of Supreme Court and high court judgments reiterate that prisoners
are human beings with basic rights.

Yet, violations are an everyday routine. They get highlighted when there is a mishap with a high-pro le person.
Notorious criminals, too, like Charles Sobhraj and Sunil Batra, who had the gumption to take matters to the
Supreme Court, have played their part in prison reforms over the years.

But standard setting doesn't mean compliance. Our prisons are full of people in compromised states. Around 70
per cent are under trials and more than 75 per cent come from marginalised sections. They know little about the
law. Even if they are aware, they have little recourse to any complaints mechanism and must fall back on an
uneven legal aid system. If at all a prisoner can reach the judge, they are either too busy in their routine work or
too enmeshed in the local culture to act. Overcrowding in individual prisons stands much higher; some prisons
are more than three times crowded than their o cial capacity.

Despite efforts at clearing cases, an acute shortage of judges and court infrastructure ensures an accelerating
accumulation of cases in courts. According to the India Justice report 2019, it takes an average of three years
for the case to traverse the high court and six years in the subordinate courts.

It doesn't have to be this way. There are pathways to accountability and to reform. But since both supervisor and
policy maker know the on-ground constraints and the biases within the culture, much is excused or pushed [1]
the carpet. Reform attempts like the constitution of Undertrial Review Committees mandated by the Supreme
Court work patchily. In most prisons, the Board of Visitors meant to function as an oversight mechanism are not
even constituted. The policy rhetoric wants prisons to be places of reform and rehabilitation. Their neglect by
the executive and oversight bodies ensures they act as warehouses for the poor and the marginalised.
Prolongation of these socalled criminal cases is unconscionable, as is forcing vulnerable people into remaining
in hotspots of increased infection and fatal risk.

https://www.aspiration.ai/LAW/sis/Solution.jsp?qsetId=JAmrn2p8H1A=&qsetName=LST Mock 27 2021 (CLAT) 6/223


1/17/2021 Mock Analysis

Q.3 [30465398]
Out of the following options, which one is a synonym of the word 'gumption' as used in the passage?

a Emaciation

b Remiss

c Lacklustre

d Resourcefulness

Solution:
 Answer key/Solution
Correct Answer : d
Your Answer : d
'Gumption' is a noun and means shrewd or 'spirited initiative and resourcefulness.'

FeedBack Bookmark

https://www.aspiration.ai/LAW/sis/Solution.jsp?qsetId=JAmrn2p8H1A=&qsetName=LST Mock 27 2021 (CLAT) 7/223


1/17/2021 Mock Analysis

Directions for questions 1 to 30: Each set of questions in this section is based on a single passage. Please
answer each question on the basis of what is stated or implied in the corresponding passage. In some
instances, more than one option may be the answer to the question; in such a case, please choose the option
that most accurately and comprehensively answers the question.

Passage – 1

Two recent incidents of Stan Swamy and Gautam Navlakha show up the state of our prisons today. Both are
undertrial prisoners in the Bhima-Koregaon case. Both had to move the courts for the simplest of necessities.
One, to get a sipper cup and a straw because he is an 80-year-old man with Parkinson's and the other to wrest a
pair of glasses to replace a broken pair. It led to the Bombay High Court into making an observation that
workshops should be conducted for the prison staff to sensitise them.

Why did these matters have to go to the courts? It is, perhaps, an indication of the fact that there is little space
for ground-level staff to make operational decisions. O cials do not want to get into trouble, especially in cases
of political prisoners arrested under national security laws.

The law on provision of basic facilities to prisoners is clear - it lies squarely with custodial authorities. The
Nelson Mandela Rules 2015 issued by the UN and the Model Prison Manual 2016 by the Bureau of Police
Research and Development, Ministry of Home Affairs, have elaborate provisions regarding the care, treatment
and rehabilitation of prisoners. A plethora of Supreme Court and high court judgments reiterate that prisoners
are human beings with basic rights.

Yet, violations are an everyday routine. They get highlighted when there is a mishap with a high-pro le person.
Notorious criminals, too, like Charles Sobhraj and Sunil Batra, who had the gumption to take matters to the
Supreme Court, have played their part in prison reforms over the years.

But standard setting doesn't mean compliance. Our prisons are full of people in compromised states. Around 70
per cent are under trials and more than 75 per cent come from marginalised sections. They know little about the
law. Even if they are aware, they have little recourse to any complaints mechanism and must fall back on an
uneven legal aid system. If at all a prisoner can reach the judge, they are either too busy in their routine work or
too enmeshed in the local culture to act. Overcrowding in individual prisons stands much higher; some prisons
are more than three times crowded than their o cial capacity.

Despite efforts at clearing cases, an acute shortage of judges and court infrastructure ensures an accelerating
accumulation of cases in courts. According to the India Justice report 2019, it takes an average of three years
for the case to traverse the high court and six years in the subordinate courts.

It doesn't have to be this way. There are pathways to accountability and to reform. But since both supervisor and
policy maker know the on-ground constraints and the biases within the culture, much is excused or pushed [1]
the carpet. Reform attempts like the constitution of Undertrial Review Committees mandated by the Supreme
Court work patchily. In most prisons, the Board of Visitors meant to function as an oversight mechanism are not
even constituted. The policy rhetoric wants prisons to be places of reform and rehabilitation. Their neglect by
the executive and oversight bodies ensures they act as warehouses for the poor and the marginalised.
Prolongation of these socalled criminal cases is unconscionable, as is forcing vulnerable people into remaining
in hotspots of increased infection and fatal risk.

https://www.aspiration.ai/LAW/sis/Solution.jsp?qsetId=JAmrn2p8H1A=&qsetName=LST Mock 27 2021 (CLAT) 8/223


1/17/2021 Mock Analysis

Q.4 [30465398]
Which of the following prepositions can suitably replace [1] in the passage?

a Under

b Above

c In

d On

Solution:
 Answer key/Solution
Correct Answer : a
Your Answer : a
'Push under the carpet' refers to hiding a problem or keeping it a secret.
FeedBack Bookmark

https://www.aspiration.ai/LAW/sis/Solution.jsp?qsetId=JAmrn2p8H1A=&qsetName=LST Mock 27 2021 (CLAT) 9/223


1/17/2021 Mock Analysis

Directions for questions 1 to 30: Each set of questions in this section is based on a single passage. Please
answer each question on the basis of what is stated or implied in the corresponding passage. In some
instances, more than one option may be the answer to the question; in such a case, please choose the option
that most accurately and comprehensively answers the question.

Passage – 1

Two recent incidents of Stan Swamy and Gautam Navlakha show up the state of our prisons today. Both are
undertrial prisoners in the Bhima-Koregaon case. Both had to move the courts for the simplest of necessities.
One, to get a sipper cup and a straw because he is an 80-year-old man with Parkinson's and the other to wrest a
pair of glasses to replace a broken pair. It led to the Bombay High Court into making an observation that
workshops should be conducted for the prison staff to sensitise them.

Why did these matters have to go to the courts? It is, perhaps, an indication of the fact that there is little space
for ground-level staff to make operational decisions. O cials do not want to get into trouble, especially in cases
of political prisoners arrested under national security laws.

The law on provision of basic facilities to prisoners is clear - it lies squarely with custodial authorities. The
Nelson Mandela Rules 2015 issued by the UN and the Model Prison Manual 2016 by the Bureau of Police
Research and Development, Ministry of Home Affairs, have elaborate provisions regarding the care, treatment
and rehabilitation of prisoners. A plethora of Supreme Court and high court judgments reiterate that prisoners
are human beings with basic rights.

Yet, violations are an everyday routine. They get highlighted when there is a mishap with a high-pro le person.
Notorious criminals, too, like Charles Sobhraj and Sunil Batra, who had the gumption to take matters to the
Supreme Court, have played their part in prison reforms over the years.

But standard setting doesn't mean compliance. Our prisons are full of people in compromised states. Around 70
per cent are under trials and more than 75 per cent come from marginalised sections. They know little about the
law. Even if they are aware, they have little recourse to any complaints mechanism and must fall back on an
uneven legal aid system. If at all a prisoner can reach the judge, they are either too busy in their routine work or
too enmeshed in the local culture to act. Overcrowding in individual prisons stands much higher; some prisons
are more than three times crowded than their o cial capacity.

Despite efforts at clearing cases, an acute shortage of judges and court infrastructure ensures an accelerating
accumulation of cases in courts. According to the India Justice report 2019, it takes an average of three years
for the case to traverse the high court and six years in the subordinate courts.

It doesn't have to be this way. There are pathways to accountability and to reform. But since both supervisor and
policy maker know the on-ground constraints and the biases within the culture, much is excused or pushed [1]
the carpet. Reform attempts like the constitution of Undertrial Review Committees mandated by the Supreme
Court work patchily. In most prisons, the Board of Visitors meant to function as an oversight mechanism are not
even constituted. The policy rhetoric wants prisons to be places of reform and rehabilitation. Their neglect by
the executive and oversight bodies ensures they act as warehouses for the poor and the marginalised.
Prolongation of these socalled criminal cases is unconscionable, as is forcing vulnerable people into remaining
in hotspots of increased infection and fatal risk.

https://www.aspiration.ai/LAW/sis/Solution.jsp?qsetId=JAmrn2p8H1A=&qsetName=LST Mock 27 2021 (CLAT) 10/223


1/17/2021 Mock Analysis

Q.5 [30465398]
In the light of the passage, it can be understood that:

a The minority sections of the society are mostly involved in petty crimes.

b The minority sections of the society consider themselves to be exploited by the judiciary.

c Legal awareness is absent among the minority sections of the society.

d The judiciary refuses to accord equal status to women in the country.

Solution:
 Answer key/Solution
Correct Answer : c
Your Answer : c
Refer to the sentence, "They know little about the law." Therefore, option (c) is
true. The remaining options cannot be considered as true in the light of the passage.
FeedBack Bookmark

https://www.aspiration.ai/LAW/sis/Solution.jsp?qsetId=JAmrn2p8H1A=&qsetName=LST Mock 27 2021 (CLAT) 11/223


1/17/2021 Mock Analysis

Directions for questions 1 to 30: Each set of questions in this section is based on a single passage. Please
answer each question on the basis of what is stated or implied in the corresponding passage. In some
instances, more than one option may be the answer to the question; in such a case, please choose the option
that most accurately and comprehensively answers the question.

Passage – 2

Let's face it: we've all had second thoughts about language. Hardly a day goes by when we don't stumble over
words, stagger into misunderstandings, or struggle with a double negative. It's a frightfully cumbersome way to
express ourselves. If language is such a slippery medium, perhaps it is time to replace it with something more
dependable. Why not cut out the middleman and connect brains directly? The idea is not new. As the American
physicist and Nobel laureate Murray Gell-Mann mused in The Quark and the Jaguar (1994): 'Thoughts and
feelings would be completely shared, with none of the selectivity or deception that language permits.'

It is useful to examine this view of language carefully, for it is quite alluring. Computer scientist Rajesh Rao and
his team complain about how hard it can be to verbalise feelings or forms of knowledge even if they are
introspectively available. On Twitter, Musk has described words as 'a very lossy compression of thought'. How
frustrating to have such a rich mental life and be stuck with such poor resources for expressing it! But no matter
how much we can sympathise with this view, it misses a few crucial insights about language. First, words are
tools. They can be misplaced or misused like any tool, but they are often useful for what they've been designed
to do: help us say just what we want to say, and no more. When we choose our words carefully, it is because we
know that there is a difference between private worlds and public words. There had better be, since social life
depends on it.

Second, and more subtly, this view sees language as merely a channel for information: just as the speaking tube
has made way for the telephone, so language can be done away with if we connect brains directly. This
overlooks that language is also an infrastructure for social action. Think of everyday conversations, in which we
riff off on a theme, recruit others to do stuff, relate to those around us. We don't just spout information
indiscriminately; we apportion our words in conversational turns and build on each other's contributions.
Language in everyday use is less like a channel and more like a tango: a uid interplay of moves in which people
can act as one, yet also retain their individuality. In social interaction there is room, by design, for consent and
dissent.

The difference with current concepts of brain-to-brain interfaces couldn't be greater. A transcranial magnetic
pulse leaves no room for doubt, but none for deliberation either. Its effect is as immediate as it is involuntary.
We can admire the sheer e ciency of this form of interaction, but we also have to admit that something is lost.
A sense of agency and autonomy; and along with that, perhaps even a sense of self. Nor does this problem go
away merely by upgrading bandwidth, as is Musk's ambition for Neuralink, his implantable brain-computer
interface. The very possibility of social (as opposed to merely symbiotic) life depends on there being some
separation of private worlds, along with powers to interact on our own terms. In other words, we need
something like language in order to be human.

Q.6 [30465398]
The author cites Nobel laureate Murray Gell-Mann in order to:

a To indicate that language can be selective or deceptive at times.

https://www.aspiration.ai/LAW/sis/Solution.jsp?qsetId=JAmrn2p8H1A=&qsetName=LST Mock 27 2021 (CLAT) 12/223


1/17/2021 Mock Analysis

b To show the possibility of a mechanism through which thoughts and feelings can be shared more
comprehensively.

c To demonstrate how Gell-Mann found the idea of sharing thoughts and feelings completely without using
language attractive.

d To show that the thought to eliminate language for expression of thoughts and connect brains directly is
not new.

Solution:
 Answer key/Solution
Correct Answer : d
Your Answer : b
The author cites Gell-man as an example to support his assertion that the idea
that brains be connected directly is not new. Hence, (d). (a) and (b) focus on the content of what Gell-man
said, and not on why the author cites him. (c) incorrectly connects the example of Gell-man to the beginning of
the second paragraph. The author has cited examples of Ramesh Rao and Musk to support the assertion
about "that vie of language" being alluring.
FeedBack Bookmark

https://www.aspiration.ai/LAW/sis/Solution.jsp?qsetId=JAmrn2p8H1A=&qsetName=LST Mock 27 2021 (CLAT) 13/223


1/17/2021 Mock Analysis

Directions for questions 1 to 30: Each set of questions in this section is based on a single passage. Please
answer each question on the basis of what is stated or implied in the corresponding passage. In some
instances, more than one option may be the answer to the question; in such a case, please choose the option
that most accurately and comprehensively answers the question.

Passage – 2

Let's face it: we've all had second thoughts about language. Hardly a day goes by when we don't stumble over
words, stagger into misunderstandings, or struggle with a double negative. It's a frightfully cumbersome way to
express ourselves. If language is such a slippery medium, perhaps it is time to replace it with something more
dependable. Why not cut out the middleman and connect brains directly? The idea is not new. As the American
physicist and Nobel laureate Murray Gell-Mann mused in The Quark and the Jaguar (1994): 'Thoughts and
feelings would be completely shared, with none of the selectivity or deception that language permits.'

It is useful to examine this view of language carefully, for it is quite alluring. Computer scientist Rajesh Rao and
his team complain about how hard it can be to verbalise feelings or forms of knowledge even if they are
introspectively available. On Twitter, Musk has described words as 'a very lossy compression of thought'. How
frustrating to have such a rich mental life and be stuck with such poor resources for expressing it! But no matter
how much we can sympathise with this view, it misses a few crucial insights about language. First, words are
tools. They can be misplaced or misused like any tool, but they are often useful for what they've been designed
to do: help us say just what we want to say, and no more. When we choose our words carefully, it is because we
know that there is a difference between private worlds and public words. There had better be, since social life
depends on it.

Second, and more subtly, this view sees language as merely a channel for information: just as the speaking tube
has made way for the telephone, so language can be done away with if we connect brains directly. This
overlooks that language is also an infrastructure for social action. Think of everyday conversations, in which we
riff off on a theme, recruit others to do stuff, relate to those around us. We don't just spout information
indiscriminately; we apportion our words in conversational turns and build on each other's contributions.
Language in everyday use is less like a channel and more like a tango: a uid interplay of moves in which people
can act as one, yet also retain their individuality. In social interaction there is room, by design, for consent and
dissent.

The difference with current concepts of brain-to-brain interfaces couldn't be greater. A transcranial magnetic
pulse leaves no room for doubt, but none for deliberation either. Its effect is as immediate as it is involuntary.
We can admire the sheer e ciency of this form of interaction, but we also have to admit that something is lost.
A sense of agency and autonomy; and along with that, perhaps even a sense of self. Nor does this problem go
away merely by upgrading bandwidth, as is Musk's ambition for Neuralink, his implantable brain-computer
interface. The very possibility of social (as opposed to merely symbiotic) life depends on there being some
separation of private worlds, along with powers to interact on our own terms. In other words, we need
something like language in order to be human.

Q.7 [30465398]
The author opines all of the following about language EXCEPT:

a It compresses thoughts so that they lose some of their essence.

b It can be misunderstood at times.

https://www.aspiration.ai/LAW/sis/Solution.jsp?qsetId=JAmrn2p8H1A=&qsetName=LST Mock 27 2021 (CLAT) 14/223


1/17/2021 Mock Analysis

c Some of its grammatical elements can be challenging.

d It can result in people expressing themselves in a confused manner.

Solution:
 Answer key/Solution
Correct Answer : a
Refer the second sentence of Para 1. The author implies (b), (c), and (d) in it. To
stumble over your words means to speak in a stuttering confused manner. (a) is
roughly the opinion of Elon Musk, not the author.

FeedBack Bookmark

https://www.aspiration.ai/LAW/sis/Solution.jsp?qsetId=JAmrn2p8H1A=&qsetName=LST Mock 27 2021 (CLAT) 15/223


1/17/2021 Mock Analysis

Directions for questions 1 to 30: Each set of questions in this section is based on a single passage. Please
answer each question on the basis of what is stated or implied in the corresponding passage. In some
instances, more than one option may be the answer to the question; in such a case, please choose the option
that most accurately and comprehensively answers the question.

Passage – 2

Let's face it: we've all had second thoughts about language. Hardly a day goes by when we don't stumble over
words, stagger into misunderstandings, or struggle with a double negative. It's a frightfully cumbersome way to
express ourselves. If language is such a slippery medium, perhaps it is time to replace it with something more
dependable. Why not cut out the middleman and connect brains directly? The idea is not new. As the American
physicist and Nobel laureate Murray Gell-Mann mused in The Quark and the Jaguar (1994): 'Thoughts and
feelings would be completely shared, with none of the selectivity or deception that language permits.'

It is useful to examine this view of language carefully, for it is quite alluring. Computer scientist Rajesh Rao and
his team complain about how hard it can be to verbalise feelings or forms of knowledge even if they are
introspectively available. On Twitter, Musk has described words as 'a very lossy compression of thought'. How
frustrating to have such a rich mental life and be stuck with such poor resources for expressing it! But no matter
how much we can sympathise with this view, it misses a few crucial insights about language. First, words are
tools. They can be misplaced or misused like any tool, but they are often useful for what they've been designed
to do: help us say just what we want to say, and no more. When we choose our words carefully, it is because we
know that there is a difference between private worlds and public words. There had better be, since social life
depends on it.

Second, and more subtly, this view sees language as merely a channel for information: just as the speaking tube
has made way for the telephone, so language can be done away with if we connect brains directly. This
overlooks that language is also an infrastructure for social action. Think of everyday conversations, in which we
riff off on a theme, recruit others to do stuff, relate to those around us. We don't just spout information
indiscriminately; we apportion our words in conversational turns and build on each other's contributions.
Language in everyday use is less like a channel and more like a tango: a uid interplay of moves in which people
can act as one, yet also retain their individuality. In social interaction there is room, by design, for consent and
dissent.

The difference with current concepts of brain-to-brain interfaces couldn't be greater. A transcranial magnetic
pulse leaves no room for doubt, but none for deliberation either. Its effect is as immediate as it is involuntary.
We can admire the sheer e ciency of this form of interaction, but we also have to admit that something is lost.
A sense of agency and autonomy; and along with that, perhaps even a sense of self. Nor does this problem go
away merely by upgrading bandwidth, as is Musk's ambition for Neuralink, his implantable brain-computer
interface. The very possibility of social (as opposed to merely symbiotic) life depends on there being some
separation of private worlds, along with powers to interact on our own terms. In other words, we need
something like language in order to be human.

Q.8 [30465398]
What does the author allude to when they say, "When we choose our words carefully, it is because we know that
there is a difference between private worlds and public words."

a Choosing words carefully is not a drawback of language, it is rather an imperative imposed upon us by
societal norms.

https://www.aspiration.ai/LAW/sis/Solution.jsp?qsetId=JAmrn2p8H1A=&qsetName=LST Mock 27 2021 (CLAT) 16/223


1/17/2021 Mock Analysis

b A careful use of words re ects the choice to not disclose all our inner thoughts to others, rather than the
inadequacy of resources for expression.

c Choosing our words carefully shows our desire to have an active social life, apart from a private life, rather
than showing a drawback of language.

d A person's careful use of words simply re ects their maturity as they become more conscious of the
difference between their private world and their social life.

Solution:
 Answer key/Solution
Correct Answer : b
Your Answer : c
Refer Para 2. The author says that words help us say what we want to say, and no
more. This coupled with the statement in the question, makes (b) the answer. The author makes these
statements while defending language, implying that a careful choice of words is not a 'drawback'. (a) is
eliminated because it is a 'choice', not an imperative, and societal 'norms' haven't been discussed. (c) misses
the point of choosing our words carefully. 'Active' social life hasn't been discussed. (d) is incorrect because of
'more' and incomplete because it does not talk about language not being a drawback.
FeedBack Bookmark

https://www.aspiration.ai/LAW/sis/Solution.jsp?qsetId=JAmrn2p8H1A=&qsetName=LST Mock 27 2021 (CLAT) 17/223


1/17/2021 Mock Analysis

Directions for questions 1 to 30: Each set of questions in this section is based on a single passage. Please
answer each question on the basis of what is stated or implied in the corresponding passage. In some
instances, more than one option may be the answer to the question; in such a case, please choose the option
that most accurately and comprehensively answers the question.

Passage – 2

Let's face it: we've all had second thoughts about language. Hardly a day goes by when we don't stumble over
words, stagger into misunderstandings, or struggle with a double negative. It's a frightfully cumbersome way to
express ourselves. If language is such a slippery medium, perhaps it is time to replace it with something more
dependable. Why not cut out the middleman and connect brains directly? The idea is not new. As the American
physicist and Nobel laureate Murray Gell-Mann mused in The Quark and the Jaguar (1994): 'Thoughts and
feelings would be completely shared, with none of the selectivity or deception that language permits.'

It is useful to examine this view of language carefully, for it is quite alluring. Computer scientist Rajesh Rao and
his team complain about how hard it can be to verbalise feelings or forms of knowledge even if they are
introspectively available. On Twitter, Musk has described words as 'a very lossy compression of thought'. How
frustrating to have such a rich mental life and be stuck with such poor resources for expressing it! But no matter
how much we can sympathise with this view, it misses a few crucial insights about language. First, words are
tools. They can be misplaced or misused like any tool, but they are often useful for what they've been designed
to do: help us say just what we want to say, and no more. When we choose our words carefully, it is because we
know that there is a difference between private worlds and public words. There had better be, since social life
depends on it.

Second, and more subtly, this view sees language as merely a channel for information: just as the speaking tube
has made way for the telephone, so language can be done away with if we connect brains directly. This
overlooks that language is also an infrastructure for social action. Think of everyday conversations, in which we
riff off on a theme, recruit others to do stuff, relate to those around us. We don't just spout information
indiscriminately; we apportion our words in conversational turns and build on each other's contributions.
Language in everyday use is less like a channel and more like a tango: a uid interplay of moves in which people
can act as one, yet also retain their individuality. In social interaction there is room, by design, for consent and
dissent.

The difference with current concepts of brain-to-brain interfaces couldn't be greater. A transcranial magnetic
pulse leaves no room for doubt, but none for deliberation either. Its effect is as immediate as it is involuntary.
We can admire the sheer e ciency of this form of interaction, but we also have to admit that something is lost.
A sense of agency and autonomy; and along with that, perhaps even a sense of self. Nor does this problem go
away merely by upgrading bandwidth, as is Musk's ambition for Neuralink, his implantable brain-computer
interface. The very possibility of social (as opposed to merely symbiotic) life depends on there being some
separation of private worlds, along with powers to interact on our own terms. In other words, we need
something like language in order to be human.

Q.9 [30465398]
Which of the following aspects of the current concepts of brain-to-brain interfaces makes them more like a
"channel" and less like a "tango"?

a Its e ciency

https://www.aspiration.ai/LAW/sis/Solution.jsp?qsetId=JAmrn2p8H1A=&qsetName=LST Mock 27 2021 (CLAT) 18/223


1/17/2021 Mock Analysis

b Its immediacy

c Its involuntariness

d Its uidity

Solution:
 Answer key/Solution
Correct Answer : c
Your Answer : d
The author calls language more like a tango and less like a channel because it
allows individuals to retain their individuality even after enabling them to act as one. He then contrasts
language with the brain-to-brain interfaces. Here he points out how they can result in a loss of autonomy,
agency and self. All this points to (c) as the answer. Fluidity is rejected as even language has been mentioned
as a uid interplay. Immediacy and e ciency, although features of these interfaces, are not the main points of
contrast.

FeedBack Bookmark

https://www.aspiration.ai/LAW/sis/Solution.jsp?qsetId=JAmrn2p8H1A=&qsetName=LST Mock 27 2021 (CLAT) 19/223


1/17/2021 Mock Analysis

Directions for questions 1 to 30: Each set of questions in this section is based on a single passage. Please
answer each question on the basis of what is stated or implied in the corresponding passage. In some
instances, more than one option may be the answer to the question; in such a case, please choose the option
that most accurately and comprehensively answers the question.

Passage – 2

Let's face it: we've all had second thoughts about language. Hardly a day goes by when we don't stumble over
words, stagger into misunderstandings, or struggle with a double negative. It's a frightfully cumbersome way to
express ourselves. If language is such a slippery medium, perhaps it is time to replace it with something more
dependable. Why not cut out the middleman and connect brains directly? The idea is not new. As the American
physicist and Nobel laureate Murray Gell-Mann mused in The Quark and the Jaguar (1994): 'Thoughts and
feelings would be completely shared, with none of the selectivity or deception that language permits.'

It is useful to examine this view of language carefully, for it is quite alluring. Computer scientist Rajesh Rao and
his team complain about how hard it can be to verbalise feelings or forms of knowledge even if they are
introspectively available. On Twitter, Musk has described words as 'a very lossy compression of thought'. How
frustrating to have such a rich mental life and be stuck with such poor resources for expressing it! But no matter
how much we can sympathise with this view, it misses a few crucial insights about language. First, words are
tools. They can be misplaced or misused like any tool, but they are often useful for what they've been designed
to do: help us say just what we want to say, and no more. When we choose our words carefully, it is because we
know that there is a difference between private worlds and public words. There had better be, since social life
depends on it.

Second, and more subtly, this view sees language as merely a channel for information: just as the speaking tube
has made way for the telephone, so language can be done away with if we connect brains directly. This
overlooks that language is also an infrastructure for social action. Think of everyday conversations, in which we
riff off on a theme, recruit others to do stuff, relate to those around us. We don't just spout information
indiscriminately; we apportion our words in conversational turns and build on each other's contributions.
Language in everyday use is less like a channel and more like a tango: a uid interplay of moves in which people
can act as one, yet also retain their individuality. In social interaction there is room, by design, for consent and
dissent.

The difference with current concepts of brain-to-brain interfaces couldn't be greater. A transcranial magnetic
pulse leaves no room for doubt, but none for deliberation either. Its effect is as immediate as it is involuntary.
We can admire the sheer e ciency of this form of interaction, but we also have to admit that something is lost.
A sense of agency and autonomy; and along with that, perhaps even a sense of self. Nor does this problem go
away merely by upgrading bandwidth, as is Musk's ambition for Neuralink, his implantable brain-computer
interface. The very possibility of social (as opposed to merely symbiotic) life depends on there being some
separation of private worlds, along with powers to interact on our own terms. In other words, we need
something like language in order to be human.

Q.10 [30465398]
Based on the features of the current brain-to-brain interfaces mentioned in the last paragraph, in which of the
following scenarios would their use be the most suitable for communication?

a A discussion between team members on how to proceed with a new project.

https://www.aspiration.ai/LAW/sis/Solution.jsp?qsetId=JAmrn2p8H1A=&qsetName=LST Mock 27 2021 (CLAT) 20/223


1/17/2021 Mock Analysis

b An exchange between a military commander and his junior in a war-like situation that demands clear,
immediate, and carefully thought out commands.

c An exchange between an art connoisseur and a painter where the connoisseur wants the painter to
interpret and paint the connoisseur's un ltered impulses.

d An exchange between a couple that wants a quick and lasting resolution of their differences.

Solution:
 Answer key/Solution
Correct Answer : c
(a) is rejected because a discussion about a new project would need deliberation
and not hasty exchanges. (b) is rejected because it mentions that the commands
have to be 'carefully thought'. (d) is rejected because the couple, in order to arrive at a lasting resolution,
would want to deliberate, negotiate and both the participants would not want to lose their agency. (c) is the
best choice as the connoisseur wants to convey his 'un ltered impulses' to the painter, and words may distort
them.

FeedBack Bookmark

https://www.aspiration.ai/LAW/sis/Solution.jsp?qsetId=JAmrn2p8H1A=&qsetName=LST Mock 27 2021 (CLAT) 21/223


1/17/2021 Mock Analysis

Directions for questions 1 to 30: Each set of questions in this section is based on a single passage. Please
answer each question on the basis of what is stated or implied in the corresponding passage. In some
instances, more than one option may be the answer to the question; in such a case, please choose the option
that most accurately and comprehensively answers the question.

Passage – 3

Karnataka has a dismal record in implementing the Forest Rights Act compared to other States. According to
the Ministry of Tribal Affairs, as of April 30, 2018, the State had recognised only 5.7% of the total claims made.
Notably, 70% of the claims were disposed off. There appeared to be clear inconsistency in the government's
approach in settling the claims made by the tribals versus the claims made by other traditional forest dwellers.
The inconsistency re ected in their argument. According to them, tribal applications constituted 17.5% of the
claims and nearly all of them were settled, while other claims were rejected as they were not backed by valid
evidence. This means that claims made by other traditional forest dwellers were treated as inconsequential.

Assuming that denying tribals or other traditional forest dwellers their rights in the forest would serve the
purpose of conservation is far from the truth. The Forest Rights Act is not about the indiscriminate distribution
of forest land to anyone applying for it. As per the law, only those lands are recognised where people prove their
occupation not later than December 13, 2005. Moreover, the combined stretch of land claimed by them is
comparatively smaller by any account than what has been taken away for building dams, mining, laying railway
lines and roads, power plants, etc. The government records also reveal that 43 lakh hectares of forestland were
encroached both legally and illegally until 1980 when the Forest Conservation Act came into force. Sadly, there
is no signi cant conservation even after this landmark law.

Invariably, an approach adopted to isolate the indigenous people from their natural habitats to protect
biodiversity is the root cause of con ict between them and conservationists. The latter think that resources
have to be controlled and managed. However, this theory is fast proving unproductive. The Global Environment
Outlook Report 5 mentions that there is decreased biodiversity across the globe even as 'protected areas' have
been expanding. People living in nature's surroundings are integral to conservation as they relate with it in a
more integrated and spiritual way.

Declaration of the Western Ghats as a World Heritage Site is as important in preserving the rich biodiversity of
the region as the recognition of the rights of the people who depend on the forests. As con rmed internationally,
preserving biodiversity requires the legal empowerment of the people living in those areas. The Forest Rights
Act is an ideal instrument to push [1] the objective. To realise it on the ground, the government must make an
effort to build trust between its agencies in the area and the people who depend on these forests by treating
them as equal citizens like everyone else in the country.

Q.11 [30465398]
Which of the following is the main idea of the given passage?

a The people who depend on the forests should not be deprived of their rights.

b Forests dwellers in Karnataka are denied their rights to live in the forests.

c The Forest Rights Act grants rights to the tribals and other forest dwellers to live in the forests.

d Biodiversity of a place can be preserved through the intervention of the conservationists.

https://www.aspiration.ai/LAW/sis/Solution.jsp?qsetId=JAmrn2p8H1A=&qsetName=LST Mock 27 2021 (CLAT) 22/223


1/17/2021 Mock Analysis

Solution:
 Answer key/Solution
Correct Answer : a
Your Answer : b
The given passage is about the Forest Rights Act and need to protect the rights of
people who depend on the forests. Refer to "Assuming that denying tribals or other traditional forest dwellers
their rights in the forest would serve the purpose of conservation is far from the truth." Also refer to the last
paragraph of the given passage. Hence, option (a) is the answer. Options (b), (c) and (d) are not the main idea
of the given passage.

FeedBack Bookmark

https://www.aspiration.ai/LAW/sis/Solution.jsp?qsetId=JAmrn2p8H1A=&qsetName=LST Mock 27 2021 (CLAT) 23/223


1/17/2021 Mock Analysis

Directions for questions 1 to 30: Each set of questions in this section is based on a single passage. Please
answer each question on the basis of what is stated or implied in the corresponding passage. In some
instances, more than one option may be the answer to the question; in such a case, please choose the option
that most accurately and comprehensively answers the question.

Passage – 3

Karnataka has a dismal record in implementing the Forest Rights Act compared to other States. According to
the Ministry of Tribal Affairs, as of April 30, 2018, the State had recognised only 5.7% of the total claims made.
Notably, 70% of the claims were disposed off. There appeared to be clear inconsistency in the government's
approach in settling the claims made by the tribals versus the claims made by other traditional forest dwellers.
The inconsistency re ected in their argument. According to them, tribal applications constituted 17.5% of the
claims and nearly all of them were settled, while other claims were rejected as they were not backed by valid
evidence. This means that claims made by other traditional forest dwellers were treated as inconsequential.

Assuming that denying tribals or other traditional forest dwellers their rights in the forest would serve the
purpose of conservation is far from the truth. The Forest Rights Act is not about the indiscriminate distribution
of forest land to anyone applying for it. As per the law, only those lands are recognised where people prove their
occupation not later than December 13, 2005. Moreover, the combined stretch of land claimed by them is
comparatively smaller by any account than what has been taken away for building dams, mining, laying railway
lines and roads, power plants, etc. The government records also reveal that 43 lakh hectares of forestland were
encroached both legally and illegally until 1980 when the Forest Conservation Act came into force. Sadly, there
is no signi cant conservation even after this landmark law.

Invariably, an approach adopted to isolate the indigenous people from their natural habitats to protect
biodiversity is the root cause of con ict between them and conservationists. The latter think that resources
have to be controlled and managed. However, this theory is fast proving unproductive. The Global Environment
Outlook Report 5 mentions that there is decreased biodiversity across the globe even as 'protected areas' have
been expanding. People living in nature's surroundings are integral to conservation as they relate with it in a
more integrated and spiritual way.

Declaration of the Western Ghats as a World Heritage Site is as important in preserving the rich biodiversity of
the region as the recognition of the rights of the people who depend on the forests. As con rmed internationally,
preserving biodiversity requires the legal empowerment of the people living in those areas. The Forest Rights
Act is an ideal instrument to push [1] the objective. To realise it on the ground, the government must make an
effort to build trust between its agencies in the area and the people who depend on these forests by treating
them as equal citizens like everyone else in the country.

Q.12 [30465398]
In the light of the given passage all of the following are false EXCEPT:

a Karnataka was the state with the least record of the implementation of the Forest Rights Act in India.

b Karnataka was the state with the highest record of the implementation of the Forest Rights Act in India.

c Karnataka was the state with a high record of the implementation of the Forest Rights Act in India.

d Karnataka was the state with a very low record of the implementation of the Forest Rights Act in India.

https://www.aspiration.ai/LAW/sis/Solution.jsp?qsetId=JAmrn2p8H1A=&qsetName=LST Mock 27 2021 (CLAT) 24/223


1/17/2021 Mock Analysis

Solution:
 Answer key/Solution
Correct Answer : d
Your Answer : d
Only option (d) is true. Hence, it is the answer. Refer to "Karnataka has a dismal
record in implementing the Forest Rights Act compared to other States." Options (a), (b) and (c) are not stated
in the given passage.

FeedBack Bookmark

https://www.aspiration.ai/LAW/sis/Solution.jsp?qsetId=JAmrn2p8H1A=&qsetName=LST Mock 27 2021 (CLAT) 25/223


1/17/2021 Mock Analysis

Directions for questions 1 to 30: Each set of questions in this section is based on a single passage. Please
answer each question on the basis of what is stated or implied in the corresponding passage. In some
instances, more than one option may be the answer to the question; in such a case, please choose the option
that most accurately and comprehensively answers the question.

Passage – 3

Karnataka has a dismal record in implementing the Forest Rights Act compared to other States. According to
the Ministry of Tribal Affairs, as of April 30, 2018, the State had recognised only 5.7% of the total claims made.
Notably, 70% of the claims were disposed off. There appeared to be clear inconsistency in the government's
approach in settling the claims made by the tribals versus the claims made by other traditional forest dwellers.
The inconsistency re ected in their argument. According to them, tribal applications constituted 17.5% of the
claims and nearly all of them were settled, while other claims were rejected as they were not backed by valid
evidence. This means that claims made by other traditional forest dwellers were treated as inconsequential.

Assuming that denying tribals or other traditional forest dwellers their rights in the forest would serve the
purpose of conservation is far from the truth. The Forest Rights Act is not about the indiscriminate distribution
of forest land to anyone applying for it. As per the law, only those lands are recognised where people prove their
occupation not later than December 13, 2005. Moreover, the combined stretch of land claimed by them is
comparatively smaller by any account than what has been taken away for building dams, mining, laying railway
lines and roads, power plants, etc. The government records also reveal that 43 lakh hectares of forestland were
encroached both legally and illegally until 1980 when the Forest Conservation Act came into force. Sadly, there
is no signi cant conservation even after this landmark law.

Invariably, an approach adopted to isolate the indigenous people from their natural habitats to protect
biodiversity is the root cause of con ict between them and conservationists. The latter think that resources
have to be controlled and managed. However, this theory is fast proving unproductive. The Global Environment
Outlook Report 5 mentions that there is decreased biodiversity across the globe even as 'protected areas' have
been expanding. People living in nature's surroundings are integral to conservation as they relate with it in a
more integrated and spiritual way.

Declaration of the Western Ghats as a World Heritage Site is as important in preserving the rich biodiversity of
the region as the recognition of the rights of the people who depend on the forests. As con rmed internationally,
preserving biodiversity requires the legal empowerment of the people living in those areas. The Forest Rights
Act is an ideal instrument to push [1] the objective. To realise it on the ground, the government must make an
effort to build trust between its agencies in the area and the people who depend on these forests by treating
them as equal citizens like everyone else in the country.

Q.13 [30465398]
According to the given passage, which of the following options would be regarded as wrong approaches to the
conservation of the forest?

(i) Granting rights only to tribals to inhabit the forests and denying these rights to other traditional forest
dwellers.
(ii) Expanding the protected areas and allowing tribals and traditional forest dwellers to inhabit those areas.
(iii) Isolating the indigenous people from their natural habitats in the forests to protect biodiversity.
(iv) Granting rights only to traditional forest dwellers to live in the forests and denying these rights to tribals.

https://www.aspiration.ai/LAW/sis/Solution.jsp?qsetId=JAmrn2p8H1A=&qsetName=LST Mock 27 2021 (CLAT) 26/223


1/17/2021 Mock Analysis

a (i), (ii) and (iii)

b (i) and (iii)

c (i), (iii) and (iv)

d (iii) and (iv)

Solution:
 Answer key/Solution
Correct Answer : c
Your Answer : c
Denying rights to traditional forest dwellers to inhabit the forest would be a wrong
approach to conserve the forest. Isolating the indigenous people from their natural habitats in the forests to
protect biodiversity would also be a wrong approach to conserve the forest. Also, denying rights to tribals to
inhabit the forest would be a wrong approach to conserve the forest. So, option (c) is the answer. (ii) would be
the right approach to conserve the forest. Refer to "People living in nature's surroundings are integral to
conservation as they relate with it in a more integrated and spiritual way. "

FeedBack Bookmark

https://www.aspiration.ai/LAW/sis/Solution.jsp?qsetId=JAmrn2p8H1A=&qsetName=LST Mock 27 2021 (CLAT) 27/223


1/17/2021 Mock Analysis

Directions for questions 1 to 30: Each set of questions in this section is based on a single passage. Please
answer each question on the basis of what is stated or implied in the corresponding passage. In some
instances, more than one option may be the answer to the question; in such a case, please choose the option
that most accurately and comprehensively answers the question.

Passage – 3

Karnataka has a dismal record in implementing the Forest Rights Act compared to other States. According to
the Ministry of Tribal Affairs, as of April 30, 2018, the State had recognised only 5.7% of the total claims made.
Notably, 70% of the claims were disposed off. There appeared to be clear inconsistency in the government's
approach in settling the claims made by the tribals versus the claims made by other traditional forest dwellers.
The inconsistency re ected in their argument. According to them, tribal applications constituted 17.5% of the
claims and nearly all of them were settled, while other claims were rejected as they were not backed by valid
evidence. This means that claims made by other traditional forest dwellers were treated as inconsequential.

Assuming that denying tribals or other traditional forest dwellers their rights in the forest would serve the
purpose of conservation is far from the truth. The Forest Rights Act is not about the indiscriminate distribution
of forest land to anyone applying for it. As per the law, only those lands are recognised where people prove their
occupation not later than December 13, 2005. Moreover, the combined stretch of land claimed by them is
comparatively smaller by any account than what has been taken away for building dams, mining, laying railway
lines and roads, power plants, etc. The government records also reveal that 43 lakh hectares of forestland were
encroached both legally and illegally until 1980 when the Forest Conservation Act came into force. Sadly, there
is no signi cant conservation even after this landmark law.

Invariably, an approach adopted to isolate the indigenous people from their natural habitats to protect
biodiversity is the root cause of con ict between them and conservationists. The latter think that resources
have to be controlled and managed. However, this theory is fast proving unproductive. The Global Environment
Outlook Report 5 mentions that there is decreased biodiversity across the globe even as 'protected areas' have
been expanding. People living in nature's surroundings are integral to conservation as they relate with it in a
more integrated and spiritual way.

Declaration of the Western Ghats as a World Heritage Site is as important in preserving the rich biodiversity of
the region as the recognition of the rights of the people who depend on the forests. As con rmed internationally,
preserving biodiversity requires the legal empowerment of the people living in those areas. The Forest Rights
Act is an ideal instrument to push [1] the objective. To realise it on the ground, the government must make an
effort to build trust between its agencies in the area and the people who depend on these forests by treating
them as equal citizens like everyone else in the country.

Q.14 [30465398]
Choose the correct preposition in place of '[1]' from the following:

a unto

b forward

c against

d for

https://www.aspiration.ai/LAW/sis/Solution.jsp?qsetId=JAmrn2p8H1A=&qsetName=LST Mock 27 2021 (CLAT) 28/223


1/17/2021 Mock Analysis

Solution:
 Answer key/Solution
Correct Answer : b
Your Answer : b
'Push forward' means to accelerate or advance. Thus, option (b) is the correct
answer. Other options are grammatically incorrect in the given context.

FeedBack Bookmark

https://www.aspiration.ai/LAW/sis/Solution.jsp?qsetId=JAmrn2p8H1A=&qsetName=LST Mock 27 2021 (CLAT) 29/223


1/17/2021 Mock Analysis

Directions for questions 1 to 30: Each set of questions in this section is based on a single passage. Please
answer each question on the basis of what is stated or implied in the corresponding passage. In some
instances, more than one option may be the answer to the question; in such a case, please choose the option
that most accurately and comprehensively answers the question.

Passage – 3

Karnataka has a dismal record in implementing the Forest Rights Act compared to other States. According to
the Ministry of Tribal Affairs, as of April 30, 2018, the State had recognised only 5.7% of the total claims made.
Notably, 70% of the claims were disposed off. There appeared to be clear inconsistency in the government's
approach in settling the claims made by the tribals versus the claims made by other traditional forest dwellers.
The inconsistency re ected in their argument. According to them, tribal applications constituted 17.5% of the
claims and nearly all of them were settled, while other claims were rejected as they were not backed by valid
evidence. This means that claims made by other traditional forest dwellers were treated as inconsequential.

Assuming that denying tribals or other traditional forest dwellers their rights in the forest would serve the
purpose of conservation is far from the truth. The Forest Rights Act is not about the indiscriminate distribution
of forest land to anyone applying for it. As per the law, only those lands are recognised where people prove their
occupation not later than December 13, 2005. Moreover, the combined stretch of land claimed by them is
comparatively smaller by any account than what has been taken away for building dams, mining, laying railway
lines and roads, power plants, etc. The government records also reveal that 43 lakh hectares of forestland were
encroached both legally and illegally until 1980 when the Forest Conservation Act came into force. Sadly, there
is no signi cant conservation even after this landmark law.

Invariably, an approach adopted to isolate the indigenous people from their natural habitats to protect
biodiversity is the root cause of con ict between them and conservationists. The latter think that resources
have to be controlled and managed. However, this theory is fast proving unproductive. The Global Environment
Outlook Report 5 mentions that there is decreased biodiversity across the globe even as 'protected areas' have
been expanding. People living in nature's surroundings are integral to conservation as they relate with it in a
more integrated and spiritual way.

Declaration of the Western Ghats as a World Heritage Site is as important in preserving the rich biodiversity of
the region as the recognition of the rights of the people who depend on the forests. As con rmed internationally,
preserving biodiversity requires the legal empowerment of the people living in those areas. The Forest Rights
Act is an ideal instrument to push [1] the objective. To realise it on the ground, the government must make an
effort to build trust between its agencies in the area and the people who depend on these forests by treating
them as equal citizens like everyone else in the country.

Q.15 [30465398]
It can be inferred from the given passage that the effort to conserve the biodiversity of the forest:

a is possible only when tribals and other traditional forest dwellers are allowed to live in the forest.

b will prove futile unless tribals and other traditional forest dwellers are evicted from the forest.

c is possible only when tribals and other forest dwellers are shifted to towns and cities.

https://www.aspiration.ai/LAW/sis/Solution.jsp?qsetId=JAmrn2p8H1A=&qsetName=LST Mock 27 2021 (CLAT) 30/223


1/17/2021 Mock Analysis

d will be possible only when tribals and other forest dwellers are educated about the importance of the
biodiversity of the forest.

Solution:
 Answer key/Solution
Correct Answer : a
Your Answer : a
Option (a) can be inferred from "People living in nature's surroundings are integral
to conservation as they relate with it in a more integrated and spiritual way." Options (b), (c) and (d) cannot be
inferred from the given passage.
FeedBack Bookmark

Directions for questions 1 to 30: Each set of questions in this section is based on a single passage. Please
answer each question on the basis of what is stated or implied in the corresponding passage. In some
instances, more than one option may be the answer to the question; in such a case, please choose the option
that most accurately and comprehensively answers the question.

Passage – 4

India currently consumes only one-third of the global average consumption of energy. As we move towards
more robust energy consumption gures, the country faces the unique challenge of fusing together disparate
needs: A ballooning appetite while cutting dependence on imports; greening the grid while ensuring
affordability; and replacing old forms of energy production while boosting employment, and the human and
economic capital of the people. Then there's the fact that the energy sector is closely intertwined with issues
related to climate change.

In this context, bio-fuels have become a tool for achieving these delicate balances of outcomes. In the past few
years, progress made in the use of ethanol, compressed biogas and biodiesel - all different forms of bio-fuel -
will have a direct positive impact on both farm incomes (and the prosperity of agricultural communities), even
as it cuts down our import dependence for energy.

In 2019, in a rst for the Republic Day parade, the Indian Air Force ew aircraft in a Vic formation, with the lead
plane using a mix of traditional fuel and bio-fuel, symbolising the government's determination to seek alternative
sources of fuel. The primary raw material for ethanol production in India is sugarcane and its by-products,
accounting for more than 90% of fuel ethanol supplies under the ethanol blended petrol (EBP) programme of the
ministry. This programme injects liquidity into the stressed sugar sector and provides an alternative revenue
stream to farmers. This programme also encourages diversion of sugarcane for ethanol production, which
results in a decrease in the sugar glut in the country.

Offers of about 3.5 billion litres from both sugar/molasses and grain-based distilleries are expected this year. In
addition to sugarcane, ethanol is also produced from damaged food grains, B-heavy molasses and sugarcane
juice. This translates into a sum of nearly Rs 35,000 crore in the past six years - money that has own back to
farmers through sugar mills and distilleries as oil marketing companies (OMCs) provide off-take guarantee at
xed prices. This arrangement also improves the health of the payment cycle to farmers since OMCs settle their

https://www.aspiration.ai/LAW/sis/Solution.jsp?qsetId=JAmrn2p8H1A=&qsetName=LST Mock 27 2021 (CLAT) 31/223


1/17/2021 Mock Analysis

ethanol dues to distilleries in 21 days instead of the months that the farmers had to wait for their payment from
sugar mills.

The policy encourages setting up of supply chain mechanisms for biodiesel production from non-edible
oilseeds, used cooking oil and short-gestation crops. These crops can be easily cultivated in various states on
land that is barren or not t for edible crops, thus boosting farm incomes.

The Sustainable Alternative Towards Affordable Transportation (SATAT) scheme launched in October 2018
aims to establish an ecosystem for production of compressed biogas (CBG) from various waste biomass
sources in the country. This initiative offers a potential for investment of about Rs 1.75 lakh crore, generating
about 75,000 direct employment opportunities.

Many of the proposed plants will use crop residue such as paddy straw and biomass as feedstock for
production of CBG, especially in Haryana, Punjab and Uttar Pradesh. The SATAT scheme will not only stanch
greenhouse gas emissions, but will reduce burning of agricultural residue, which results in signi cant air
pollution in cities like Delhi, generate employment in rural and waste management sectors, and boost income
for farmers from their unutilised organic waste. One of the by-products of CBG plants is bio-manure, which can
be used in farming.

Q.16 [30465398]
Out of the following options, which one can be a suitable title of the given passage?

a How bio-fuels can negate carbon emission

b How bio-fuels can augment farm income

c How bio-fuels can be exported to other countries

d How bio-fuels can boost the defense sector

Solution:
 Answer key/Solution
Correct Answer : b
Your Answer : b
The passage talks about the importance of bio-fuel and its role in augmenting
farm income. Therefore, option (b) is the most appropriate title of the passage. Refer to the last 3 paragraphs
of the passage. The other options are not the core points of discussion in the passage.

FeedBack Bookmark

Directions for questions 1 to 30: Each set of questions in this section is based on a single passage. Please
answer each question on the basis of what is stated or implied in the corresponding passage. In some
instances, more than one option may be the answer to the question; in such a case, please choose the option
that most accurately and comprehensively answers the question.

Passage – 4

https://www.aspiration.ai/LAW/sis/Solution.jsp?qsetId=JAmrn2p8H1A=&qsetName=LST Mock 27 2021 (CLAT) 32/223


1/17/2021 Mock Analysis

India currently consumes only one-third of the global average consumption of energy. As we move towards
more robust energy consumption gures, the country faces the unique challenge of fusing together disparate
needs: A ballooning appetite while cutting dependence on imports; greening the grid while ensuring
affordability; and replacing old forms of energy production while boosting employment, and the human and
economic capital of the people. Then there's the fact that the energy sector is closely intertwined with issues
related to climate change.

In this context, bio-fuels have become a tool for achieving these delicate balances of outcomes. In the past few
years, progress made in the use of ethanol, compressed biogas and biodiesel - all different forms of bio-fuel -
will have a direct positive impact on both farm incomes (and the prosperity of agricultural communities), even
as it cuts down our import dependence for energy.

In 2019, in a rst for the Republic Day parade, the Indian Air Force ew aircraft in a Vic formation, with the lead
plane using a mix of traditional fuel and bio-fuel, symbolising the government's determination to seek alternative
sources of fuel. The primary raw material for ethanol production in India is sugarcane and its by-products,
accounting for more than 90% of fuel ethanol supplies under the ethanol blended petrol (EBP) programme of the
ministry. This programme injects liquidity into the stressed sugar sector and provides an alternative revenue
stream to farmers. This programme also encourages diversion of sugarcane for ethanol production, which
results in a decrease in the sugar glut in the country.

Offers of about 3.5 billion litres from both sugar/molasses and grain-based distilleries are expected this year. In
addition to sugarcane, ethanol is also produced from damaged food grains, B-heavy molasses and sugarcane
juice. This translates into a sum of nearly Rs 35,000 crore in the past six years - money that has own back to
farmers through sugar mills and distilleries as oil marketing companies (OMCs) provide off-take guarantee at
xed prices. This arrangement also improves the health of the payment cycle to farmers since OMCs settle their
ethanol dues to distilleries in 21 days instead of the months that the farmers had to wait for their payment from
sugar mills.

The policy encourages setting up of supply chain mechanisms for biodiesel production from non-edible
oilseeds, used cooking oil and short-gestation crops. These crops can be easily cultivated in various states on
land that is barren or not t for edible crops, thus boosting farm incomes.

The Sustainable Alternative Towards Affordable Transportation (SATAT) scheme launched in October 2018
aims to establish an ecosystem for production of compressed biogas (CBG) from various waste biomass
sources in the country. This initiative offers a potential for investment of about Rs 1.75 lakh crore, generating
about 75,000 direct employment opportunities.

Many of the proposed plants will use crop residue such as paddy straw and biomass as feedstock for
production of CBG, especially in Haryana, Punjab and Uttar Pradesh. The SATAT scheme will not only stanch
greenhouse gas emissions, but will reduce burning of agricultural residue, which results in signi cant air
pollution in cities like Delhi, generate employment in rural and waste management sectors, and boost income
for farmers from their unutilised organic waste. One of the by-products of CBG plants is bio-manure, which can
be used in farming.

Q.17 [30465398]
Out of the following options, which one is an antonym of the word 'glut' as used in the passage?

https://www.aspiration.ai/LAW/sis/Solution.jsp?qsetId=JAmrn2p8H1A=&qsetName=LST Mock 27 2021 (CLAT) 33/223


1/17/2021 Mock Analysis

a Surfeit

b Excess

c Dearth

d Glob

Solution:
 Answer key/Solution
Correct Answer : c
Your Answer : c
'Glut' means 'surfeit' or excess of anything. Hence, 'dearth' is an antonym of 'glut.'
Options (a) and (b) are synonyms.

FeedBack Bookmark

Directions for questions 1 to 30: Each set of questions in this section is based on a single passage. Please
answer each question on the basis of what is stated or implied in the corresponding passage. In some
instances, more than one option may be the answer to the question; in such a case, please choose the option
that most accurately and comprehensively answers the question.

Passage – 4

India currently consumes only one-third of the global average consumption of energy. As we move towards
more robust energy consumption gures, the country faces the unique challenge of fusing together disparate
needs: A ballooning appetite while cutting dependence on imports; greening the grid while ensuring
affordability; and replacing old forms of energy production while boosting employment, and the human and
economic capital of the people. Then there's the fact that the energy sector is closely intertwined with issues
related to climate change.

In this context, bio-fuels have become a tool for achieving these delicate balances of outcomes. In the past few
years, progress made in the use of ethanol, compressed biogas and biodiesel - all different forms of bio-fuel -
will have a direct positive impact on both farm incomes (and the prosperity of agricultural communities), even
as it cuts down our import dependence for energy.

In 2019, in a rst for the Republic Day parade, the Indian Air Force ew aircraft in a Vic formation, with the lead
plane using a mix of traditional fuel and bio-fuel, symbolising the government's determination to seek alternative
sources of fuel. The primary raw material for ethanol production in India is sugarcane and its by-products,
accounting for more than 90% of fuel ethanol supplies under the ethanol blended petrol (EBP) programme of the
ministry. This programme injects liquidity into the stressed sugar sector and provides an alternative revenue
stream to farmers. This programme also encourages diversion of sugarcane for ethanol production, which
results in a decrease in the sugar glut in the country.

Offers of about 3.5 billion litres from both sugar/molasses and grain-based distilleries are expected this year. In

https://www.aspiration.ai/LAW/sis/Solution.jsp?qsetId=JAmrn2p8H1A=&qsetName=LST Mock 27 2021 (CLAT) 34/223


1/17/2021 Mock Analysis

addition to sugarcane, ethanol is also produced from damaged food grains, B-heavy molasses and sugarcane
juice. This translates into a sum of nearly Rs 35,000 crore in the past six years - money that has own back to
farmers through sugar mills and distilleries as oil marketing companies (OMCs) provide off-take guarantee at
xed prices. This arrangement also improves the health of the payment cycle to farmers since OMCs settle their
ethanol dues to distilleries in 21 days instead of the months that the farmers had to wait for their payment from
sugar mills.

The policy encourages setting up of supply chain mechanisms for biodiesel production from non-edible
oilseeds, used cooking oil and short-gestation crops. These crops can be easily cultivated in various states on
land that is barren or not t for edible crops, thus boosting farm incomes.

The Sustainable Alternative Towards Affordable Transportation (SATAT) scheme launched in October 2018
aims to establish an ecosystem for production of compressed biogas (CBG) from various waste biomass
sources in the country. This initiative offers a potential for investment of about Rs 1.75 lakh crore, generating
about 75,000 direct employment opportunities.

Many of the proposed plants will use crop residue such as paddy straw and biomass as feedstock for
production of CBG, especially in Haryana, Punjab and Uttar Pradesh. The SATAT scheme will not only stanch
greenhouse gas emissions, but will reduce burning of agricultural residue, which results in signi cant air
pollution in cities like Delhi, generate employment in rural and waste management sectors, and boost income
for farmers from their unutilised organic waste. One of the by-products of CBG plants is bio-manure, which can
be used in farming.

Q.18 [30465398]
The author of the passage is most likely to agree with which of the following statements:

a The government of India is keen to take foreign help for exploring greener sources of power.

b The government of India wants to sell bio-fuel to other countries and so, is eager to harness it.

c The government of India is reluctant to explore greener sources of energy.

d The government of India is keen to explore greener sources of fuel.

Solution:
 Answer key/Solution
Correct Answer : d
Your Answer : b
Refer to the rst sentence of the third paragraph for the answer. Option (d) can be
inferred from the Indian Air Force example cited by the author. Options (a) and (b) are out of scope. Option (c)
is the opposite of what the passage says.

FeedBack Bookmark

Directions for questions 1 to 30: Each set of questions in this section is based on a single passage. Please
answer each question on the basis of what is stated or implied in the corresponding passage. In some

https://www.aspiration.ai/LAW/sis/Solution.jsp?qsetId=JAmrn2p8H1A=&qsetName=LST Mock 27 2021 (CLAT) 35/223


1/17/2021 Mock Analysis

instances, more than one option may be the answer to the question; in such a case, please choose the option
that most accurately and comprehensively answers the question.

Passage – 4

India currently consumes only one-third of the global average consumption of energy. As we move towards
more robust energy consumption gures, the country faces the unique challenge of fusing together disparate
needs: A ballooning appetite while cutting dependence on imports; greening the grid while ensuring
affordability; and replacing old forms of energy production while boosting employment, and the human and
economic capital of the people. Then there's the fact that the energy sector is closely intertwined with issues
related to climate change.

In this context, bio-fuels have become a tool for achieving these delicate balances of outcomes. In the past few
years, progress made in the use of ethanol, compressed biogas and biodiesel - all different forms of bio-fuel -
will have a direct positive impact on both farm incomes (and the prosperity of agricultural communities), even
as it cuts down our import dependence for energy.

In 2019, in a rst for the Republic Day parade, the Indian Air Force ew aircraft in a Vic formation, with the lead
plane using a mix of traditional fuel and bio-fuel, symbolising the government's determination to seek alternative
sources of fuel. The primary raw material for ethanol production in India is sugarcane and its by-products,
accounting for more than 90% of fuel ethanol supplies under the ethanol blended petrol (EBP) programme of the
ministry. This programme injects liquidity into the stressed sugar sector and provides an alternative revenue
stream to farmers. This programme also encourages diversion of sugarcane for ethanol production, which
results in a decrease in the sugar glut in the country.

Offers of about 3.5 billion litres from both sugar/molasses and grain-based distilleries are expected this year. In
addition to sugarcane, ethanol is also produced from damaged food grains, B-heavy molasses and sugarcane
juice. This translates into a sum of nearly Rs 35,000 crore in the past six years - money that has own back to
farmers through sugar mills and distilleries as oil marketing companies (OMCs) provide off-take guarantee at
xed prices. This arrangement also improves the health of the payment cycle to farmers since OMCs settle their
ethanol dues to distilleries in 21 days instead of the months that the farmers had to wait for their payment from
sugar mills.

The policy encourages setting up of supply chain mechanisms for biodiesel production from non-edible
oilseeds, used cooking oil and short-gestation crops. These crops can be easily cultivated in various states on
land that is barren or not t for edible crops, thus boosting farm incomes.

The Sustainable Alternative Towards Affordable Transportation (SATAT) scheme launched in October 2018
aims to establish an ecosystem for production of compressed biogas (CBG) from various waste biomass
sources in the country. This initiative offers a potential for investment of about Rs 1.75 lakh crore, generating
about 75,000 direct employment opportunities.

Many of the proposed plants will use crop residue such as paddy straw and biomass as feedstock for
production of CBG, especially in Haryana, Punjab and Uttar Pradesh. The SATAT scheme will not only stanch
greenhouse gas emissions, but will reduce burning of agricultural residue, which results in signi cant air
pollution in cities like Delhi, generate employment in rural and waste management sectors, and boost income
for farmers from their unutilised organic waste. One of the by-products of CBG plants is bio-manure, which can
be used in farming.

https://www.aspiration.ai/LAW/sis/Solution.jsp?qsetId=JAmrn2p8H1A=&qsetName=LST Mock 27 2021 (CLAT) 36/223


1/17/2021 Mock Analysis

Q.19 [30465398]
Out of the following options, which one is true in the light of the passage?

a Bio-fuel can be produced from fallow lands.

b Bio-fuel should only be produced from fallow land.

c Bio-fuel can be a health hazard if not cultivated properly.

d Bio-fuel can be produced in hilly regions because of the type of the soil available in the region.

Solution:
 Answer key/Solution
Correct Answer : a
Your Answer : a
Refer to the paragraph beginning with, "The policy encourages setting up of
supply chain mechanisms for biodiesel production from ...." 'Fallow land' refers to lands that cannot be
cultivated.

FeedBack Bookmark

Directions for questions 1 to 30: Each set of questions in this section is based on a single passage. Please
answer each question on the basis of what is stated or implied in the corresponding passage. In some
instances, more than one option may be the answer to the question; in such a case, please choose the option
that most accurately and comprehensively answers the question.

Passage – 4

India currently consumes only one-third of the global average consumption of energy. As we move towards
more robust energy consumption gures, the country faces the unique challenge of fusing together disparate
needs: A ballooning appetite while cutting dependence on imports; greening the grid while ensuring
affordability; and replacing old forms of energy production while boosting employment, and the human and
economic capital of the people. Then there's the fact that the energy sector is closely intertwined with issues
related to climate change.

In this context, bio-fuels have become a tool for achieving these delicate balances of outcomes. In the past few
years, progress made in the use of ethanol, compressed biogas and biodiesel - all different forms of bio-fuel -
will have a direct positive impact on both farm incomes (and the prosperity of agricultural communities), even
as it cuts down our import dependence for energy.

In 2019, in a rst for the Republic Day parade, the Indian Air Force ew aircraft in a Vic formation, with the lead
plane using a mix of traditional fuel and bio-fuel, symbolising the government's determination to seek alternative
sources of fuel. The primary raw material for ethanol production in India is sugarcane and its by-products,
accounting for more than 90% of fuel ethanol supplies under the ethanol blended petrol (EBP) programme of the
ministry. This programme injects liquidity into the stressed sugar sector and provides an alternative revenue

https://www.aspiration.ai/LAW/sis/Solution.jsp?qsetId=JAmrn2p8H1A=&qsetName=LST Mock 27 2021 (CLAT) 37/223


1/17/2021 Mock Analysis

stream to farmers. This programme also encourages diversion of sugarcane for ethanol production, which
results in a decrease in the sugar glut in the country.

Offers of about 3.5 billion litres from both sugar/molasses and grain-based distilleries are expected this year. In
addition to sugarcane, ethanol is also produced from damaged food grains, B-heavy molasses and sugarcane
juice. This translates into a sum of nearly Rs 35,000 crore in the past six years - money that has own back to
farmers through sugar mills and distilleries as oil marketing companies (OMCs) provide off-take guarantee at
xed prices. This arrangement also improves the health of the payment cycle to farmers since OMCs settle their
ethanol dues to distilleries in 21 days instead of the months that the farmers had to wait for their payment from
sugar mills.

The policy encourages setting up of supply chain mechanisms for biodiesel production from non-edible
oilseeds, used cooking oil and short-gestation crops. These crops can be easily cultivated in various states on
land that is barren or not t for edible crops, thus boosting farm incomes.

The Sustainable Alternative Towards Affordable Transportation (SATAT) scheme launched in October 2018
aims to establish an ecosystem for production of compressed biogas (CBG) from various waste biomass
sources in the country. This initiative offers a potential for investment of about Rs 1.75 lakh crore, generating
about 75,000 direct employment opportunities.

Many of the proposed plants will use crop residue such as paddy straw and biomass as feedstock for
production of CBG, especially in Haryana, Punjab and Uttar Pradesh. The SATAT scheme will not only stanch
greenhouse gas emissions, but will reduce burning of agricultural residue, which results in signi cant air
pollution in cities like Delhi, generate employment in rural and waste management sectors, and boost income
for farmers from their unutilised organic waste. One of the by-products of CBG plants is bio-manure, which can
be used in farming.

Q.20 [30465398]
In the light of the passage, it can be understood that:

a Climate change is a pertinent topic in the domain of energy needs of South Asian countries.

b Climate change is a pertinent topic in the domain of energy needs of very few countries.

c Climate change is a pertinent topic in the domain of energy needs.

d The maximum degree of climate change may be witnessed in the next 2 decades.

Solution:
 Answer key/Solution
Correct Answer : c
Your Answer : c
Refer to the last sentence of the rst paragraph for the answer. The remaining
options are not stated, nor can they be inferred from the passage.

FeedBack Bookmark

https://www.aspiration.ai/LAW/sis/Solution.jsp?qsetId=JAmrn2p8H1A=&qsetName=LST Mock 27 2021 (CLAT) 38/223


1/17/2021 Mock Analysis

Directions for questions 1 to 30: Each set of questions in this section is based on a single passage. Please
answer each question on the basis of what is stated or implied in the corresponding passage. In some
instances, more than one option may be the answer to the question; in such a case, please choose the option
that most accurately and comprehensively answers the question.

Passage – 5

Climate disasters generally have a disproportionate impact on the lives of disadvantaged people who are least
responsible for contributing to climate change but are most vulnerable to have their lives, homes, families and
livelihoods destroyed by climate disasters.

Gender inequalities make things worse. Women and children are facing the brunt of climate-induced migration.
United Nations gures suggest that about 80 per cent of people displaced by climate change are women.

In India, when men are forced to abandon their farms and migrate in search of work, women are left behind to
manage the household responsibilities and care for their families.Women also make up for a signi cant share in
the workforce of different sectors, like agriculture, that are most vulnerable to the impact of climate change.

Migrants are often considered a hot topic in Indian politics. Climate-induced migration and displacement,
however, is yet to nd the attention of political discourse and public policy. A good starting point to create
effective policies is researching the reasons for climate migration and coping mechanisms adopted by
migrants.

Political actors in India should divorce climate migrants from undocumented immigrants. An appropriate
humanitarian response would be to recognise and de ne climate migrants as well as create legal frameworks to
grant them their rights and entitlements to their recovery from climate disasters.

Public policy actors cannot remain gender-neutral particularly when climate change in icts a disproportionate
burden on women. A gender perspective is important to better understand the linkage between climate
disasters and climate migrants.

Gender-based violence needs to be acknowledged to reimagine climate governance and to adopt gender-
sensitive responses to climate-induced migration. Moreover, disaggregated migration data by gender will help in
better understanding the inequalities and drafting suitable policies to support women and strengthen their
resilience.

India's ambitious targets to cut carbon emissions require strong political will, meaningful engagements and
sustainable plans. Large-scale construction projects should be assessed for greenhouse gas emissions,
environmental degradation and potential impact on the lives of local people and resources.

India should increase its spending on research and innovation to make housing more climate-resilient. It also
needs long-term plans to construct rehabilitation centers for climate migrants, especially for 250 million
vulnerable people residing along the 7,500 km long coastline of India.

Climate nance can prove to be a compelling nancial tool to align India's growth with various climate change
measures. India's Intended Nationally Determined Contribution (INDC) estimated that $2.5 trillion is needed
from 2015 to 2030 to reduce India's carbon intensity by 33-35 per cent by 2030 from its 2005 levels.

https://www.aspiration.ai/LAW/sis/Solution.jsp?qsetId=JAmrn2p8H1A=&qsetName=LST Mock 27 2021 (CLAT) 39/223


1/17/2021 Mock Analysis

The 2015 Economic Survey of India argued that international climate nance is necessary to meet the
difference over what can be made available from domestic sources. Therefore, the government of India needs
to recognise that there is an urgent need to invest in climate action.

Q.21 [30465398]
Which of the following would be the most suitable title to the given passage?

a Climate change and climate-induced migrants

b Migrants and climate-induced migrants

c The need for India to invest urgently in climate action

d Climate change and the gender issues

Solution:
 Answer key/Solution
Correct Answer : c
Your Answer : c
The passage is essentially about the need for India to invest urgently in climate
action. Refer to the last three paragraphs of the given passage. Hence, option (c) is the answer. None of the
other options can be the title of the given passage.

FeedBack Bookmark

Directions for questions 1 to 30: Each set of questions in this section is based on a single passage. Please
answer each question on the basis of what is stated or implied in the corresponding passage. In some
instances, more than one option may be the answer to the question; in such a case, please choose the option
that most accurately and comprehensively answers the question.

Passage – 5

Climate disasters generally have a disproportionate impact on the lives of disadvantaged people who are least
responsible for contributing to climate change but are most vulnerable to have their lives, homes, families and
livelihoods destroyed by climate disasters.

Gender inequalities make things worse. Women and children are facing the brunt of climate-induced migration.
United Nations gures suggest that about 80 per cent of people displaced by climate change are women.

In India, when men are forced to abandon their farms and migrate in search of work, women are left behind to
manage the household responsibilities and care for their families.Women also make up for a signi cant share in
the workforce of different sectors, like agriculture, that are most vulnerable to the impact of climate change.

Migrants are often considered a hot topic in Indian politics. Climate-induced migration and displacement,
however, is yet to nd the attention of political discourse and public policy. A good starting point to create

https://www.aspiration.ai/LAW/sis/Solution.jsp?qsetId=JAmrn2p8H1A=&qsetName=LST Mock 27 2021 (CLAT) 40/223


1/17/2021 Mock Analysis

effective policies is researching the reasons for climate migration and coping mechanisms adopted by
migrants.

Political actors in India should divorce climate migrants from undocumented immigrants. An appropriate
humanitarian response would be to recognise and de ne climate migrants as well as create legal frameworks to
grant them their rights and entitlements to their recovery from climate disasters.

Public policy actors cannot remain gender-neutral particularly when climate change in icts a disproportionate
burden on women. A gender perspective is important to better understand the linkage between climate
disasters and climate migrants.

Gender-based violence needs to be acknowledged to reimagine climate governance and to adopt gender-
sensitive responses to climate-induced migration. Moreover, disaggregated migration data by gender will help in
better understanding the inequalities and drafting suitable policies to support women and strengthen their
resilience.

India's ambitious targets to cut carbon emissions require strong political will, meaningful engagements and
sustainable plans. Large-scale construction projects should be assessed for greenhouse gas emissions,
environmental degradation and potential impact on the lives of local people and resources.

India should increase its spending on research and innovation to make housing more climate-resilient. It also
needs long-term plans to construct rehabilitation centers for climate migrants, especially for 250 million
vulnerable people residing along the 7,500 km long coastline of India.

Climate nance can prove to be a compelling nancial tool to align India's growth with various climate change
measures. India's Intended Nationally Determined Contribution (INDC) estimated that $2.5 trillion is needed
from 2015 to 2030 to reduce India's carbon intensity by 33-35 per cent by 2030 from its 2005 levels.

The 2015 Economic Survey of India argued that international climate nance is necessary to meet the
difference over what can be made available from domestic sources. Therefore, the government of India needs
to recognise that there is an urgent need to invest in climate action.

Q.22 [30465398]
Which of the following cannot be inferred from the passage?

a In India migration is necessitated by climate change.

b At the moment, there is absolutely no assistance available for the rehabilitation of climate-induced
migrants in India.

c Women are affected to a great degree by climate change.

d India needs a strong political will to cut its carbon emissions.

https://www.aspiration.ai/LAW/sis/Solution.jsp?qsetId=JAmrn2p8H1A=&qsetName=LST Mock 27 2021 (CLAT) 41/223


1/17/2021 Mock Analysis

Solution:
 Answer key/Solution
Correct Answer : b
Your Answer : b
Option (a) is true to a certain extent and this is mentioned in the given passage.
Option (c) is also mentioned in the given passage. Refer to "… about 80 per cent of people displaced by
climate change are women." Option (d) is mentioned in the given passage. Refer to "India's ambitious targets
to cut carbon emissions require strong political will…" Option (b) is a bit extreme in its claim.
FeedBack Bookmark

Directions for questions 1 to 30: Each set of questions in this section is based on a single passage. Please
answer each question on the basis of what is stated or implied in the corresponding passage. In some
instances, more than one option may be the answer to the question; in such a case, please choose the option
that most accurately and comprehensively answers the question.

Passage – 5

Climate disasters generally have a disproportionate impact on the lives of disadvantaged people who are least
responsible for contributing to climate change but are most vulnerable to have their lives, homes, families and
livelihoods destroyed by climate disasters.

Gender inequalities make things worse. Women and children are facing the brunt of climate-induced migration.
United Nations gures suggest that about 80 per cent of people displaced by climate change are women.

In India, when men are forced to abandon their farms and migrate in search of work, women are left behind to
manage the household responsibilities and care for their families.Women also make up for a signi cant share in
the workforce of different sectors, like agriculture, that are most vulnerable to the impact of climate change.

Migrants are often considered a hot topic in Indian politics. Climate-induced migration and displacement,
however, is yet to nd the attention of political discourse and public policy. A good starting point to create
effective policies is researching the reasons for climate migration and coping mechanisms adopted by
migrants.

Political actors in India should divorce climate migrants from undocumented immigrants. An appropriate
humanitarian response would be to recognise and de ne climate migrants as well as create legal frameworks to
grant them their rights and entitlements to their recovery from climate disasters.

Public policy actors cannot remain gender-neutral particularly when climate change in icts a disproportionate
burden on women. A gender perspective is important to better understand the linkage between climate
disasters and climate migrants.

Gender-based violence needs to be acknowledged to reimagine climate governance and to adopt gender-
sensitive responses to climate-induced migration. Moreover, disaggregated migration data by gender will help in
better understanding the inequalities and drafting suitable policies to support women and strengthen their
resilience.

India's ambitious targets to cut carbon emissions require strong political will, meaningful engagements and
https://www.aspiration.ai/LAW/sis/Solution.jsp?qsetId=JAmrn2p8H1A=&qsetName=LST Mock 27 2021 (CLAT) 42/223
1/17/2021 Mock Analysis

sustainable plans. Large-scale construction projects should be assessed for greenhouse gas emissions,
environmental degradation and potential impact on the lives of local people and resources.

India should increase its spending on research and innovation to make housing more climate-resilient. It also
needs long-term plans to construct rehabilitation centers for climate migrants, especially for 250 million
vulnerable people residing along the 7,500 km long coastline of India.

Climate nance can prove to be a compelling nancial tool to align India's growth with various climate change
measures. India's Intended Nationally Determined Contribution (INDC) estimated that $2.5 trillion is needed
from 2015 to 2030 to reduce India's carbon intensity by 33-35 per cent by 2030 from its 2005 levels.

The 2015 Economic Survey of India argued that international climate nance is necessary to meet the
difference over what can be made available from domestic sources. Therefore, the government of India needs
to recognise that there is an urgent need to invest in climate action.

Q.23 [30465398]
Which of the following is NOT true according to the given passage?

a According to the UN, more than 50 percent of people displaced by climate change are women.

b As far as the climate related issues are concerned, gender-based policies are securely in place.

c In India, women contribute signi cantly to the agricultural sector.

d Agriculture is susceptible to the impact of climate change.

Solution:
 Answer key/Solution
Correct Answer : b
Your Answer : b
Refer to "Public policy actors cannot remain gender-neutral particularly when
climate change in icts a disproportionate burden on women. A gender perspective is important to better
understand the linkage between climate disasters and climate migrants. The quoted sentences clearly
indicate that gender-based public policy cannot remain gender-neutral as women are disproportionately
affected by climate change. Hence, option (b) is the answer. Options (a), (c) and (d) are true according to the
given passage, hence, they are incorrect.

FeedBack Bookmark

Directions for questions 1 to 30: Each set of questions in this section is based on a single passage. Please
answer each question on the basis of what is stated or implied in the corresponding passage. In some
instances, more than one option may be the answer to the question; in such a case, please choose the option
that most accurately and comprehensively answers the question.

Passage – 5

https://www.aspiration.ai/LAW/sis/Solution.jsp?qsetId=JAmrn2p8H1A=&qsetName=LST Mock 27 2021 (CLAT) 43/223


1/17/2021 Mock Analysis

Climate disasters generally have a disproportionate impact on the lives of disadvantaged people who are least
responsible for contributing to climate change but are most vulnerable to have their lives, homes, families and
livelihoods destroyed by climate disasters.

Gender inequalities make things worse. Women and children are facing the brunt of climate-induced migration.
United Nations gures suggest that about 80 per cent of people displaced by climate change are women.

In India, when men are forced to abandon their farms and migrate in search of work, women are left behind to
manage the household responsibilities and care for their families.Women also make up for a signi cant share in
the workforce of different sectors, like agriculture, that are most vulnerable to the impact of climate change.

Migrants are often considered a hot topic in Indian politics. Climate-induced migration and displacement,
however, is yet to nd the attention of political discourse and public policy. A good starting point to create
effective policies is researching the reasons for climate migration and coping mechanisms adopted by
migrants.

Political actors in India should divorce climate migrants from undocumented immigrants. An appropriate
humanitarian response would be to recognise and de ne climate migrants as well as create legal frameworks to
grant them their rights and entitlements to their recovery from climate disasters.

Public policy actors cannot remain gender-neutral particularly when climate change in icts a disproportionate
burden on women. A gender perspective is important to better understand the linkage between climate
disasters and climate migrants.

Gender-based violence needs to be acknowledged to reimagine climate governance and to adopt gender-
sensitive responses to climate-induced migration. Moreover, disaggregated migration data by gender will help in
better understanding the inequalities and drafting suitable policies to support women and strengthen their
resilience.

India's ambitious targets to cut carbon emissions require strong political will, meaningful engagements and
sustainable plans. Large-scale construction projects should be assessed for greenhouse gas emissions,
environmental degradation and potential impact on the lives of local people and resources.

India should increase its spending on research and innovation to make housing more climate-resilient. It also
needs long-term plans to construct rehabilitation centers for climate migrants, especially for 250 million
vulnerable people residing along the 7,500 km long coastline of India.

Climate nance can prove to be a compelling nancial tool to align India's growth with various climate change
measures. India's Intended Nationally Determined Contribution (INDC) estimated that $2.5 trillion is needed
from 2015 to 2030 to reduce India's carbon intensity by 33-35 per cent by 2030 from its 2005 levels.

The 2015 Economic Survey of India argued that international climate nance is necessary to meet the
difference over what can be made available from domestic sources. Therefore, the government of India needs
to recognise that there is an urgent need to invest in climate action.

Q.24 [30465398]
It can be inferred from the given passage that:

https://www.aspiration.ai/LAW/sis/Solution.jsp?qsetId=JAmrn2p8H1A=&qsetName=LST Mock 27 2021 (CLAT) 44/223


1/17/2021 Mock Analysis

a at the moment, large-scale construction projects in India are solely responsible for greenhouse gas
emissions.

b at the moment, large-scale construction projects in India are properly assessed for greenhouse gas
emissions.

c large-scale construction projects in India strike a perfect balance between policy and practicality.

d at the moment, large-scale construction projects in India are not properly assessed for greenhouse gas
emissions.

Solution:
 Answer key/Solution
Correct Answer : d
Your Answer : d
Option (a) is extreme and cannot be inferred. Option (b) is not true and cannot be
inferred either. Option (c) is incorrect as well. Option (d) can be inferred from "Large-scale construction
projects should be assessed for greenhouse gas emissions…" The quoted sentence suggests that at the
moment, large-scale construction projects in India are not properly assessed for greenhouse gas emissions.
Hence, option (d) is the answer.

FeedBack Bookmark

Directions for questions 1 to 30: Each set of questions in this section is based on a single passage. Please
answer each question on the basis of what is stated or implied in the corresponding passage. In some
instances, more than one option may be the answer to the question; in such a case, please choose the option
that most accurately and comprehensively answers the question.

Passage – 5

Climate disasters generally have a disproportionate impact on the lives of disadvantaged people who are least
responsible for contributing to climate change but are most vulnerable to have their lives, homes, families and
livelihoods destroyed by climate disasters.

Gender inequalities make things worse. Women and children are facing the brunt of climate-induced migration.
United Nations gures suggest that about 80 per cent of people displaced by climate change are women.

In India, when men are forced to abandon their farms and migrate in search of work, women are left behind to
manage the household responsibilities and care for their families.Women also make up for a signi cant share in
the workforce of different sectors, like agriculture, that are most vulnerable to the impact of climate change.

Migrants are often considered a hot topic in Indian politics. Climate-induced migration and displacement,
however, is yet to nd the attention of political discourse and public policy. A good starting point to create
effective policies is researching the reasons for climate migration and coping mechanisms adopted by
migrants.

https://www.aspiration.ai/LAW/sis/Solution.jsp?qsetId=JAmrn2p8H1A=&qsetName=LST Mock 27 2021 (CLAT) 45/223


1/17/2021 Mock Analysis

Political actors in India should divorce climate migrants from undocumented immigrants. An appropriate
humanitarian response would be to recognise and de ne climate migrants as well as create legal frameworks to
grant them their rights and entitlements to their recovery from climate disasters.

Public policy actors cannot remain gender-neutral particularly when climate change in icts a disproportionate
burden on women. A gender perspective is important to better understand the linkage between climate
disasters and climate migrants.

Gender-based violence needs to be acknowledged to reimagine climate governance and to adopt gender-
sensitive responses to climate-induced migration. Moreover, disaggregated migration data by gender will help in
better understanding the inequalities and drafting suitable policies to support women and strengthen their
resilience.

India's ambitious targets to cut carbon emissions require strong political will, meaningful engagements and
sustainable plans. Large-scale construction projects should be assessed for greenhouse gas emissions,
environmental degradation and potential impact on the lives of local people and resources.

India should increase its spending on research and innovation to make housing more climate-resilient. It also
needs long-term plans to construct rehabilitation centers for climate migrants, especially for 250 million
vulnerable people residing along the 7,500 km long coastline of India.

Climate nance can prove to be a compelling nancial tool to align India's growth with various climate change
measures. India's Intended Nationally Determined Contribution (INDC) estimated that $2.5 trillion is needed
from 2015 to 2030 to reduce India's carbon intensity by 33-35 per cent by 2030 from its 2005 levels.

The 2015 Economic Survey of India argued that international climate nance is necessary to meet the
difference over what can be made available from domestic sources. Therefore, the government of India needs
to recognise that there is an urgent need to invest in climate action.

Q.25 [30465398]
Which of the following words would be the synonym of 'vulnerable' as given in the passage?

a Overwhelming

b Buoyant

c Prone

d Resilient

https://www.aspiration.ai/LAW/sis/Solution.jsp?qsetId=JAmrn2p8H1A=&qsetName=LST Mock 27 2021 (CLAT) 46/223


1/17/2021 Mock Analysis

Solution:
 Answer key/Solution
Correct Answer : c
Your Answer : c
Vulnerable means susceptible or prone. So, option (c) is the answer.
Overwhelming means irresistible or awe-inspiring. Buoyant means oating. Resilient means robust or supple.

FeedBack Bookmark

https://www.aspiration.ai/LAW/sis/Solution.jsp?qsetId=JAmrn2p8H1A=&qsetName=LST Mock 27 2021 (CLAT) 47/223


1/17/2021 Mock Analysis

Directions for questions 1 to 30: Each set of questions in this section is based on a single passage. Please
answer each question on the basis of what is stated or implied in the corresponding passage. In some
instances, more than one option may be the answer to the question; in such a case, please choose the option
that most accurately and comprehensively answers the question.

Passage – 6

Coronavirus and political anxiety have accelerated this shift toward comfort TV. Who wants to be frustrated or
made sadder by their entertainment when the outside world itself feels like a su ciently brutal place?

On the other side stands Prestige TV -- shows watched for the "prestige" they carry -- that has always been
di cult to categorize, in part because it is a mirage. Its gradual dissolution over the past several years is equally
tricky to nail to any speci c benchmark. But since the fever pitch of critical excitement over shows like Mad
Men, The Wire, and the early days of Game of Thrones, prestige TV as an aesthetic has persisted, even while
prestige as a reliable indicator of exciting TV you must talk about has undeniably waned.

In 2013, Vulture published "The 13 Rules for Creating a Prestige TV Drama," a full-feature list that began with
installing a middle-aged anti-hero at the end of some major cultural era, making him great at his job and bad at
his family, providing a healthy dose of both sex and violence, and sprinkling in a smattering of highfalutin
cultural references to make sure the audience could recognize the show's pedigree and intelligence. I wrote an
updated version of that list in 2017 re ecting the way prestige has become as much about how we talk about
the genre as the genre itself. In 2015, as a bouncing-off point to de ne a genre he called "mid-reputable TV," the
critic Noel Murray laid out some "hallmarks of a prestige drama - heavy themes, high production values,
accomplished actors," before conceding that "prestige is a state of mind." Much of its de nition comes from a
show's reception, Murray concluded. "Prestige television is often subject to intense scrutiny, with fans and
critics evaluating every plot twist, stylistic choice, and coded method (in terms of both literary symbolism and
the show's attitudes about gender, race, and politics)." In an essay titled "How American TV Became
Respectable Without Getting Better," Matthew Christman offers a compelling de nition: "Mostly crime. Mostly
male. Mostly extravagantly unlikable antiheroes whose sheer awfulness makes us feel better about our own,
more mundane foibles."

I suspect some of the reason for prestige TV's simultaneous rise is that prestige, as a term, neatly dodges the
best-versus-favorite (quality vs enjoyment) problem. Rather than de ne television by the pretense of objective
greatness, the word prestige, with its snobbish implications, was indicative of a perhaps unintentional truth
about these shows. It is a classist assessment, one that implies inaccessibility, scarcity, but also widespread
in uence and importance. It's not easy or uffy or fun because, somewhere deep in the American psyche,
there's still a puritanical belief that fun things cannot also be serious. Prestige TV is demanding, something that
creates a further classist implication about its audience. It imagines itself being viewed by educated, wealthy
people with enough leisure hours to devote to epic run times and slow-burning, byzantine plots (it imagines this
regardless of who may actually be watching). It's not a coincidence that the outlets that de ned prestige TV are
also the ones, like HBO, that historically required an expensive monthly premium-cable package

Q.26 [30465398]
All of the following are present in the 2013 full-feature list of "13 Rules for Creating a prestige TV Drama",
published by Vulture, EXCEPT:

a Grandiose cultural references

https://www.aspiration.ai/LAW/sis/Solution.jsp?qsetId=JAmrn2p8H1A=&qsetName=LST Mock 27 2021 (CLAT) 48/223


1/17/2021 Mock Analysis

b A healthy dose of violence

c A white, middle-aged anti-hero

d Protagonist portrayed as good at his job but bad at his family

Solution:
 Answer key/Solution
Correct Answer : c
Refer the rst few sentences of Para 3. It is not mentioned that the anti-hero is
white. Hence (c). The other features are mentioned.

FeedBack Bookmark

https://www.aspiration.ai/LAW/sis/Solution.jsp?qsetId=JAmrn2p8H1A=&qsetName=LST Mock 27 2021 (CLAT) 49/223


1/17/2021 Mock Analysis

Directions for questions 1 to 30: Each set of questions in this section is based on a single passage. Please
answer each question on the basis of what is stated or implied in the corresponding passage. In some
instances, more than one option may be the answer to the question; in such a case, please choose the option
that most accurately and comprehensively answers the question.

Passage – 6

Coronavirus and political anxiety have accelerated this shift toward comfort TV. Who wants to be frustrated or
made sadder by their entertainment when the outside world itself feels like a su ciently brutal place?

On the other side stands Prestige TV -- shows watched for the "prestige" they carry -- that has always been
di cult to categorize, in part because it is a mirage. Its gradual dissolution over the past several years is equally
tricky to nail to any speci c benchmark. But since the fever pitch of critical excitement over shows like Mad
Men, The Wire, and the early days of Game of Thrones, prestige TV as an aesthetic has persisted, even while
prestige as a reliable indicator of exciting TV you must talk about has undeniably waned.

In 2013, Vulture published "The 13 Rules for Creating a Prestige TV Drama," a full-feature list that began with
installing a middle-aged anti-hero at the end of some major cultural era, making him great at his job and bad at
his family, providing a healthy dose of both sex and violence, and sprinkling in a smattering of highfalutin
cultural references to make sure the audience could recognize the show's pedigree and intelligence. I wrote an
updated version of that list in 2017 re ecting the way prestige has become as much about how we talk about
the genre as the genre itself. In 2015, as a bouncing-off point to de ne a genre he called "mid-reputable TV," the
critic Noel Murray laid out some "hallmarks of a prestige drama - heavy themes, high production values,
accomplished actors," before conceding that "prestige is a state of mind." Much of its de nition comes from a
show's reception, Murray concluded. "Prestige television is often subject to intense scrutiny, with fans and
critics evaluating every plot twist, stylistic choice, and coded method (in terms of both literary symbolism and
the show's attitudes about gender, race, and politics)." In an essay titled "How American TV Became
Respectable Without Getting Better," Matthew Christman offers a compelling de nition: "Mostly crime. Mostly
male. Mostly extravagantly unlikable antiheroes whose sheer awfulness makes us feel better about our own,
more mundane foibles."

I suspect some of the reason for prestige TV's simultaneous rise is that prestige, as a term, neatly dodges the
best-versus-favorite (quality vs enjoyment) problem. Rather than de ne television by the pretense of objective
greatness, the word prestige, with its snobbish implications, was indicative of a perhaps unintentional truth
about these shows. It is a classist assessment, one that implies inaccessibility, scarcity, but also widespread
in uence and importance. It's not easy or uffy or fun because, somewhere deep in the American psyche,
there's still a puritanical belief that fun things cannot also be serious. Prestige TV is demanding, something that
creates a further classist implication about its audience. It imagines itself being viewed by educated, wealthy
people with enough leisure hours to devote to epic run times and slow-burning, byzantine plots (it imagines this
regardless of who may actually be watching). It's not a coincidence that the outlets that de ned prestige TV are
also the ones, like HBO, that historically required an expensive monthly premium-cable package

Q.27 [30465398]
In Para 3, what does the author imply by the statement, "…prestige has become as much about how we talk
about a genre as the genre itself."?

a The audience considers prestige TV to be the TV that deals with heavy themes.

https://www.aspiration.ai/LAW/sis/Solution.jsp?qsetId=JAmrn2p8H1A=&qsetName=LST Mock 27 2021 (CLAT) 50/223


1/17/2021 Mock Analysis

b The categorisation of a show as prestige TV depends on the perception that a genre carries among the
viewers.

c Prestige TV is di cult to categorise because it does not just depend on the genre to which it belongs, but
also on the viewers' state of mind.

d The categorisation of a show as prestige TV is determined by the discussion that it generates on the genre
to which it belongs.

Solution:
 Answer key/Solution
Correct Answer : b
Your Answer : d
'How we talk about a genre' here means how we perceive a genre. Hence (b). (a) is
a fact stated by Murray, not implied by the author. (c) is incomplete, distorted and does not give the meaning
of the phrase asked in the question. The last phrase should have been 'the viewers' perception of a genre'. (d)
is eliminated because the sentence does not talk about the discussion "generated by the show" about the
genre.
FeedBack Bookmark

https://www.aspiration.ai/LAW/sis/Solution.jsp?qsetId=JAmrn2p8H1A=&qsetName=LST Mock 27 2021 (CLAT) 51/223


1/17/2021 Mock Analysis

Directions for questions 1 to 30: Each set of questions in this section is based on a single passage. Please
answer each question on the basis of what is stated or implied in the corresponding passage. In some
instances, more than one option may be the answer to the question; in such a case, please choose the option
that most accurately and comprehensively answers the question.

Passage – 6

Coronavirus and political anxiety have accelerated this shift toward comfort TV. Who wants to be frustrated or
made sadder by their entertainment when the outside world itself feels like a su ciently brutal place?

On the other side stands Prestige TV -- shows watched for the "prestige" they carry -- that has always been
di cult to categorize, in part because it is a mirage. Its gradual dissolution over the past several years is equally
tricky to nail to any speci c benchmark. But since the fever pitch of critical excitement over shows like Mad
Men, The Wire, and the early days of Game of Thrones, prestige TV as an aesthetic has persisted, even while
prestige as a reliable indicator of exciting TV you must talk about has undeniably waned.

In 2013, Vulture published "The 13 Rules for Creating a Prestige TV Drama," a full-feature list that began with
installing a middle-aged anti-hero at the end of some major cultural era, making him great at his job and bad at
his family, providing a healthy dose of both sex and violence, and sprinkling in a smattering of highfalutin
cultural references to make sure the audience could recognize the show's pedigree and intelligence. I wrote an
updated version of that list in 2017 re ecting the way prestige has become as much about how we talk about
the genre as the genre itself. In 2015, as a bouncing-off point to de ne a genre he called "mid-reputable TV," the
critic Noel Murray laid out some "hallmarks of a prestige drama - heavy themes, high production values,
accomplished actors," before conceding that "prestige is a state of mind." Much of its de nition comes from a
show's reception, Murray concluded. "Prestige television is often subject to intense scrutiny, with fans and
critics evaluating every plot twist, stylistic choice, and coded method (in terms of both literary symbolism and
the show's attitudes about gender, race, and politics)." In an essay titled "How American TV Became
Respectable Without Getting Better," Matthew Christman offers a compelling de nition: "Mostly crime. Mostly
male. Mostly extravagantly unlikable antiheroes whose sheer awfulness makes us feel better about our own,
more mundane foibles."

I suspect some of the reason for prestige TV's simultaneous rise is that prestige, as a term, neatly dodges the
best-versus-favorite (quality vs enjoyment) problem. Rather than de ne television by the pretense of objective
greatness, the word prestige, with its snobbish implications, was indicative of a perhaps unintentional truth
about these shows. It is a classist assessment, one that implies inaccessibility, scarcity, but also widespread
in uence and importance. It's not easy or uffy or fun because, somewhere deep in the American psyche,
there's still a puritanical belief that fun things cannot also be serious. Prestige TV is demanding, something that
creates a further classist implication about its audience. It imagines itself being viewed by educated, wealthy
people with enough leisure hours to devote to epic run times and slow-burning, byzantine plots (it imagines this
regardless of who may actually be watching). It's not a coincidence that the outlets that de ned prestige TV are
also the ones, like HBO, that historically required an expensive monthly premium-cable package

Q.28 [30465398]
Which of the following, if true, would contribute the most to weakening Matthew Christman's views as
mentioned in the passage?

a American TV has given us some highly admired, principled protagonists or heroes over the years.

https://www.aspiration.ai/LAW/sis/Solution.jsp?qsetId=JAmrn2p8H1A=&qsetName=LST Mock 27 2021 (CLAT) 52/223


1/17/2021 Mock Analysis

b American TV has been very adaptable to responding to the needs and tastes of its viewers since its
inception.

c American TV, when analysed, revealed a cast that could be complimented for its equal sex ratio, with
women playing roles that echoed the times.

d American TV is popular for the racial diversity that its shows have portrayed since its early days, with its
eclectic mix going beyond African Americans.

Solution:
 Answer key/Solution
Correct Answer : c
Mathew Christman's essay calls American TV mostly crime, mostly male, mostly
anti-hero. (a) is eliminated as Christman does not call the shows 'entirely' anti-
hero, but 'mostly' anti-hero. (b) does not weaken the argument because Matthew indirectly acknowledges that
American TV was 'respectable' and 'made people feel better about their own foibles'. It does not attack his
main argument. (d) is eliminated because race has not been discussed by Matthew. (c) attacks his "mostly
male" argument.

FeedBack Bookmark

https://www.aspiration.ai/LAW/sis/Solution.jsp?qsetId=JAmrn2p8H1A=&qsetName=LST Mock 27 2021 (CLAT) 53/223


1/17/2021 Mock Analysis

Directions for questions 1 to 30: Each set of questions in this section is based on a single passage. Please
answer each question on the basis of what is stated or implied in the corresponding passage. In some
instances, more than one option may be the answer to the question; in such a case, please choose the option
that most accurately and comprehensively answers the question.

Passage – 6

Coronavirus and political anxiety have accelerated this shift toward comfort TV. Who wants to be frustrated or
made sadder by their entertainment when the outside world itself feels like a su ciently brutal place?

On the other side stands Prestige TV -- shows watched for the "prestige" they carry -- that has always been
di cult to categorize, in part because it is a mirage. Its gradual dissolution over the past several years is equally
tricky to nail to any speci c benchmark. But since the fever pitch of critical excitement over shows like Mad
Men, The Wire, and the early days of Game of Thrones, prestige TV as an aesthetic has persisted, even while
prestige as a reliable indicator of exciting TV you must talk about has undeniably waned.

In 2013, Vulture published "The 13 Rules for Creating a Prestige TV Drama," a full-feature list that began with
installing a middle-aged anti-hero at the end of some major cultural era, making him great at his job and bad at
his family, providing a healthy dose of both sex and violence, and sprinkling in a smattering of highfalutin
cultural references to make sure the audience could recognize the show's pedigree and intelligence. I wrote an
updated version of that list in 2017 re ecting the way prestige has become as much about how we talk about
the genre as the genre itself. In 2015, as a bouncing-off point to de ne a genre he called "mid-reputable TV," the
critic Noel Murray laid out some "hallmarks of a prestige drama - heavy themes, high production values,
accomplished actors," before conceding that "prestige is a state of mind." Much of its de nition comes from a
show's reception, Murray concluded. "Prestige television is often subject to intense scrutiny, with fans and
critics evaluating every plot twist, stylistic choice, and coded method (in terms of both literary symbolism and
the show's attitudes about gender, race, and politics)." In an essay titled "How American TV Became
Respectable Without Getting Better," Matthew Christman offers a compelling de nition: "Mostly crime. Mostly
male. Mostly extravagantly unlikable antiheroes whose sheer awfulness makes us feel better about our own,
more mundane foibles."

I suspect some of the reason for prestige TV's simultaneous rise is that prestige, as a term, neatly dodges the
best-versus-favorite (quality vs enjoyment) problem. Rather than de ne television by the pretense of objective
greatness, the word prestige, with its snobbish implications, was indicative of a perhaps unintentional truth
about these shows. It is a classist assessment, one that implies inaccessibility, scarcity, but also widespread
in uence and importance. It's not easy or uffy or fun because, somewhere deep in the American psyche,
there's still a puritanical belief that fun things cannot also be serious. Prestige TV is demanding, something that
creates a further classist implication about its audience. It imagines itself being viewed by educated, wealthy
people with enough leisure hours to devote to epic run times and slow-burning, byzantine plots (it imagines this
regardless of who may actually be watching). It's not a coincidence that the outlets that de ned prestige TV are
also the ones, like HBO, that historically required an expensive monthly premium-cable package

Q.29 [30465398]
Which of the following can be inferred about the author's opinion on prestige TV?

a The author thinks that prestige TV's popular perception as exciting TV has declined, though it would always
carry an aesthetic aspiration.

https://www.aspiration.ai/LAW/sis/Solution.jsp?qsetId=JAmrn2p8H1A=&qsetName=LST Mock 27 2021 (CLAT) 54/223


1/17/2021 Mock Analysis

b The author, after an objective analysis, concludes that prestige TV is snobbish and classist.

c The author thinks that prestige TV is meant for an audience that has the time to watch slow-burning plots
with epic run times.

d The author thinks of prestige TV as something that does not offer much comfort and is di cult to
categorise.

Solution:
 Answer key/Solution
Correct Answer : d
Your Answer : c
(a) is eliminated because the author states in Para 2 that Prestige TV as an
aesthetic has persisted, and not that it will always be aspired to. (b) is incorrect because the author does not
conclude Prestige TV being snobbish and classist after looking at some objective evidence. Her tone has been
negative about Prestige shows right from the beginning (when she calls it a mirage). (d) can be clearly
inferred from Para 2. (c) cannot be inferred as it is the Prestige TV that imagines itself being watched by such
people, not the author.

FeedBack Bookmark

https://www.aspiration.ai/LAW/sis/Solution.jsp?qsetId=JAmrn2p8H1A=&qsetName=LST Mock 27 2021 (CLAT) 55/223


1/17/2021 Mock Analysis

Directions for questions 1 to 30: Each set of questions in this section is based on a single passage. Please
answer each question on the basis of what is stated or implied in the corresponding passage. In some
instances, more than one option may be the answer to the question; in such a case, please choose the option
that most accurately and comprehensively answers the question.

Passage – 6

Coronavirus and political anxiety have accelerated this shift toward comfort TV. Who wants to be frustrated or
made sadder by their entertainment when the outside world itself feels like a su ciently brutal place?

On the other side stands Prestige TV -- shows watched for the "prestige" they carry -- that has always been
di cult to categorize, in part because it is a mirage. Its gradual dissolution over the past several years is equally
tricky to nail to any speci c benchmark. But since the fever pitch of critical excitement over shows like Mad
Men, The Wire, and the early days of Game of Thrones, prestige TV as an aesthetic has persisted, even while
prestige as a reliable indicator of exciting TV you must talk about has undeniably waned.

In 2013, Vulture published "The 13 Rules for Creating a Prestige TV Drama," a full-feature list that began with
installing a middle-aged anti-hero at the end of some major cultural era, making him great at his job and bad at
his family, providing a healthy dose of both sex and violence, and sprinkling in a smattering of highfalutin
cultural references to make sure the audience could recognize the show's pedigree and intelligence. I wrote an
updated version of that list in 2017 re ecting the way prestige has become as much about how we talk about
the genre as the genre itself. In 2015, as a bouncing-off point to de ne a genre he called "mid-reputable TV," the
critic Noel Murray laid out some "hallmarks of a prestige drama - heavy themes, high production values,
accomplished actors," before conceding that "prestige is a state of mind." Much of its de nition comes from a
show's reception, Murray concluded. "Prestige television is often subject to intense scrutiny, with fans and
critics evaluating every plot twist, stylistic choice, and coded method (in terms of both literary symbolism and
the show's attitudes about gender, race, and politics)." In an essay titled "How American TV Became
Respectable Without Getting Better," Matthew Christman offers a compelling de nition: "Mostly crime. Mostly
male. Mostly extravagantly unlikable antiheroes whose sheer awfulness makes us feel better about our own,
more mundane foibles."

I suspect some of the reason for prestige TV's simultaneous rise is that prestige, as a term, neatly dodges the
best-versus-favorite (quality vs enjoyment) problem. Rather than de ne television by the pretense of objective
greatness, the word prestige, with its snobbish implications, was indicative of a perhaps unintentional truth
about these shows. It is a classist assessment, one that implies inaccessibility, scarcity, but also widespread
in uence and importance. It's not easy or uffy or fun because, somewhere deep in the American psyche,
there's still a puritanical belief that fun things cannot also be serious. Prestige TV is demanding, something that
creates a further classist implication about its audience. It imagines itself being viewed by educated, wealthy
people with enough leisure hours to devote to epic run times and slow-burning, byzantine plots (it imagines this
regardless of who may actually be watching). It's not a coincidence that the outlets that de ned prestige TV are
also the ones, like HBO, that historically required an expensive monthly premium-cable package

Q.30 [30465398]
What purpose does the last sentence of the last paragraph serve?

a It presents a point of criticism of prestige TV.

b It sums up the author's criticism of prestige TV as classist and snobbish.

https://www.aspiration.ai/LAW/sis/Solution.jsp?qsetId=JAmrn2p8H1A=&qsetName=LST Mock 27 2021 (CLAT) 56/223


1/17/2021 Mock Analysis

c It supports the assertion that outlets are making a lot of money by presenting prestige TV.

d It is an assertion that supports another assertion made earlier in the paragraph.

Solution:
 Answer key/Solution
Correct Answer : d
Your Answer : c
The last sentence supports the assertion that prestige TV is classist and
imagines its audience to be wealthy, stated earlier in the paragraph. Hence, (d). (a) is eliminated as this
assertion is a support of the criticism done earlier. (b) can be easily eliminated as the last sentence is not in
the nature of a summary or conclusion. (c) is distorted because the sentence's focus is not on the outlets
making a lot of money, but on their excluding people by requiring expensive packages, thus contributing to
prestige TV's classist element.

FeedBack Bookmark

Sec 2

https://www.aspiration.ai/LAW/sis/Solution.jsp?qsetId=JAmrn2p8H1A=&qsetName=LST Mock 27 2021 (CLAT) 57/223


1/17/2021 Mock Analysis

Passage-01

[1] didn’t meet all the action points laid out by the Financial Action Task Force (FATF), but avoided getting
blacklisted yet again. However, the [2]-based terror nancing watchdog kept the country in its ‘grey list’ at its
plenary that concluded.

The plenary, FATF’s highest decision-making body, will next meet in February 2021 and has given [1] time until
then to meet all the 27 parameters, especially those that pertain to imposing sanctions on terrorist out ts.

“[1] has made progress… It has largely completed 21 items of the 27; it de nitely means that the world has
become safer, but the six outstanding items mean the risks have not gone. The [1] government must do its best
to repair and work on these outstanding six items,” [3], the FATF president, said in a virtual press conference.

[3] said if [1] is able to act on the six outstanding points and complete all 27 by the next plenary, then an on-site
visit will be undertaken to verify them.

If the member countries are satis ed, then there are chances that by the following plenary, which takes place in
June 2021, [1], will be able to come off the grey list. The FATF’s plenary year begins in July and ends in June.

However, [3] also there is a parallel process going on under the Asia-Paci c Group on Money Laundering (APG),
which will decide on [1]’s performance on terror nancing.

“This is the norm with every country. There is no discrimination … We must treat [1] equally with other countries,”
he said.

“As long as we see that the country is progressing with action items, and we have seen progress with [1], we
give them a chance to repair the outstanding issues. But we don’t do this forever,” [3] warned.

The FATF president also said countries are “pushed to the black list” if they fail to meet all the parameters, and
no country is allowed to stay in the grey list permanently. Currently, [4] and [5] are on the FATF black list.

Q.31 [30465398]
What is the name of the country which has been kept continuously in the FATF’s ‘grey list’ redacted with [1] in the
passage above?

a Afghanistan

b Syria

c Algeria

d Pakistan

https://www.aspiration.ai/LAW/sis/Solution.jsp?qsetId=JAmrn2p8H1A=&qsetName=LST Mock 27 2021 (CLAT) 58/223


1/17/2021 Mock Analysis

Solution:
 Answer key/Solution
Correct Answer : d
Your Answer : d
Pakistan didn’t meet all the action points laid out by the Financial Action Task
Force (FATF), but avoided getting blacklisted yet again. However, the Paris-based terror nancing watchdog
kept the country in its ‘grey list’ at its plenary that concluded.

FeedBack Bookmark

Passage-01

[1] didn’t meet all the action points laid out by the Financial Action Task Force (FATF), but avoided getting
blacklisted yet again. However, the [2]-based terror nancing watchdog kept the country in its ‘grey list’ at its
plenary that concluded.

The plenary, FATF’s highest decision-making body, will next meet in February 2021 and has given [1] time until
then to meet all the 27 parameters, especially those that pertain to imposing sanctions on terrorist out ts.

“[1] has made progress… It has largely completed 21 items of the 27; it de nitely means that the world has
become safer, but the six outstanding items mean the risks have not gone. The [1] government must do its best
to repair and work on these outstanding six items,” [3], the FATF president, said in a virtual press conference.

[3] said if [1] is able to act on the six outstanding points and complete all 27 by the next plenary, then an on-site
visit will be undertaken to verify them.

If the member countries are satis ed, then there are chances that by the following plenary, which takes place in
June 2021, [1], will be able to come off the grey list. The FATF’s plenary year begins in July and ends in June.

However, [3] also there is a parallel process going on under the Asia-Paci c Group on Money Laundering (APG),
which will decide on [1]’s performance on terror nancing.

“This is the norm with every country. There is no discrimination … We must treat [1] equally with other countries,”
he said.

“As long as we see that the country is progressing with action items, and we have seen progress with [1], we
give them a chance to repair the outstanding issues. But we don’t do this forever,” [3] warned.

The FATF president also said countries are “pushed to the black list” if they fail to meet all the parameters, and
no country is allowed to stay in the grey list permanently. Currently, [4] and [5] are on the FATF black list.

Q.32 [30465398]
Which of the following is not an implication on being in the FATF’s grey list?

a It will be di cult for the country to get nancial aid from the IMF, the World Bank, the ADB and the
European Union.

https://www.aspiration.ai/LAW/sis/Solution.jsp?qsetId=JAmrn2p8H1A=&qsetName=LST Mock 27 2021 (CLAT) 59/223


1/17/2021 Mock Analysis

b This will further enhance problems for the nation which is in a precarious economic situation.

c There is every possibility that the global body may put the country in the ‘Black List’.

d None of the above

Solution:
 Answer key/Solution
Correct Answer : d
Your Answer : d
The Financial Action Task Force (FATF) has given an extension of four months to
Pakistan to act against organisations involved in terror nancing. With Pakistan’s continuation in the ‘Grey
List’, it will be di cult for the country to get nancial aid from the IMF, the World Bank, the ADB and the
European Union. This will further enhance problems for the nation which is in a precarious economic situation.
Also, there is every possibility that the global body may put the country in the ‘Black List’ along with North
Korea and Iran.

FeedBack Bookmark

https://www.aspiration.ai/LAW/sis/Solution.jsp?qsetId=JAmrn2p8H1A=&qsetName=LST Mock 27 2021 (CLAT) 60/223


1/17/2021 Mock Analysis

Passage-01

[1] didn’t meet all the action points laid out by the Financial Action Task Force (FATF), but avoided getting
blacklisted yet again. However, the [2]-based terror nancing watchdog kept the country in its ‘grey list’ at its
plenary that concluded.

The plenary, FATF’s highest decision-making body, will next meet in February 2021 and has given [1] time until
then to meet all the 27 parameters, especially those that pertain to imposing sanctions on terrorist out ts.

“[1] has made progress… It has largely completed 21 items of the 27; it de nitely means that the world has
become safer, but the six outstanding items mean the risks have not gone. The [1] government must do its best
to repair and work on these outstanding six items,” [3], the FATF president, said in a virtual press conference.

[3] said if [1] is able to act on the six outstanding points and complete all 27 by the next plenary, then an on-site
visit will be undertaken to verify them.

If the member countries are satis ed, then there are chances that by the following plenary, which takes place in
June 2021, [1], will be able to come off the grey list. The FATF’s plenary year begins in July and ends in June.

However, [3] also there is a parallel process going on under the Asia-Paci c Group on Money Laundering (APG),
which will decide on [1]’s performance on terror nancing.

“This is the norm with every country. There is no discrimination … We must treat [1] equally with other countries,”
he said.

“As long as we see that the country is progressing with action items, and we have seen progress with [1], we
give them a chance to repair the outstanding issues. But we don’t do this forever,” [3] warned.

The FATF president also said countries are “pushed to the black list” if they fail to meet all the parameters, and
no country is allowed to stay in the grey list permanently. Currently, [4] and [5] are on the FATF black list.

Q.33 [30465398]
Where is the headquarters of FATF located which has been redacted with [2] in the passage above?

a London

b Paris

c New York

d Geneva

https://www.aspiration.ai/LAW/sis/Solution.jsp?qsetId=JAmrn2p8H1A=&qsetName=LST Mock 27 2021 (CLAT) 61/223


1/17/2021 Mock Analysis

Solution:
 Answer key/Solution
Correct Answer : b
Your Answer : b
The Financial Action Task Force (FATF) is the global money laundering and
terrorist nancing watchdog. The inter-governmental body sets international standards that aim to prevent
these illegal activities and the harm they cause to society. As a policy-making body, the FATF works to
generate the necessary political will to bring about national legislative and regulatory reforms in these areas.
The FATF is headquartered in Paris.

FeedBack Bookmark

Passage-01

[1] didn’t meet all the action points laid out by the Financial Action Task Force (FATF), but avoided getting
blacklisted yet again. However, the [2]-based terror nancing watchdog kept the country in its ‘grey list’ at its
plenary that concluded.

The plenary, FATF’s highest decision-making body, will next meet in February 2021 and has given [1] time until
then to meet all the 27 parameters, especially those that pertain to imposing sanctions on terrorist out ts.

“[1] has made progress… It has largely completed 21 items of the 27; it de nitely means that the world has
become safer, but the six outstanding items mean the risks have not gone. The [1] government must do its best
to repair and work on these outstanding six items,” [3], the FATF president, said in a virtual press conference.

[3] said if [1] is able to act on the six outstanding points and complete all 27 by the next plenary, then an on-site
visit will be undertaken to verify them.

If the member countries are satis ed, then there are chances that by the following plenary, which takes place in
June 2021, [1], will be able to come off the grey list. The FATF’s plenary year begins in July and ends in June.

However, [3] also there is a parallel process going on under the Asia-Paci c Group on Money Laundering (APG),
which will decide on [1]’s performance on terror nancing.

“This is the norm with every country. There is no discrimination … We must treat [1] equally with other countries,”
he said.

“As long as we see that the country is progressing with action items, and we have seen progress with [1], we
give them a chance to repair the outstanding issues. But we don’t do this forever,” [3] warned.

The FATF president also said countries are “pushed to the black list” if they fail to meet all the parameters, and
no country is allowed to stay in the grey list permanently. Currently, [4] and [5] are on the FATF black list.

Q.34 [30465398]
Who is the current President of FATF whose name has been redacted with [3] in the passage above?

a Dr. Marcus Pleyer

https://www.aspiration.ai/LAW/sis/Solution.jsp?qsetId=JAmrn2p8H1A=&qsetName=LST Mock 27 2021 (CLAT) 62/223


1/17/2021 Mock Analysis

b Xiangmin Liu

c Shinzo Abe

d David Malapas

Solution:
 Answer key/Solution
Correct Answer : a
Your Answer : a
Dr. Marcus Pleyer of Germany assumed the position of President of the FATF on 1
July 2020. He succeeded Xiangmin Liu of the People's Republic of China. “Pakistan has made progress. It has
largely completed 21 items of the 27; it de nitely means that the world has become safer, but the six
outstanding items mean the risks have not gone. The Pakistan government must do its best to repair and work
on these outstanding six items,” Marcus Pleyer, the FATF president, said in a virtual press conference.

FeedBack Bookmark

https://www.aspiration.ai/LAW/sis/Solution.jsp?qsetId=JAmrn2p8H1A=&qsetName=LST Mock 27 2021 (CLAT) 63/223


1/17/2021 Mock Analysis

Passage-01

[1] didn’t meet all the action points laid out by the Financial Action Task Force (FATF), but avoided getting
blacklisted yet again. However, the [2]-based terror nancing watchdog kept the country in its ‘grey list’ at its
plenary that concluded.

The plenary, FATF’s highest decision-making body, will next meet in February 2021 and has given [1] time until
then to meet all the 27 parameters, especially those that pertain to imposing sanctions on terrorist out ts.

“[1] has made progress… It has largely completed 21 items of the 27; it de nitely means that the world has
become safer, but the six outstanding items mean the risks have not gone. The [1] government must do its best
to repair and work on these outstanding six items,” [3], the FATF president, said in a virtual press conference.

[3] said if [1] is able to act on the six outstanding points and complete all 27 by the next plenary, then an on-site
visit will be undertaken to verify them.

If the member countries are satis ed, then there are chances that by the following plenary, which takes place in
June 2021, [1], will be able to come off the grey list. The FATF’s plenary year begins in July and ends in June.

However, [3] also there is a parallel process going on under the Asia-Paci c Group on Money Laundering (APG),
which will decide on [1]’s performance on terror nancing.

“This is the norm with every country. There is no discrimination … We must treat [1] equally with other countries,”
he said.

“As long as we see that the country is progressing with action items, and we have seen progress with [1], we
give them a chance to repair the outstanding issues. But we don’t do this forever,” [3] warned.

The FATF president also said countries are “pushed to the black list” if they fail to meet all the parameters, and
no country is allowed to stay in the grey list permanently. Currently, [4] and [5] are on the FATF black list.

Q.35 [30465398]
What are the names of the two countries which are currently on the FATF black list redacted with [4] and [5] in
the passage above?

a Libya and South Sudan

b South Sudan and South Korea

c Iran and North Korea

d Iraq and North Korea

https://www.aspiration.ai/LAW/sis/Solution.jsp?qsetId=JAmrn2p8H1A=&qsetName=LST Mock 27 2021 (CLAT) 64/223


1/17/2021 Mock Analysis

Solution:
 Answer key/Solution
Correct Answer : c
Your Answer : c
The FATF President, Dr. Marcus Pleyer also said countries are “pushed to the
black list” if they fail to meet all the parameters, and no country is allowed to stay in the grey list permanently.
Currently, Iran and North Korea are on the FATF black list.

FeedBack Bookmark

Passage-01

[1] didn’t meet all the action points laid out by the Financial Action Task Force (FATF), but avoided getting
blacklisted yet again. However, the [2]-based terror nancing watchdog kept the country in its ‘grey list’ at its
plenary that concluded.

The plenary, FATF’s highest decision-making body, will next meet in February 2021 and has given [1] time until
then to meet all the 27 parameters, especially those that pertain to imposing sanctions on terrorist out ts.

“[1] has made progress… It has largely completed 21 items of the 27; it de nitely means that the world has
become safer, but the six outstanding items mean the risks have not gone. The [1] government must do its best
to repair and work on these outstanding six items,” [3], the FATF president, said in a virtual press conference.

[3] said if [1] is able to act on the six outstanding points and complete all 27 by the next plenary, then an on-site
visit will be undertaken to verify them.

If the member countries are satis ed, then there are chances that by the following plenary, which takes place in
June 2021, [1], will be able to come off the grey list. The FATF’s plenary year begins in July and ends in June.

However, [3] also there is a parallel process going on under the Asia-Paci c Group on Money Laundering (APG),
which will decide on [1]’s performance on terror nancing.

“This is the norm with every country. There is no discrimination … We must treat [1] equally with other countries,”
he said.

“As long as we see that the country is progressing with action items, and we have seen progress with [1], we
give them a chance to repair the outstanding issues. But we don’t do this forever,” [3] warned.

The FATF president also said countries are “pushed to the black list” if they fail to meet all the parameters, and
no country is allowed to stay in the grey list permanently. Currently, [4] and [5] are on the FATF black list.

Q.36 [30465398]
In response to mounting concern over money laundering, the Financial Action Task Force on Money Laundering
(FATF) was established by the __________that was held in 1989.

a UN-Summit

https://www.aspiration.ai/LAW/sis/Solution.jsp?qsetId=JAmrn2p8H1A=&qsetName=LST Mock 27 2021 (CLAT) 65/223


1/17/2021 Mock Analysis

b G-7 Summit

c G-20 Summit

d G-77 Summit

Solution:
 Answer key/Solution
Correct Answer : b
Your Answer : b
In response to mounting concern over money laundering, the Financial Action
Task Force on Money Laundering (FATF) was established by the G-7 Summit that was held in Paris in 1989.
Recognising the threat posed to the banking system and to nancial institutions, the G-7 Heads of State or
Government and President of the European Commission convened the Task Force from the G-7 member
States, the European Commission and eight other countries.

FeedBack Bookmark

https://www.aspiration.ai/LAW/sis/Solution.jsp?qsetId=JAmrn2p8H1A=&qsetName=LST Mock 27 2021 (CLAT) 66/223


1/17/2021 Mock Analysis

Passage-02

The Madhya Pradesh government is set to follow two other BJP-ruled states — Uttar Pradesh and Himachal
Pradesh — in passing an anti-conversion law that outlaws religious conversion solely for the purpose of
marriage. The MP Cabinet has approved the Freedom to Religion Bill, 2020 as an Ordinance.

While a common feature of all three laws is the declaration of such marriages as “null and void” and the
penalising of conversions done without the prior approval of the state, they differ in the quantum of punishment
prescribed, and in attributing the burden of proof that a conversion is lawful. Also, the MP law seeks to protect
the rights of women of such marriages.

The MP law requires a 60-day prior “declaration of the intention to convert” to the District Magistrate for
conversion to be valid, following which a couple from different religions can be legally married.

The Uttar Pradesh Prohibition of Unlawful Conversion of Religious Ordinance, 2020 promulgated in November,
too requires a 60-day notice but also requires the Magistrate to conduct a police inquiry to ascertain the real
intention behind the conversion.

The Himachal Pradesh Freedom of Religion Act, 2019 that came into effect last week, requires a 30-day prior
“declaration of intention to convert”.

The offence of illegal conversion under the laws of all three states is cognisable and non-bailable, which means
an arrest can be made without a warrant and bail is granted only by the discretion of the judge.

Under the Madhya Pradesh law, a person can be sentenced to a jail term between one and ve years for
converting or attempting to convert unlawfully. If the person converted is a woman, a minor or a person
belonging to a Scheduled Caste or Scheduled Tribe (SC/ST), the sentence is two to 10 years. It also provides for
a jail term of [1] years for concealing one’s religion during the marriage.

The UP law provides for a minimum punishment of one year, which can be extended up to ve years, and repeat
offences can carry double the maximum sentence. Men are awarded a higher punishment if convicted of
causing conversion of a woman, a minor or a person belonging to an SC/ST — in which case the sentence is
between two and 10 years.

Q.37 [30465398]
According to the Supreme Court judgment, the right to marry a person of one’s choice is integral to Article
__________ of the Indian Constitution.

a Article 20

b Article 21

c Article 22

d Article 25

https://www.aspiration.ai/LAW/sis/Solution.jsp?qsetId=JAmrn2p8H1A=&qsetName=LST Mock 27 2021 (CLAT) 67/223


1/17/2021 Mock Analysis

Solution:
 Answer key/Solution
Correct Answer : b
Your Answer : b
Article 21 of the constitution guarantees individuals the right to marry a person of
one’s choice. “The right to marry a person of one’s choice is integral to Article 21 (right to life and liberty) of
the Constitution”, the Supreme Court said and set aside a 2017 order of the Kerala High Court which annulled
the marriage of Kerala Muslim convert girl Hadiya and She n Jahan. The three-judge bench delivered two
separate detailed concurring judgments, one by Chief Justice Dipak Misra and Justice A M Khanwilkar and the
other by Justice D Y Chandrachud. According to the Supreme Court, the Constitution protects the ability of
each individual to pursue a way of life or faith to which she or he seeks to adhere. Therefore, the right to marry
a person of one’s choice is integral to Article 21 of the Constitution.

FeedBack Bookmark

https://www.aspiration.ai/LAW/sis/Solution.jsp?qsetId=JAmrn2p8H1A=&qsetName=LST Mock 27 2021 (CLAT) 68/223


1/17/2021 Mock Analysis

Passage-02

The Madhya Pradesh government is set to follow two other BJP-ruled states — Uttar Pradesh and Himachal
Pradesh — in passing an anti-conversion law that outlaws religious conversion solely for the purpose of
marriage. The MP Cabinet has approved the Freedom to Religion Bill, 2020 as an Ordinance.

While a common feature of all three laws is the declaration of such marriages as “null and void” and the
penalising of conversions done without the prior approval of the state, they differ in the quantum of punishment
prescribed, and in attributing the burden of proof that a conversion is lawful. Also, the MP law seeks to protect
the rights of women of such marriages.

The MP law requires a 60-day prior “declaration of the intention to convert” to the District Magistrate for
conversion to be valid, following which a couple from different religions can be legally married.

The Uttar Pradesh Prohibition of Unlawful Conversion of Religious Ordinance, 2020 promulgated in November,
too requires a 60-day notice but also requires the Magistrate to conduct a police inquiry to ascertain the real
intention behind the conversion.

The Himachal Pradesh Freedom of Religion Act, 2019 that came into effect last week, requires a 30-day prior
“declaration of intention to convert”.

The offence of illegal conversion under the laws of all three states is cognisable and non-bailable, which means
an arrest can be made without a warrant and bail is granted only by the discretion of the judge.

Under the Madhya Pradesh law, a person can be sentenced to a jail term between one and ve years for
converting or attempting to convert unlawfully. If the person converted is a woman, a minor or a person
belonging to a Scheduled Caste or Scheduled Tribe (SC/ST), the sentence is two to 10 years. It also provides for
a jail term of [1] years for concealing one’s religion during the marriage.

The UP law provides for a minimum punishment of one year, which can be extended up to ve years, and repeat
offences can carry double the maximum sentence. Men are awarded a higher punishment if convicted of
causing conversion of a woman, a minor or a person belonging to an SC/ST — in which case the sentence is
between two and 10 years.

Q.38 [30465398]
Which of the following articles of the Indian Constitution guarantees that all persons have equal right to freely
profess, practice and propagate religion, subject to public order, morality, health and other provisions of Part-III
of the Constitution?

a Article 14

b Article 21

c Article 25

d Article 26

https://www.aspiration.ai/LAW/sis/Solution.jsp?qsetId=JAmrn2p8H1A=&qsetName=LST Mock 27 2021 (CLAT) 69/223


1/17/2021 Mock Analysis

Solution:
 Answer key/Solution
Correct Answer : c
Your Answer : c
Article 25 provides that all persons have equal right to freely profess, practice and
propagate religion, subject to public order, morality, health and other provisions of Part-III of the Constitution.
This article also gives a provision that the State can make laws: That regulates and restricts any nancial,
economic, political or other secular activity associated with any religious practice.

FeedBack Bookmark

Passage-02

The Madhya Pradesh government is set to follow two other BJP-ruled states — Uttar Pradesh and Himachal
Pradesh — in passing an anti-conversion law that outlaws religious conversion solely for the purpose of
marriage. The MP Cabinet has approved the Freedom to Religion Bill, 2020 as an Ordinance.

While a common feature of all three laws is the declaration of such marriages as “null and void” and the
penalising of conversions done without the prior approval of the state, they differ in the quantum of punishment
prescribed, and in attributing the burden of proof that a conversion is lawful. Also, the MP law seeks to protect
the rights of women of such marriages.

The MP law requires a 60-day prior “declaration of the intention to convert” to the District Magistrate for
conversion to be valid, following which a couple from different religions can be legally married.

The Uttar Pradesh Prohibition of Unlawful Conversion of Religious Ordinance, 2020 promulgated in November,
too requires a 60-day notice but also requires the Magistrate to conduct a police inquiry to ascertain the real
intention behind the conversion.

The Himachal Pradesh Freedom of Religion Act, 2019 that came into effect last week, requires a 30-day prior
“declaration of intention to convert”.

The offence of illegal conversion under the laws of all three states is cognisable and non-bailable, which means
an arrest can be made without a warrant and bail is granted only by the discretion of the judge.

Under the Madhya Pradesh law, a person can be sentenced to a jail term between one and ve years for
converting or attempting to convert unlawfully. If the person converted is a woman, a minor or a person
belonging to a Scheduled Caste or Scheduled Tribe (SC/ST), the sentence is two to 10 years. It also provides for
a jail term of [1] years for concealing one’s religion during the marriage.

The UP law provides for a minimum punishment of one year, which can be extended up to ve years, and repeat
offences can carry double the maximum sentence. Men are awarded a higher punishment if convicted of
causing conversion of a woman, a minor or a person belonging to an SC/ST — in which case the sentence is
between two and 10 years.

https://www.aspiration.ai/LAW/sis/Solution.jsp?qsetId=JAmrn2p8H1A=&qsetName=LST Mock 27 2021 (CLAT) 70/223


1/17/2021 Mock Analysis

Q.39 [30465398]
The word ‘secular’ was added to the Indian Constitution by which of the following amendment acts?

a 22nd amendment act

b 32nd amendment act

c 42nd amendment act

d 44th amendment act

Solution:
 Answer key/Solution
Correct Answer : c
Your Answer : c
The word ‘secularism’ means separate from religion. It entails the separation of
religion from the government, social, economic and cultural aspects of life. Here religion is an entirely
personal matter. India is a secular country with no State religion. However, this in India also means that there
is equal respect for all religions and faiths. The word is also a part of the Basic Structure of the Constitution. It
was added by the 42nd Amendment to the Constitution. The 42nd Amendment changed the description of
India from a "sovereign democratic republic" to a "sovereign, socialist secular democratic republic", and also
changed the words "unity of the nation" to "unity and integrity of the nation".
FeedBack Bookmark

https://www.aspiration.ai/LAW/sis/Solution.jsp?qsetId=JAmrn2p8H1A=&qsetName=LST Mock 27 2021 (CLAT) 71/223


1/17/2021 Mock Analysis

Passage-02

The Madhya Pradesh government is set to follow two other BJP-ruled states — Uttar Pradesh and Himachal
Pradesh — in passing an anti-conversion law that outlaws religious conversion solely for the purpose of
marriage. The MP Cabinet has approved the Freedom to Religion Bill, 2020 as an Ordinance.

While a common feature of all three laws is the declaration of such marriages as “null and void” and the
penalising of conversions done without the prior approval of the state, they differ in the quantum of punishment
prescribed, and in attributing the burden of proof that a conversion is lawful. Also, the MP law seeks to protect
the rights of women of such marriages.

The MP law requires a 60-day prior “declaration of the intention to convert” to the District Magistrate for
conversion to be valid, following which a couple from different religions can be legally married.

The Uttar Pradesh Prohibition of Unlawful Conversion of Religious Ordinance, 2020 promulgated in November,
too requires a 60-day notice but also requires the Magistrate to conduct a police inquiry to ascertain the real
intention behind the conversion.

The Himachal Pradesh Freedom of Religion Act, 2019 that came into effect last week, requires a 30-day prior
“declaration of intention to convert”.

The offence of illegal conversion under the laws of all three states is cognisable and non-bailable, which means
an arrest can be made without a warrant and bail is granted only by the discretion of the judge.

Under the Madhya Pradesh law, a person can be sentenced to a jail term between one and ve years for
converting or attempting to convert unlawfully. If the person converted is a woman, a minor or a person
belonging to a Scheduled Caste or Scheduled Tribe (SC/ST), the sentence is two to 10 years. It also provides for
a jail term of [1] years for concealing one’s religion during the marriage.

The UP law provides for a minimum punishment of one year, which can be extended up to ve years, and repeat
offences can carry double the maximum sentence. Men are awarded a higher punishment if convicted of
causing conversion of a woman, a minor or a person belonging to an SC/ST — in which case the sentence is
between two and 10 years.

Q.40 [30465398]
Which of the following religions has not been awarded the "National minority" status by the Government of
India?

a Christians

b Jains

c Zoroastrians

d Jews

https://www.aspiration.ai/LAW/sis/Solution.jsp?qsetId=JAmrn2p8H1A=&qsetName=LST Mock 27 2021 (CLAT) 72/223


1/17/2021 Mock Analysis

Solution:
 Answer key/Solution
Correct Answer : d
Your Answer : d
Religion in India is characterised by a diversity of religious beliefs and practices.
The preamble of the Indian constitution states that India is a secular state. The Indian subcontinent is the
birthplace of four of the world's major religions: namely Hinduism, Buddhism, Jainism, and Sikhism—
collectively known as Dharmic religions that believe Moksha is the most supreme state of the Atman (soul).
There are six religions in India which have been awarded "National minority" status—Muslims, Christians,
Sikhs, Jains, Buddhists and Zoroastrians.

FeedBack Bookmark

Passage-02

The Madhya Pradesh government is set to follow two other BJP-ruled states — Uttar Pradesh and Himachal
Pradesh — in passing an anti-conversion law that outlaws religious conversion solely for the purpose of
marriage. The MP Cabinet has approved the Freedom to Religion Bill, 2020 as an Ordinance.

While a common feature of all three laws is the declaration of such marriages as “null and void” and the
penalising of conversions done without the prior approval of the state, they differ in the quantum of punishment
prescribed, and in attributing the burden of proof that a conversion is lawful. Also, the MP law seeks to protect
the rights of women of such marriages.

The MP law requires a 60-day prior “declaration of the intention to convert” to the District Magistrate for
conversion to be valid, following which a couple from different religions can be legally married.

The Uttar Pradesh Prohibition of Unlawful Conversion of Religious Ordinance, 2020 promulgated in November,
too requires a 60-day notice but also requires the Magistrate to conduct a police inquiry to ascertain the real
intention behind the conversion.

The Himachal Pradesh Freedom of Religion Act, 2019 that came into effect last week, requires a 30-day prior
“declaration of intention to convert”.

The offence of illegal conversion under the laws of all three states is cognisable and non-bailable, which means
an arrest can be made without a warrant and bail is granted only by the discretion of the judge.

Under the Madhya Pradesh law, a person can be sentenced to a jail term between one and ve years for
converting or attempting to convert unlawfully. If the person converted is a woman, a minor or a person
belonging to a Scheduled Caste or Scheduled Tribe (SC/ST), the sentence is two to 10 years. It also provides for
a jail term of [1] years for concealing one’s religion during the marriage.

The UP law provides for a minimum punishment of one year, which can be extended up to ve years, and repeat
offences can carry double the maximum sentence. Men are awarded a higher punishment if convicted of
causing conversion of a woman, a minor or a person belonging to an SC/ST — in which case the sentence is
between two and 10 years.

https://www.aspiration.ai/LAW/sis/Solution.jsp?qsetId=JAmrn2p8H1A=&qsetName=LST Mock 27 2021 (CLAT) 73/223


1/17/2021 Mock Analysis

Q.41 [30465398]
Under the Madhya Pradesh new bill, what is the jail term for concealing one’s religion during the marriage which
has been redacted with[ 1] in the passage above?

a Two to ve years

b Three to ten years

c Three to 7 years

d Two to 12 years

Solution:
 Answer key/Solution
Correct Answer : b
Under the Madhya Pradesh law, a person can be sentenced to a jail term between
one and ve years for converting or attempting to convert unlawfully. If the
person converted is a woman, a minor or a person belonging to a Scheduled Caste or Scheduled Tribe
(SC/ST), the sentence is two to 10 years. It also provides for a jail term of three to 10 years for concealing
one’s religion during the marriage.

FeedBack Bookmark

Passage-03
[1], who had scored 20,396 runs in all international cricket during the time period, which is more than anyone
else, was also part of team India that won the ICC CWC 2011 and ICC Champions Trophy 2013. He was also
named ICC Cricketer of the Year in both 2017 and 2018, so it was no surprise that he emerged as the clear jury
favourite with winning the Sir Gar eld Sobers Award for ICC Male Cricketer of the Decade. All-rounder, [2] was a
clear favourite in the Female Cricketer of the Decade category and won the Rachael Heyhoe-Flint Award, as she
had scored 4,349 runs with four centuries and took 213 wickets in all international cricket during the decade –
which was also the most wickets taken by any player. [2] was also part of the Australian team that won ICC
Women’s Cricket World Cup in 2013 and ICC Women’s T20 World Cup in 2012, 2014, 2018 and 2020 and also
won the Rachael Heyhoe Flint Award for leading Women’s Cricketer in 2017 and 2019. She also went on to win
the ICC Women’s ODI Cricketer of the Decade as well as the ICC Women’s T20I Cricketer of the Decade.

[1] was also awarded the ICC Men’s ODI Cricketer of the Decade, whilst [3] bagged the ICC Men’s Test Cricketer
of the Decade for scoring 7,040 runs in 69 Tests in the period at an average of 65.79 with 26 centuries. He was
also named the ICC Player of the Year in 2015 and ICC Test Player of the Year in 2015 and 2017. In the MRF
Tyres ICC Test Player Rankings, his points tally was of 947 at the end of 2017 which is the second-highest in the
history of the ICC Test batting rankings history.

[4] took home the ICC Men’s T20I Cricketer of the Decade award, for taking 89 wickets at an average of just
12.62 apiece with an economy rate of 6.14 runs per over in 48 matches during the decade.

[5] won the ICC Spirit of Cricket Award of the Decade, chosen by fans unanimously for his gesture of calling
back former England batter Ian Bell following a disputable runout at Trent Bridge in 2011.

https://www.aspiration.ai/LAW/sis/Solution.jsp?qsetId=JAmrn2p8H1A=&qsetName=LST Mock 27 2021 (CLAT) 74/223


1/17/2021 Mock Analysis

Q.42 [30465398]
Who won the Sir Gar eld Sobers Award for ICC Male Cricketer of the Decade whose name has been redacted
with [1] in the passage above?

a Virat Kohli

b Rohit Sharma

c AB de Villiers

d M.S.Dhoni

Solution:
 Answer key/Solution
Correct Answer : a
Your Answer : a
Virat Kohli, who had scored 20,396 runs in all international cricket during the time
period, which is more than anyone else, was also part of team India that won the ICC CWC 2011 and ICC
Champions Trophy 2013. He was also named ICC Cricketer of the Year in both 2017 and 2018, so it was no
surprise that he emerged as the clear jury favourite with winning the Sir Gar eld Sobers Award for ICC Male
Cricketer of the Decade.

FeedBack Bookmark

https://www.aspiration.ai/LAW/sis/Solution.jsp?qsetId=JAmrn2p8H1A=&qsetName=LST Mock 27 2021 (CLAT) 75/223


1/17/2021 Mock Analysis

Passage-03
[1], who had scored 20,396 runs in all international cricket during the time period, which is more than anyone
else, was also part of team India that won the ICC CWC 2011 and ICC Champions Trophy 2013. He was also
named ICC Cricketer of the Year in both 2017 and 2018, so it was no surprise that he emerged as the clear jury
favourite with winning the Sir Gar eld Sobers Award for ICC Male Cricketer of the Decade. All-rounder, [2] was a
clear favourite in the Female Cricketer of the Decade category and won the Rachael Heyhoe-Flint Award, as she
had scored 4,349 runs with four centuries and took 213 wickets in all international cricket during the decade –
which was also the most wickets taken by any player. [2] was also part of the Australian team that won ICC
Women’s Cricket World Cup in 2013 and ICC Women’s T20 World Cup in 2012, 2014, 2018 and 2020 and also
won the Rachael Heyhoe Flint Award for leading Women’s Cricketer in 2017 and 2019. She also went on to win
the ICC Women’s ODI Cricketer of the Decade as well as the ICC Women’s T20I Cricketer of the Decade.

[1] was also awarded the ICC Men’s ODI Cricketer of the Decade, whilst [3] bagged the ICC Men’s Test Cricketer
of the Decade for scoring 7,040 runs in 69 Tests in the period at an average of 65.79 with 26 centuries. He was
also named the ICC Player of the Year in 2015 and ICC Test Player of the Year in 2015 and 2017. In the MRF
Tyres ICC Test Player Rankings, his points tally was of 947 at the end of 2017 which is the second-highest in the
history of the ICC Test batting rankings history.

[4] took home the ICC Men’s T20I Cricketer of the Decade award, for taking 89 wickets at an average of just
12.62 apiece with an economy rate of 6.14 runs per over in 48 matches during the decade.

[5] won the ICC Spirit of Cricket Award of the Decade, chosen by fans unanimously for his gesture of calling
back former England batter Ian Bell following a disputable runout at Trent Bridge in 2011.

Q.43 [30465398]
Who won Female Cricketer of the Decade category and Rachael Heyhoe-Flint Award whose name has been
redacted with [2] in the passage above?

a Mithali Raj

b Ellyse Perry

c Sarah Taylor

d Stafanie Taylor

https://www.aspiration.ai/LAW/sis/Solution.jsp?qsetId=JAmrn2p8H1A=&qsetName=LST Mock 27 2021 (CLAT) 76/223


1/17/2021 Mock Analysis

Solution:
 Answer key/Solution
Correct Answer : b
Your Answer : b
All-rounder, Ellyse Perry was a clear favourite in the Female Cricketer of the
Decade category and won the Rachael Heyhoe-Flint Award, as she had scored 4,349 runs with four centuries
and took 213 wickets in all international cricket during the decade – which was also the most wickets taken by
any player. Perry was also part of the Australian team that won ICC Women’s Cricket World Cup in 2013 and
ICC Women’s T20 World Cup in 2012, 2014, 2018 and 2020 and also won the Rachael Heyhoe Flint Award for
leading Women’s Cricketer in 2017 and 2019. She also went on to win the ICC Women’s ODI Cricketer of the
Decade as well as the ICC Women’s T20I Cricketer of the Decade.

FeedBack Bookmark

Passage-03
[1], who had scored 20,396 runs in all international cricket during the time period, which is more than anyone
else, was also part of team India that won the ICC CWC 2011 and ICC Champions Trophy 2013. He was also
named ICC Cricketer of the Year in both 2017 and 2018, so it was no surprise that he emerged as the clear jury
favourite with winning the Sir Gar eld Sobers Award for ICC Male Cricketer of the Decade. All-rounder, [2] was a
clear favourite in the Female Cricketer of the Decade category and won the Rachael Heyhoe-Flint Award, as she
had scored 4,349 runs with four centuries and took 213 wickets in all international cricket during the decade –
which was also the most wickets taken by any player. [2] was also part of the Australian team that won ICC
Women’s Cricket World Cup in 2013 and ICC Women’s T20 World Cup in 2012, 2014, 2018 and 2020 and also
won the Rachael Heyhoe Flint Award for leading Women’s Cricketer in 2017 and 2019. She also went on to win
the ICC Women’s ODI Cricketer of the Decade as well as the ICC Women’s T20I Cricketer of the Decade.

[1] was also awarded the ICC Men’s ODI Cricketer of the Decade, whilst [3] bagged the ICC Men’s Test Cricketer
of the Decade for scoring 7,040 runs in 69 Tests in the period at an average of 65.79 with 26 centuries. He was
also named the ICC Player of the Year in 2015 and ICC Test Player of the Year in 2015 and 2017. In the MRF
Tyres ICC Test Player Rankings, his points tally was of 947 at the end of 2017 which is the second-highest in the
history of the ICC Test batting rankings history.

[4] took home the ICC Men’s T20I Cricketer of the Decade award, for taking 89 wickets at an average of just
12.62 apiece with an economy rate of 6.14 runs per over in 48 matches during the decade.

[5] won the ICC Spirit of Cricket Award of the Decade, chosen by fans unanimously for his gesture of calling
back former England batter Ian Bell following a disputable runout at Trent Bridge in 2011.

Q.44 [30465398]
Who won the ICC Men’s Test Cricketer of the Decade award whose name has been redacted with [3] in the
passage above?

a James Anderson

b Steve Smith

c Kane Williamson

https://www.aspiration.ai/LAW/sis/Solution.jsp?qsetId=JAmrn2p8H1A=&qsetName=LST Mock 27 2021 (CLAT) 77/223


1/17/2021 Mock Analysis

d Joe Root

Solution:
 Answer key/Solution
Correct Answer : b
Your Answer : b
Steve Smith bagged the ICC Men’s Test Cricketer of the Decade for scoring 7,040
runs in 69 Tests in the period at an average of 65.79 with 26 centuries. He was also named the ICC Player of
the Year in 2015 and ICC Test Player of the Year in 2015 and 2017. In the MRF Tyres ICC Test Player Rankings,
his points tally was of 947 at the end of 2017 which is the second-highest in the history of the ICC Test batting
rankings history.

FeedBack Bookmark

Passage-03
[1], who had scored 20,396 runs in all international cricket during the time period, which is more than anyone
else, was also part of team India that won the ICC CWC 2011 and ICC Champions Trophy 2013. He was also
named ICC Cricketer of the Year in both 2017 and 2018, so it was no surprise that he emerged as the clear jury
favourite with winning the Sir Gar eld Sobers Award for ICC Male Cricketer of the Decade. All-rounder, [2] was a
clear favourite in the Female Cricketer of the Decade category and won the Rachael Heyhoe-Flint Award, as she
had scored 4,349 runs with four centuries and took 213 wickets in all international cricket during the decade –
which was also the most wickets taken by any player. [2] was also part of the Australian team that won ICC
Women’s Cricket World Cup in 2013 and ICC Women’s T20 World Cup in 2012, 2014, 2018 and 2020 and also
won the Rachael Heyhoe Flint Award for leading Women’s Cricketer in 2017 and 2019. She also went on to win
the ICC Women’s ODI Cricketer of the Decade as well as the ICC Women’s T20I Cricketer of the Decade.

[1] was also awarded the ICC Men’s ODI Cricketer of the Decade, whilst [3] bagged the ICC Men’s Test Cricketer
of the Decade for scoring 7,040 runs in 69 Tests in the period at an average of 65.79 with 26 centuries. He was
also named the ICC Player of the Year in 2015 and ICC Test Player of the Year in 2015 and 2017. In the MRF
Tyres ICC Test Player Rankings, his points tally was of 947 at the end of 2017 which is the second-highest in the
history of the ICC Test batting rankings history.

[4] took home the ICC Men’s T20I Cricketer of the Decade award, for taking 89 wickets at an average of just
12.62 apiece with an economy rate of 6.14 runs per over in 48 matches during the decade.

[5] won the ICC Spirit of Cricket Award of the Decade, chosen by fans unanimously for his gesture of calling
back former England batter Ian Bell following a disputable runout at Trent Bridge in 2011.

Q.45 [30465398]
Who won the ICC Men’s T20I Cricketer of the Decade award whose name has been redacted with [4] in the
passage above?

a Chris Gayle

b Rashid Khan

https://www.aspiration.ai/LAW/sis/Solution.jsp?qsetId=JAmrn2p8H1A=&qsetName=LST Mock 27 2021 (CLAT) 78/223


1/17/2021 Mock Analysis

c Rohit Sharma

d AB de Villiers

Solution:
 Answer key/Solution
Correct Answer : b
Your Answer : b
Rashid Khan took home the ICC Men’s T20I Cricketer of the Decade award, for
taking 89 wickets at an average of just 12.62 apiece with an economy rate of 6.14 runs per over in 48 matches
during the decade.

FeedBack Bookmark

Passage-03
[1], who had scored 20,396 runs in all international cricket during the time period, which is more than anyone
else, was also part of team India that won the ICC CWC 2011 and ICC Champions Trophy 2013. He was also
named ICC Cricketer of the Year in both 2017 and 2018, so it was no surprise that he emerged as the clear jury
favourite with winning the Sir Gar eld Sobers Award for ICC Male Cricketer of the Decade. All-rounder, [2] was a
clear favourite in the Female Cricketer of the Decade category and won the Rachael Heyhoe-Flint Award, as she
had scored 4,349 runs with four centuries and took 213 wickets in all international cricket during the decade –
which was also the most wickets taken by any player. [2] was also part of the Australian team that won ICC
Women’s Cricket World Cup in 2013 and ICC Women’s T20 World Cup in 2012, 2014, 2018 and 2020 and also
won the Rachael Heyhoe Flint Award for leading Women’s Cricketer in 2017 and 2019. She also went on to win
the ICC Women’s ODI Cricketer of the Decade as well as the ICC Women’s T20I Cricketer of the Decade.

[1] was also awarded the ICC Men’s ODI Cricketer of the Decade, whilst [3] bagged the ICC Men’s Test Cricketer
of the Decade for scoring 7,040 runs in 69 Tests in the period at an average of 65.79 with 26 centuries. He was
also named the ICC Player of the Year in 2015 and ICC Test Player of the Year in 2015 and 2017. In the MRF
Tyres ICC Test Player Rankings, his points tally was of 947 at the end of 2017 which is the second-highest in the
history of the ICC Test batting rankings history.

[4] took home the ICC Men’s T20I Cricketer of the Decade award, for taking 89 wickets at an average of just
12.62 apiece with an economy rate of 6.14 runs per over in 48 matches during the decade.

[5] won the ICC Spirit of Cricket Award of the Decade, chosen by fans unanimously for his gesture of calling
back former England batter Ian Bell following a disputable runout at Trent Bridge in 2011.

Q.46 [30465398]
Who won the ICC Spirit of Cricket Award of the Decade whose name has been redacted with [5] in the passage
above?

a MS Dhoni

b Daniel Vettori

https://www.aspiration.ai/LAW/sis/Solution.jsp?qsetId=JAmrn2p8H1A=&qsetName=LST Mock 27 2021 (CLAT) 79/223


1/17/2021 Mock Analysis

c Kane Williamson

d Brendon McCullum

Solution:
 Answer key/Solution
Correct Answer : a
Your Answer : a
MS Dhoni won the ICC Spirit of Cricket Award of the Decade, chosen by fans
unanimously for his gesture of calling back former England batter Ian Bell following a disputable runout at
Trent Bridge in 2011.

FeedBack Bookmark

Passage-04

One more wetland in India has been added to the list of recognised sites of international importance under the
treaty of Ramsar Convention, taking the number of such wetlands in the country to [1], the highest in South Asia.

The latest site to be added to the list is a high-altitude wetland complex of two connected lakes, Startsapuk Tso
and Tso Kar, in [2]. Union Environment [3] said, “Happy to share that high-altitude wetland complex in
Changthang region of [2] is recognised as wetland of international importance. The complex is a notable
example of two connected lakes, the freshwater Startsapuk Tso and the hypersaline Tso Kar. Now, India has [1]
Ramsar sites.”

Last month, two wetlands — the Lonar Lake in Maharashtra and Sur Sarovar, also known as Keetham Lake in [4]
— were added to the list of Ramsar sites. The Tso Kar Basin is a high-altitude wetland complex, consisting of
two principal waterbodies, Startsapuk Tso, a freshwater lake of about 438 hectares to the south, and Tso Kar
itself, a hyper-saline lake of 1,800 hectares to the north, situated in the Changthang region of [2].

It is called Tso Kar, meaning white lake, because of the white salt e orescence found on the margins due to the
evaporation of highly saline water, according to a statement issued by the ministry.

Q.47 [30465398]
As on 31 December 2020, how many Indian sites has been added to the list of recognised sites of international
importance under the treaty of Ramsar Convention redacted with [1] in the passage above?

a 32

b 36

c 39

d 42

https://www.aspiration.ai/LAW/sis/Solution.jsp?qsetId=JAmrn2p8H1A=&qsetName=LST Mock 27 2021 (CLAT) 80/223


1/17/2021 Mock Analysis

Solution:
 Answer key/Solution
Correct Answer : d
Your Answer : d
One more wetland in India has been added to the list of recognised sites of
international importance under the treaty of Ramsar Convention, taking the number of such wetlands in the
country to 42, the highest in South Asia.

FeedBack Bookmark

Passage-04

One more wetland in India has been added to the list of recognised sites of international importance under the
treaty of Ramsar Convention, taking the number of such wetlands in the country to [1], the highest in South Asia.

The latest site to be added to the list is a high-altitude wetland complex of two connected lakes, Startsapuk Tso
and Tso Kar, in [2]. Union Environment [3] said, “Happy to share that high-altitude wetland complex in
Changthang region of [2] is recognised as wetland of international importance. The complex is a notable
example of two connected lakes, the freshwater Startsapuk Tso and the hypersaline Tso Kar. Now, India has [1]
Ramsar sites.”

Last month, two wetlands — the Lonar Lake in Maharashtra and Sur Sarovar, also known as Keetham Lake in [4]
— were added to the list of Ramsar sites. The Tso Kar Basin is a high-altitude wetland complex, consisting of
two principal waterbodies, Startsapuk Tso, a freshwater lake of about 438 hectares to the south, and Tso Kar
itself, a hyper-saline lake of 1,800 hectares to the north, situated in the Changthang region of [2].

It is called Tso Kar, meaning white lake, because of the white salt e orescence found on the margins due to the
evaporation of highly saline water, according to a statement issued by the ministry.

Q.48 [30465398]
The latest site to be added to the list is a high-altitude wetland complex of two connected lakes, Startsapuk Tso
and Tso Kar, in [2]. Which of the following places has been removed with [2] in the passage above?

a Kohima

b Ladakh

c Shimla

d Aravali

https://www.aspiration.ai/LAW/sis/Solution.jsp?qsetId=JAmrn2p8H1A=&qsetName=LST Mock 27 2021 (CLAT) 81/223


1/17/2021 Mock Analysis

Solution:
 Answer key/Solution
Correct Answer : b
Your Answer : b
The latest site to be added to the list is a high-altitude wetland complex of two
connected lakes, Startsapuk Tso and Tso Kar, in Ladakh. The Tso Kar Basin is a high-altitude wetland
complex, consisting of two principal waterbodies, Startsapuk Tso, a freshwater lake of about 438 hectares to
the south, and Tso Kar itself, a hyper-saline lake of 1,800 hectares to the north, situated in the Changthang
region of Ladakh.

FeedBack Bookmark

Passage-04

One more wetland in India has been added to the list of recognised sites of international importance under the
treaty of Ramsar Convention, taking the number of such wetlands in the country to [1], the highest in South Asia.

The latest site to be added to the list is a high-altitude wetland complex of two connected lakes, Startsapuk Tso
and Tso Kar, in [2]. Union Environment [3] said, “Happy to share that high-altitude wetland complex in
Changthang region of [2] is recognised as wetland of international importance. The complex is a notable
example of two connected lakes, the freshwater Startsapuk Tso and the hypersaline Tso Kar. Now, India has [1]
Ramsar sites.”

Last month, two wetlands — the Lonar Lake in Maharashtra and Sur Sarovar, also known as Keetham Lake in [4]
— were added to the list of Ramsar sites. The Tso Kar Basin is a high-altitude wetland complex, consisting of
two principal waterbodies, Startsapuk Tso, a freshwater lake of about 438 hectares to the south, and Tso Kar
itself, a hyper-saline lake of 1,800 hectares to the north, situated in the Changthang region of [2].

It is called Tso Kar, meaning white lake, because of the white salt e orescence found on the margins due to the
evaporation of highly saline water, according to a statement issued by the ministry.

Q.49 [30465398]
Who is the current Union Minister of Environment, Forest and Climate Change whose name has been redacted
with [3] in the passage above?

a Prakash Javadekar

b Nitin Gadkari

c Piyush Goyal

d Arjun Ram Meghwal

https://www.aspiration.ai/LAW/sis/Solution.jsp?qsetId=JAmrn2p8H1A=&qsetName=LST Mock 27 2021 (CLAT) 82/223


1/17/2021 Mock Analysis

Solution:
 Answer key/Solution
Correct Answer : a
Your Answer : a
The Ministry of Environment, Forest and Climate Change (MoEFCC) is an Indian
government ministry. The ministry portfolio is currently held by Prakash Javadekar, Union Minister of
Environment, Forest and Climate Change.

FeedBack Bookmark

Passage-04

One more wetland in India has been added to the list of recognised sites of international importance under the
treaty of Ramsar Convention, taking the number of such wetlands in the country to [1], the highest in South Asia.

The latest site to be added to the list is a high-altitude wetland complex of two connected lakes, Startsapuk Tso
and Tso Kar, in [2]. Union Environment [3] said, “Happy to share that high-altitude wetland complex in
Changthang region of [2] is recognised as wetland of international importance. The complex is a notable
example of two connected lakes, the freshwater Startsapuk Tso and the hypersaline Tso Kar. Now, India has [1]
Ramsar sites.”

Last month, two wetlands — the Lonar Lake in Maharashtra and Sur Sarovar, also known as Keetham Lake in [4]
— were added to the list of Ramsar sites. The Tso Kar Basin is a high-altitude wetland complex, consisting of
two principal waterbodies, Startsapuk Tso, a freshwater lake of about 438 hectares to the south, and Tso Kar
itself, a hyper-saline lake of 1,800 hectares to the north, situated in the Changthang region of [2].

It is called Tso Kar, meaning white lake, because of the white salt e orescence found on the margins due to the
evaporation of highly saline water, according to a statement issued by the ministry.

Q.50 [30465398]
Sur Sarovar also known as Keetham Lake in [4] was added to the list of Ramsar sites in November 2020. Which
of the following places has been removed with [4] in the passage above?

a Chennai

b Patna

c Agra

d Mumbai

https://www.aspiration.ai/LAW/sis/Solution.jsp?qsetId=JAmrn2p8H1A=&qsetName=LST Mock 27 2021 (CLAT) 83/223


1/17/2021 Mock Analysis

Solution:
 Answer key/Solution
Correct Answer : c
Your Answer : c
The Lonar Lake in Maharashtra and Sur Sarovar, also known as Keetham Lake, in
Agra, have been added to the list of recognised Ramsar sites. The Ramsar Convention on Wetlands of
International Importance Especially as Waterfowl Habitat is a treaty for conservation and sustainable use of
such sites. It is named after Ramsar, the Iranian city where the treaty was signed in 1971, and places chosen
for conservation under it are given the tag ''Ramsar site''.

FeedBack Bookmark

Passage-04

One more wetland in India has been added to the list of recognised sites of international importance under the
treaty of Ramsar Convention, taking the number of such wetlands in the country to [1], the highest in South Asia.

The latest site to be added to the list is a high-altitude wetland complex of two connected lakes, Startsapuk Tso
and Tso Kar, in [2]. Union Environment [3] said, “Happy to share that high-altitude wetland complex in
Changthang region of [2] is recognised as wetland of international importance. The complex is a notable
example of two connected lakes, the freshwater Startsapuk Tso and the hypersaline Tso Kar. Now, India has [1]
Ramsar sites.”

Last month, two wetlands — the Lonar Lake in Maharashtra and Sur Sarovar, also known as Keetham Lake in [4]
— were added to the list of Ramsar sites. The Tso Kar Basin is a high-altitude wetland complex, consisting of
two principal waterbodies, Startsapuk Tso, a freshwater lake of about 438 hectares to the south, and Tso Kar
itself, a hyper-saline lake of 1,800 hectares to the north, situated in the Changthang region of [2].

It is called Tso Kar, meaning white lake, because of the white salt e orescence found on the margins due to the
evaporation of highly saline water, according to a statement issued by the ministry.

Q.51 [30465398]
Which of the following statements is not true regarding the Ramsar Convention on Wetlands of International
Importance?

a It is an intergovernmental treaty adopted in 1971 and came into force in 1975.

b It is named after the city of Ramsar in Iraq, where the convention was signed.

c It came into force for India on 1 February, 1982.

d At present, two wetlands of India are in Montreux Record, they are Keoladeo National Park in Rajasthan and
Loktak Lake in Manipur.

https://www.aspiration.ai/LAW/sis/Solution.jsp?qsetId=JAmrn2p8H1A=&qsetName=LST Mock 27 2021 (CLAT) 84/223


1/17/2021 Mock Analysis

Solution:
 Answer key/Solution
Correct Answer : b
Your Answer : b
The Ramsar Convention on Wetlands of International Importance Especially as
Waterfowl Habitat is an international treaty for the conservation and sustainable use of wetlands. It is also
known as the Convention on Wetlands. It is named after the city of Ramsar in Iran, where the convention was
signed in 1971 and came into force in 1975. Ramsar Convention on Wetlands is an intergovernmental treaty
adopted in 1971 in the Iranian city of Ramsar, on the southern shore of the Caspian Sea. It came into force for
India on 1st February, 1982. Those wetlands which are of international importance are declared as Ramsar
sites. At present, two wetlands of India are in Montreux Record, they are Keoladeo National Park in Rajasthan
and Loktak Lake in Manipur.

FeedBack Bookmark

Passage-05

The 1818 battle of Bhima-Koregaon, should be included in the history textbooks, said Union Minister of State for
Social Justice and Empowerment Ramdas Athawale.

“The history of the Bhima-Koregaon battle should be taught in schools. I will be writing a letter to the Education
Ministry in this regard,” said Mr. Athawale, who is also president of the Republican Party of India (A). Mr.
Athawale visited the battle’s victory obelisk or ranstambh at Perne near Koregaon-Bhima village and offered
tributes.

Bhima-Koregaon, a small village in Pune district of Maharashtra, has a rich Maratha history. Two hundred years
ago, on January 1, 1818, a few hundred [1] soldiers of the [2], led by the British, defeated the massive [3] army,
led by Bajirao II, in Koregaon.

This battle has, since, attained legendary stature in Dalit history. The Dalits who follow B.R. Ambedkar view this
battle as a victory of [1] over the injustice and torture meted out to them by the Brahminical [3]. Every year on
January 1 as the world stays drowned under the magic of New Year celebrations, it’s the Ambedkarite Dalits
who gather at Bhima Koregaon to pay their respect at the Vijay Sthamb (victory pillar). The pillar was erected by
the [2] in memory of those who fought the battle. The names of the [1] soldiers who unknowingly brought an end
to the [3] rule in 1818 are inscribed on the pillar.

2018 was the 200th year of the “victory of [1] over the Brahmanical [3]”. During the celebrations there were
violent clashes between Dalit and Maratha groups, resulting in the death of at least one person and injuries to
several others. Being the 200th anniversary, the gathering in Bhima Koregaon was much larger than usual. Many
Dalit and Bahujan groups collectively organised a public conference in the name of [4] at Shaniwar Wada, which
was the seat of the [3] until 1818.

Q.52 [30465398]
The names of the two communities involved in the Bhima-Koregaon battle have been replaced with [1] and [3] in
the given passage. What are their names?

a Jatvas and Pushpakas

https://www.aspiration.ai/LAW/sis/Solution.jsp?qsetId=JAmrn2p8H1A=&qsetName=LST Mock 27 2021 (CLAT) 85/223


1/17/2021 Mock Analysis

b Chamars and Pancha-Gaudas

c Lonias and Halenadus

d Mahars and Peshwas

Solution:
 Answer key/Solution
Correct Answer : d
Your Answer : d
Bhima-Koregaon, a small village in Pune district of Maharashtra, has a rich
Maratha history. Two hundred years ago, on January 1, 1818, a few hundred Mahar soldiers of the East India
Company, led by the British, defeated the massive Peshwa army, led by Peshwa Bajirao II, in Koregaon.

FeedBack Bookmark

Passage-05

The 1818 battle of Bhima-Koregaon, should be included in the history textbooks, said Union Minister of State for
Social Justice and Empowerment Ramdas Athawale.

“The history of the Bhima-Koregaon battle should be taught in schools. I will be writing a letter to the Education
Ministry in this regard,” said Mr. Athawale, who is also president of the Republican Party of India (A). Mr.
Athawale visited the battle’s victory obelisk or ranstambh at Perne near Koregaon-Bhima village and offered
tributes.

Bhima-Koregaon, a small village in Pune district of Maharashtra, has a rich Maratha history. Two hundred years
ago, on January 1, 1818, a few hundred [1] soldiers of the [2], led by the British, defeated the massive [3] army,
led by Bajirao II, in Koregaon.

This battle has, since, attained legendary stature in Dalit history. The Dalits who follow B.R. Ambedkar view this
battle as a victory of [1] over the injustice and torture meted out to them by the Brahminical [3]. Every year on
January 1 as the world stays drowned under the magic of New Year celebrations, it’s the Ambedkarite Dalits
who gather at Bhima Koregaon to pay their respect at the Vijay Sthamb (victory pillar). The pillar was erected by
the [2] in memory of those who fought the battle. The names of the [1] soldiers who unknowingly brought an end
to the [3] rule in 1818 are inscribed on the pillar.

2018 was the 200th year of the “victory of [1] over the Brahmanical [3]”. During the celebrations there were
violent clashes between Dalit and Maratha groups, resulting in the death of at least one person and injuries to
several others. Being the 200th anniversary, the gathering in Bhima Koregaon was much larger than usual. Many
Dalit and Bahujan groups collectively organised a public conference in the name of [4] at Shaniwar Wada, which
was the seat of the [3] until 1818.

https://www.aspiration.ai/LAW/sis/Solution.jsp?qsetId=JAmrn2p8H1A=&qsetName=LST Mock 27 2021 (CLAT) 86/223


1/17/2021 Mock Analysis

Q.53 [30465398]
On January 1, 1927, who among the following leaders started the ritual of holding a commemoration at the site
of Vijay Sthamb, one that is repeated every year?

a Bhimrao Ambedkar

b Jogendra Nath Mandal

c Mahatma Gandhi

d B. Shyam Sunder

Solution:
 Answer key/Solution
Correct Answer : a
Your Answer : a
On January 1, 1927, Bhimrao Ambedkar started the ritual of holding a
commemoration at the site of this pillar, one that is repeated every year.

FeedBack Bookmark

https://www.aspiration.ai/LAW/sis/Solution.jsp?qsetId=JAmrn2p8H1A=&qsetName=LST Mock 27 2021 (CLAT) 87/223


1/17/2021 Mock Analysis

Passage-05

The 1818 battle of Bhima-Koregaon, should be included in the history textbooks, said Union Minister of State for
Social Justice and Empowerment Ramdas Athawale.

“The history of the Bhima-Koregaon battle should be taught in schools. I will be writing a letter to the Education
Ministry in this regard,” said Mr. Athawale, who is also president of the Republican Party of India (A). Mr.
Athawale visited the battle’s victory obelisk or ranstambh at Perne near Koregaon-Bhima village and offered
tributes.

Bhima-Koregaon, a small village in Pune district of Maharashtra, has a rich Maratha history. Two hundred years
ago, on January 1, 1818, a few hundred [1] soldiers of the [2], led by the British, defeated the massive [3] army,
led by Bajirao II, in Koregaon.

This battle has, since, attained legendary stature in Dalit history. The Dalits who follow B.R. Ambedkar view this
battle as a victory of [1] over the injustice and torture meted out to them by the Brahminical [3]. Every year on
January 1 as the world stays drowned under the magic of New Year celebrations, it’s the Ambedkarite Dalits
who gather at Bhima Koregaon to pay their respect at the Vijay Sthamb (victory pillar). The pillar was erected by
the [2] in memory of those who fought the battle. The names of the [1] soldiers who unknowingly brought an end
to the [3] rule in 1818 are inscribed on the pillar.

2018 was the 200th year of the “victory of [1] over the Brahmanical [3]”. During the celebrations there were
violent clashes between Dalit and Maratha groups, resulting in the death of at least one person and injuries to
several others. Being the 200th anniversary, the gathering in Bhima Koregaon was much larger than usual. Many
Dalit and Bahujan groups collectively organised a public conference in the name of [4] at Shaniwar Wada, which
was the seat of the [3] until 1818.

Q.54 [30465398]
Which of the following has been replaced with ‘[2]’ in the given passage?

a Maratha Army

b East India Company

c Kingdom of Mysore

d Mughal Army

Solution:
 Answer key/Solution
Correct Answer : b
Your Answer : b
Bhima-Koregaon, a small village in Pune district of Maharashtra, has a rich
Maratha history. Two hundred years ago, on January 1, 1818, a few hundred Mahar soldiers of the East India
Company, led by the British, defeated the massive Peshwa army, led by Peshwa Bajirao II, in Koregaon.

FeedBack Bookmark

https://www.aspiration.ai/LAW/sis/Solution.jsp?qsetId=JAmrn2p8H1A=&qsetName=LST Mock 27 2021 (CLAT) 88/223


1/17/2021 Mock Analysis

Passage-05

The 1818 battle of Bhima-Koregaon, should be included in the history textbooks, said Union Minister of State for
Social Justice and Empowerment Ramdas Athawale.

“The history of the Bhima-Koregaon battle should be taught in schools. I will be writing a letter to the Education
Ministry in this regard,” said Mr. Athawale, who is also president of the Republican Party of India (A). Mr.
Athawale visited the battle’s victory obelisk or ranstambh at Perne near Koregaon-Bhima village and offered
tributes.

Bhima-Koregaon, a small village in Pune district of Maharashtra, has a rich Maratha history. Two hundred years
ago, on January 1, 1818, a few hundred [1] soldiers of the [2], led by the British, defeated the massive [3] army,
led by Bajirao II, in Koregaon.

This battle has, since, attained legendary stature in Dalit history. The Dalits who follow B.R. Ambedkar view this
battle as a victory of [1] over the injustice and torture meted out to them by the Brahminical [3]. Every year on
January 1 as the world stays drowned under the magic of New Year celebrations, it’s the Ambedkarite Dalits
who gather at Bhima Koregaon to pay their respect at the Vijay Sthamb (victory pillar). The pillar was erected by
the [2] in memory of those who fought the battle. The names of the [1] soldiers who unknowingly brought an end
to the [3] rule in 1818 are inscribed on the pillar.

2018 was the 200th year of the “victory of [1] over the Brahmanical [3]”. During the celebrations there were
violent clashes between Dalit and Maratha groups, resulting in the death of at least one person and injuries to
several others. Being the 200th anniversary, the gathering in Bhima Koregaon was much larger than usual. Many
Dalit and Bahujan groups collectively organised a public conference in the name of [4] at Shaniwar Wada, which
was the seat of the [3] until 1818.

Q.55 [30465398]
The Scheduled Castes and Tribes (Prevention of Atrocities) Act, 1989 is an Act of the Parliament of India
enacted to prevent atrocities against scheduled castes and scheduled tribes. The act came into force on:

a 31 March 1989

b 31 May 1990

c 1 April 1990

d 31 March 1995

https://www.aspiration.ai/LAW/sis/Solution.jsp?qsetId=JAmrn2p8H1A=&qsetName=LST Mock 27 2021 (CLAT) 89/223


1/17/2021 Mock Analysis

Solution:
 Answer key/Solution
Correct Answer : d
The Scheduled Castes and Tribes (Prevention of Atrocities) Act, 1989 is an Act of
the Parliament of India enacted to prevent atrocities against scheduled castes
and scheduled tribes. The Act is popularly known as the SC/ST Act, POA, the Prevention of Atrocities Act, or
simply the Atrocities Act. It was enacted when the provisions of the existing laws (such as the Protection of
Civil Rights Act 1955 and Indian Penal Code) were found to be inadequate to check these crimes (de ned as
'atrocities' in the Act). Recognising the continuing gross indignities and offences against Scheduled Castes
and Tribes, the Parliament passed the ‘Scheduled Castes and Schedule Tribes (Prevention of Atrocities) Act
1989. The rules for the Act were noti ed on 31 March 1995.

FeedBack Bookmark

Passage-05

The 1818 battle of Bhima-Koregaon, should be included in the history textbooks, said Union Minister of State for
Social Justice and Empowerment Ramdas Athawale.

“The history of the Bhima-Koregaon battle should be taught in schools. I will be writing a letter to the Education
Ministry in this regard,” said Mr. Athawale, who is also president of the Republican Party of India (A). Mr.
Athawale visited the battle’s victory obelisk or ranstambh at Perne near Koregaon-Bhima village and offered
tributes.

Bhima-Koregaon, a small village in Pune district of Maharashtra, has a rich Maratha history. Two hundred years
ago, on January 1, 1818, a few hundred [1] soldiers of the [2], led by the British, defeated the massive [3] army,
led by Bajirao II, in Koregaon.

This battle has, since, attained legendary stature in Dalit history. The Dalits who follow B.R. Ambedkar view this
battle as a victory of [1] over the injustice and torture meted out to them by the Brahminical [3]. Every year on
January 1 as the world stays drowned under the magic of New Year celebrations, it’s the Ambedkarite Dalits
who gather at Bhima Koregaon to pay their respect at the Vijay Sthamb (victory pillar). The pillar was erected by
the [2] in memory of those who fought the battle. The names of the [1] soldiers who unknowingly brought an end
to the [3] rule in 1818 are inscribed on the pillar.

2018 was the 200th year of the “victory of [1] over the Brahmanical [3]”. During the celebrations there were
violent clashes between Dalit and Maratha groups, resulting in the death of at least one person and injuries to
several others. Being the 200th anniversary, the gathering in Bhima Koregaon was much larger than usual. Many
Dalit and Bahujan groups collectively organised a public conference in the name of [4] at Shaniwar Wada, which
was the seat of the [3] until 1818.

Q.56 [30465398]
What is the name of the public conference organised by Dalit and Bahujan groups whose name has been
replaced with [4] in the given passage?

a Dalit Sangh

https://www.aspiration.ai/LAW/sis/Solution.jsp?qsetId=JAmrn2p8H1A=&qsetName=LST Mock 27 2021 (CLAT) 90/223


1/17/2021 Mock Analysis

b Dalit Panthers

c Elgar Parishad

d Ranvir Sena

Solution:
 Answer key/Solution
Correct Answer : c
Being the 200th anniversary, the gathering in Bhima Koregaon last year was much
larger than usual. Many Dalit and Bahujan groups collectively organised a public
conference in the name of Elgar Parishad at Shaniwar Wada, which was the seat of the Peshwas until 1818.
FeedBack Bookmark

https://www.aspiration.ai/LAW/sis/Solution.jsp?qsetId=JAmrn2p8H1A=&qsetName=LST Mock 27 2021 (CLAT) 91/223


1/17/2021 Mock Analysis

Passage-06

Chandrayaan-3’s launch has been delayed due to the Covid-19 pandemic and it will be launched early next year
instead of the second half of 2020. The announcement was made by Union Minister Jitendra Singh where he
revealed that the mission to Moon will not include an [1] like the Chandrayaan-2. But, it will include a rover and a
lander.

Chandrayaan-2’s [2] Lander crashed 2019 and its debris was found later on by NASA. Following the
unsuccessful landing on Moon’s South Pole, the Indian Space Research Organisation (ISRO) had said that it has
plans to launch another mission to the Moon in 2020.

Recently, data from the ISRO suggested that Moon might be rusting. In the study, it is mentioned that the moon
is turning slightly red, which indicates that there is a formation of a reddish-black mineral form of iron named
hematite on its surface, especially at the poles.

Jitendra Singh, Minister of State (MoS) for the Department of Space said Chandrayaan-1 which was launched in
2008 has transmitted images suggesting Moon may be rusting.

“The sign of this nding is that even though the surface of the Moon is known to have iron-rich rocks, it is not
known for the presence of water and oxygen, which are the two elements needed to interact with iron to create
rust,” Singh said.

On the other hand, NASA’s JPL scientists Abigail Fraeman and Vivian Sun were surprised by the ndings of
Chandrayaan-1 “very puzzling”. They added: “At rst, I totally didn’t believe it. It shouldn’t exist based on the
conditions present on the Moon,” Fraeman said. “But since we discovered water on the Moon, people have been
speculating that there could be a greater variety of minerals than we realize if that water had reacted with
rocks.”

Also, the training processes and other procedures for India’s rst-ever Human Space mission [3] despite the
situation created by the pandemic.

Singh said, “Constraints due to the COVID pandemic led to some disruptions in the plan for [3] but efforts are
going on to stick to the timeline of around 2022.”

Q.57 [30465398]
Chandrayaan-3 mission to Moon will not include an [1] like the Chandrayaan-2. Which of the following has been
removed with [1] in the passage above?

a Orbiter

b Arti cial Robot

c Astronomer

d All of the above

https://www.aspiration.ai/LAW/sis/Solution.jsp?qsetId=JAmrn2p8H1A=&qsetName=LST Mock 27 2021 (CLAT) 92/223


1/17/2021 Mock Analysis

Solution:
 Answer key/Solution
Correct Answer : a
Your Answer : a
Chandrayaan-3’s launch has been delayed due to the Covid-19 pandemic and it
will be launched early next year instead of the second half of 2020. The announcement was made by Union
Minister Jitendra Singh where he revealed that the mission to Moon will not include an orbiter like the
Chandrayaan-2. But, it will include a rover and a lander.

FeedBack Bookmark

Passage-06

Chandrayaan-3’s launch has been delayed due to the Covid-19 pandemic and it will be launched early next year
instead of the second half of 2020. The announcement was made by Union Minister Jitendra Singh where he
revealed that the mission to Moon will not include an [1] like the Chandrayaan-2. But, it will include a rover and a
lander.

Chandrayaan-2’s [2] Lander crashed 2019 and its debris was found later on by NASA. Following the
unsuccessful landing on Moon’s South Pole, the Indian Space Research Organisation (ISRO) had said that it has
plans to launch another mission to the Moon in 2020.

Recently, data from the ISRO suggested that Moon might be rusting. In the study, it is mentioned that the moon
is turning slightly red, which indicates that there is a formation of a reddish-black mineral form of iron named
hematite on its surface, especially at the poles.

Jitendra Singh, Minister of State (MoS) for the Department of Space said Chandrayaan-1 which was launched in
2008 has transmitted images suggesting Moon may be rusting.

“The sign of this nding is that even though the surface of the Moon is known to have iron-rich rocks, it is not
known for the presence of water and oxygen, which are the two elements needed to interact with iron to create
rust,” Singh said.

On the other hand, NASA’s JPL scientists Abigail Fraeman and Vivian Sun were surprised by the ndings of
Chandrayaan-1 “very puzzling”. They added: “At rst, I totally didn’t believe it. It shouldn’t exist based on the
conditions present on the Moon,” Fraeman said. “But since we discovered water on the Moon, people have been
speculating that there could be a greater variety of minerals than we realize if that water had reacted with
rocks.”

Also, the training processes and other procedures for India’s rst-ever Human Space mission [3] despite the
situation created by the pandemic.

Singh said, “Constraints due to the COVID pandemic led to some disruptions in the plan for [3] but efforts are
going on to stick to the timeline of around 2022.”

Q.58 [30465398]
What is the name of Chandrayaan-2 lander whose name has been redacted with [2] in the passage above?

https://www.aspiration.ai/LAW/sis/Solution.jsp?qsetId=JAmrn2p8H1A=&qsetName=LST Mock 27 2021 (CLAT) 93/223


1/17/2021 Mock Analysis

a Kalam

b Satish

c Vikram

d Pragyan

Solution:
 Answer key/Solution
Correct Answer : c
Your Answer : c
Chandrayaan-2’s Vikram Lander crashed 2019 and its debris was found later on
by NASA. Pragyan was the rover of Chandrayaan-2, a lunar mission developed by the Indian Space Research
Organisation (ISRO), that launched in July 2019. Pragyan was destroyed along with its lander, Vikram, when it
crash-landed on the Moon in September 2019 and never got the chance to deploy.

FeedBack Bookmark

https://www.aspiration.ai/LAW/sis/Solution.jsp?qsetId=JAmrn2p8H1A=&qsetName=LST Mock 27 2021 (CLAT) 94/223


1/17/2021 Mock Analysis

Passage-06

Chandrayaan-3’s launch has been delayed due to the Covid-19 pandemic and it will be launched early next year
instead of the second half of 2020. The announcement was made by Union Minister Jitendra Singh where he
revealed that the mission to Moon will not include an [1] like the Chandrayaan-2. But, it will include a rover and a
lander.

Chandrayaan-2’s [2] Lander crashed 2019 and its debris was found later on by NASA. Following the
unsuccessful landing on Moon’s South Pole, the Indian Space Research Organisation (ISRO) had said that it has
plans to launch another mission to the Moon in 2020.

Recently, data from the ISRO suggested that Moon might be rusting. In the study, it is mentioned that the moon
is turning slightly red, which indicates that there is a formation of a reddish-black mineral form of iron named
hematite on its surface, especially at the poles.

Jitendra Singh, Minister of State (MoS) for the Department of Space said Chandrayaan-1 which was launched in
2008 has transmitted images suggesting Moon may be rusting.

“The sign of this nding is that even though the surface of the Moon is known to have iron-rich rocks, it is not
known for the presence of water and oxygen, which are the two elements needed to interact with iron to create
rust,” Singh said.

On the other hand, NASA’s JPL scientists Abigail Fraeman and Vivian Sun were surprised by the ndings of
Chandrayaan-1 “very puzzling”. They added: “At rst, I totally didn’t believe it. It shouldn’t exist based on the
conditions present on the Moon,” Fraeman said. “But since we discovered water on the Moon, people have been
speculating that there could be a greater variety of minerals than we realize if that water had reacted with
rocks.”

Also, the training processes and other procedures for India’s rst-ever Human Space mission [3] despite the
situation created by the pandemic.

Singh said, “Constraints due to the COVID pandemic led to some disruptions in the plan for [3] but efforts are
going on to stick to the timeline of around 2022.”

Q.59 [30465398]
What is the name of the India’s rst-ever Human Space mission whose name has been redacted with [3] in the
passage above?

a Spaceyaan

b Indiayaan

c Gaganyaan

d Bharatyaan

https://www.aspiration.ai/LAW/sis/Solution.jsp?qsetId=JAmrn2p8H1A=&qsetName=LST Mock 27 2021 (CLAT) 95/223


1/17/2021 Mock Analysis

Solution:
 Answer key/Solution
Correct Answer : c
Your Answer : c
Jitendra Singh, Minister of State (MoS) for the Department of Space said that the
training processes and other procedures for India’s rst-ever Human Space mission Gaganyaan despite the
situation created by the pandemic. Singh said, “Constraints due to the COVID pandemic led to some
disruptions in the plan for Gaganyaan but efforts are going on to stick to the timeline of around 2022.”

FeedBack Bookmark

Passage-06

Chandrayaan-3’s launch has been delayed due to the Covid-19 pandemic and it will be launched early next year
instead of the second half of 2020. The announcement was made by Union Minister Jitendra Singh where he
revealed that the mission to Moon will not include an [1] like the Chandrayaan-2. But, it will include a rover and a
lander.

Chandrayaan-2’s [2] Lander crashed 2019 and its debris was found later on by NASA. Following the
unsuccessful landing on Moon’s South Pole, the Indian Space Research Organisation (ISRO) had said that it has
plans to launch another mission to the Moon in 2020.

Recently, data from the ISRO suggested that Moon might be rusting. In the study, it is mentioned that the moon
is turning slightly red, which indicates that there is a formation of a reddish-black mineral form of iron named
hematite on its surface, especially at the poles.

Jitendra Singh, Minister of State (MoS) for the Department of Space said Chandrayaan-1 which was launched in
2008 has transmitted images suggesting Moon may be rusting.

“The sign of this nding is that even though the surface of the Moon is known to have iron-rich rocks, it is not
known for the presence of water and oxygen, which are the two elements needed to interact with iron to create
rust,” Singh said.

On the other hand, NASA’s JPL scientists Abigail Fraeman and Vivian Sun were surprised by the ndings of
Chandrayaan-1 “very puzzling”. They added: “At rst, I totally didn’t believe it. It shouldn’t exist based on the
conditions present on the Moon,” Fraeman said. “But since we discovered water on the Moon, people have been
speculating that there could be a greater variety of minerals than we realize if that water had reacted with
rocks.”

Also, the training processes and other procedures for India’s rst-ever Human Space mission [3] despite the
situation created by the pandemic.

Singh said, “Constraints due to the COVID pandemic led to some disruptions in the plan for [3] but efforts are
going on to stick to the timeline of around 2022.”

https://www.aspiration.ai/LAW/sis/Solution.jsp?qsetId=JAmrn2p8H1A=&qsetName=LST Mock 27 2021 (CLAT) 96/223


1/17/2021 Mock Analysis

Q.60 [30465398]
In December 2020, which of the following countries has become the second country in the world to plant ag on
the Moon?

a USA

b Israel

c India

d China

Solution:
 Answer key/Solution
Correct Answer : d
Your Answer : d
In December 2020, China has become the second country in the world to plant
ag on the Moon. China has planted its ag on the Moon, more than 50 years after the US rst planted the
Stars and Stripes there. The pictures from China's National Space Administration show the ve-starred Red
Flag holding still on the windless lunar surface. The US planted the rst ag on the Moon during the manned
Apollo 11 mission in 1969. Five further US ags were planted on the lunar surface during subsequent
missions up until 1972.
FeedBack Bookmark

https://www.aspiration.ai/LAW/sis/Solution.jsp?qsetId=JAmrn2p8H1A=&qsetName=LST Mock 27 2021 (CLAT) 97/223


1/17/2021 Mock Analysis

Passage-06

Chandrayaan-3’s launch has been delayed due to the Covid-19 pandemic and it will be launched early next year
instead of the second half of 2020. The announcement was made by Union Minister Jitendra Singh where he
revealed that the mission to Moon will not include an [1] like the Chandrayaan-2. But, it will include a rover and a
lander.

Chandrayaan-2’s [2] Lander crashed 2019 and its debris was found later on by NASA. Following the
unsuccessful landing on Moon’s South Pole, the Indian Space Research Organisation (ISRO) had said that it has
plans to launch another mission to the Moon in 2020.

Recently, data from the ISRO suggested that Moon might be rusting. In the study, it is mentioned that the moon
is turning slightly red, which indicates that there is a formation of a reddish-black mineral form of iron named
hematite on its surface, especially at the poles.

Jitendra Singh, Minister of State (MoS) for the Department of Space said Chandrayaan-1 which was launched in
2008 has transmitted images suggesting Moon may be rusting.

“The sign of this nding is that even though the surface of the Moon is known to have iron-rich rocks, it is not
known for the presence of water and oxygen, which are the two elements needed to interact with iron to create
rust,” Singh said.

On the other hand, NASA’s JPL scientists Abigail Fraeman and Vivian Sun were surprised by the ndings of
Chandrayaan-1 “very puzzling”. They added: “At rst, I totally didn’t believe it. It shouldn’t exist based on the
conditions present on the Moon,” Fraeman said. “But since we discovered water on the Moon, people have been
speculating that there could be a greater variety of minerals than we realize if that water had reacted with
rocks.”

Also, the training processes and other procedures for India’s rst-ever Human Space mission [3] despite the
situation created by the pandemic.

Singh said, “Constraints due to the COVID pandemic led to some disruptions in the plan for [3] but efforts are
going on to stick to the timeline of around 2022.”

Q.61 [30465398]
Why the south side of the moon is so important and not properly explored?

a Due to the moon’s axis, few regions on the South Pole remains forever dark especially the craters and have
higher chances of containing water.

b Because of the axis ‘weak angular tilt, the bottom of the polar craters of stay under shadows forever.

c The sunlight strikes in the Polar Regions at very low angles and thus the craters might never have received
sunlight, thus increasing the chances of ice being present on such surfaces.

d All of the above

https://www.aspiration.ai/LAW/sis/Solution.jsp?qsetId=JAmrn2p8H1A=&qsetName=LST Mock 27 2021 (CLAT) 98/223


1/17/2021 Mock Analysis

Solution:
 Answer key/Solution
Correct Answer : d
Due to the moon’s axis, few regions on the South Pole remains forever dark
especially the craters and have higher chances of containing water. Because of
the axis ‘weak angular tilt (1.54-degree tilt relative to Earth’s 23.5 degrees), the bottom of the polar craters of
stay under shadows forever. Hence the temperature remains frigid at the poles, hitting as low as -248 degrees
Celsius. That makes it among the lowest Solar System temperatures. The sunlight strikes in the polar regions
at very low angles and thus the craters might never have received sunlight, thus increasing the chances of ice
being present on such surfaces.

FeedBack Bookmark

Passage-07

By the year 2025, India will once again be part of the ve biggest economies in the world, pushing the UK to the
sixth place once again, a report by a UK-based think tank has predicted. At present, India is the sixth biggest
economy. Last year, it entered the club of top ve economies by pushing the UK to the sixth spot. The report
also forecast that by 2030, India will become the [1] biggest economy in the world.

"India has been knocked off course somewhat through the impact of the coronavirus pandemic. As a result,
after overtaking the UK in 2019, the UK overtakes India again in this year's forecasts and stays ahead till 2024
before India takes over again," the Centre for Economics and Business Research (CEBR) said in its annual report
published.

The report said the UK appears to have overtaken India and raced to the fth spot in 2020 on account of the
weakness of the rupee. Meanwhile, when it comes to India's short-term progress, the CEBR forecasts that the
Indian economy will expand by 9 per cent in 2021 and by 7 per cent in 2022.

"Growth will naturally slow as India becomes more economically developed, with the annual GDP growth
expected to sink to 5.8 per cent in 2035," it said. In a report about the prediction, news agency PTI said CEBR
forecast that this growth trajectory will see India become the world's [1] largest economy by 2030. It will
overtake the UK in 2025, Germany in 2027 and Japan in 2030.

"The UK-based think tank forecast that [2] will in 2028 overtake the US to become the world's biggest economy,
ve years earlier than previously estimated due to the contrasting recoveries of the two countries from the
Covid-19 pandemic," PTI said in a report.

Q.62 [30465398]
According to the latest report by the Centre for Economics and Business Research (CEBR), India will become the
[1] biggest economy in the world. Which of the following has been redacted with [1] in the passage above?

a Second

b Third

c Fourth

https://www.aspiration.ai/LAW/sis/Solution.jsp?qsetId=JAmrn2p8H1A=&qsetName=LST Mock 27 2021 (CLAT) 99/223


1/17/2021 Mock Analysis

d Fifth

Solution:
 Answer key/Solution
Correct Answer : b
Your Answer : d
By the year 2025, India will once again be part of the ve biggest economies in
the world, pushing the UK to the sixth place once again, a report by a UK-based think tank has predicted. At
present, India is the sixth biggest economy. Last year, it entered the club of top ve economies by pushing the
UK to the sixth spot. The report also forecast that by 2030, India will become the third biggest economy in the
world.
FeedBack Bookmark

Passage-07

By the year 2025, India will once again be part of the ve biggest economies in the world, pushing the UK to the
sixth place once again, a report by a UK-based think tank has predicted. At present, India is the sixth biggest
economy. Last year, it entered the club of top ve economies by pushing the UK to the sixth spot. The report
also forecast that by 2030, India will become the [1] biggest economy in the world.

"India has been knocked off course somewhat through the impact of the coronavirus pandemic. As a result,
after overtaking the UK in 2019, the UK overtakes India again in this year's forecasts and stays ahead till 2024
before India takes over again," the Centre for Economics and Business Research (CEBR) said in its annual report
published.

The report said the UK appears to have overtaken India and raced to the fth spot in 2020 on account of the
weakness of the rupee. Meanwhile, when it comes to India's short-term progress, the CEBR forecasts that the
Indian economy will expand by 9 per cent in 2021 and by 7 per cent in 2022.

"Growth will naturally slow as India becomes more economically developed, with the annual GDP growth
expected to sink to 5.8 per cent in 2035," it said. In a report about the prediction, news agency PTI said CEBR
forecast that this growth trajectory will see India become the world's [1] largest economy by 2030. It will
overtake the UK in 2025, Germany in 2027 and Japan in 2030.

"The UK-based think tank forecast that [2] will in 2028 overtake the US to become the world's biggest economy,
ve years earlier than previously estimated due to the contrasting recoveries of the two countries from the
Covid-19 pandemic," PTI said in a report.

Q.63 [30465398]
According to the latest report by the Centre for Economics and Business Research (CEBR), [2] will in 2028
overtake the US to become the world's biggest economy. Which of the following countries has been redacted
with [2] in the passage above?

a India

https://www.aspiration.ai/LAW/sis/Solution.jsp?qsetId=JAmrn2p8H1A=&qsetName=LST Mock 27 2021 (CLAT) 100/223


1/17/2021 Mock Analysis

b China

c France

d Japan

Solution:
 Answer key/Solution
Correct Answer : b
Your Answer : b
"The UK-based think tank forecast that China will in 2028 overtake the US to
become the world's biggest economy, ve years earlier than previously estimated due to the contrasting
recoveries of the two countries from the Covid-19 pandemic," PTI said in a report.
FeedBack Bookmark

Passage-07

By the year 2025, India will once again be part of the ve biggest economies in the world, pushing the UK to the
sixth place once again, a report by a UK-based think tank has predicted. At present, India is the sixth biggest
economy. Last year, it entered the club of top ve economies by pushing the UK to the sixth spot. The report
also forecast that by 2030, India will become the [1] biggest economy in the world.

"India has been knocked off course somewhat through the impact of the coronavirus pandemic. As a result,
after overtaking the UK in 2019, the UK overtakes India again in this year's forecasts and stays ahead till 2024
before India takes over again," the Centre for Economics and Business Research (CEBR) said in its annual report
published.

The report said the UK appears to have overtaken India and raced to the fth spot in 2020 on account of the
weakness of the rupee. Meanwhile, when it comes to India's short-term progress, the CEBR forecasts that the
Indian economy will expand by 9 per cent in 2021 and by 7 per cent in 2022.

"Growth will naturally slow as India becomes more economically developed, with the annual GDP growth
expected to sink to 5.8 per cent in 2035," it said. In a report about the prediction, news agency PTI said CEBR
forecast that this growth trajectory will see India become the world's [1] largest economy by 2030. It will
overtake the UK in 2025, Germany in 2027 and Japan in 2030.

"The UK-based think tank forecast that [2] will in 2028 overtake the US to become the world's biggest economy,
ve years earlier than previously estimated due to the contrasting recoveries of the two countries from the
Covid-19 pandemic," PTI said in a report.

Q.64 [30465398]
Global ratings agency Moody's in its latest report has revised India's 2020 GDP target and expects the
contraction of 8.9% as compared to 9.6% earlier. India's economy had the biggest contraction, 24% year-over-
year in the second quarter, as a result of___________.

https://www.aspiration.ai/LAW/sis/Solution.jsp?qsetId=JAmrn2p8H1A=&qsetName=LST Mock 27 2021 (CLAT) 101/223


1/17/2021 Mock Analysis

a Long protest by the farmers against the latest farm acts

b Long and strict nationwide lockdown

c Long protest against the Citizenship Amendment Act 2019

d All of the above

Solution:
 Answer key/Solution
Correct Answer : b
Global ratings agency Moody's in its latest report has said that it sees "very
gradual improvement in economic activity" of emerging market countries like
India, Argentina, Brazil, Mexico, Indonesia, Turkey and South Africa. The ratings agency has revised India's
2020 GDP target and expects the contraction of 8.9% as compared to 9.6% earlier. Moody's has said the scope
for additional rate cuts is limited in most emerging market countries, including India, Brazil, Mexico, and
Indonesia. India's economy had the biggest contraction, 24% year-over-year in the second quarter, as a result
of a long and strict nationwide lockdown, it said.
FeedBack Bookmark

Passage-07

By the year 2025, India will once again be part of the ve biggest economies in the world, pushing the UK to the
sixth place once again, a report by a UK-based think tank has predicted. At present, India is the sixth biggest
economy. Last year, it entered the club of top ve economies by pushing the UK to the sixth spot. The report
also forecast that by 2030, India will become the [1] biggest economy in the world.

"India has been knocked off course somewhat through the impact of the coronavirus pandemic. As a result,
after overtaking the UK in 2019, the UK overtakes India again in this year's forecasts and stays ahead till 2024
before India takes over again," the Centre for Economics and Business Research (CEBR) said in its annual report
published.

The report said the UK appears to have overtaken India and raced to the fth spot in 2020 on account of the
weakness of the rupee. Meanwhile, when it comes to India's short-term progress, the CEBR forecasts that the
Indian economy will expand by 9 per cent in 2021 and by 7 per cent in 2022.

"Growth will naturally slow as India becomes more economically developed, with the annual GDP growth
expected to sink to 5.8 per cent in 2035," it said. In a report about the prediction, news agency PTI said CEBR
forecast that this growth trajectory will see India become the world's [1] largest economy by 2030. It will
overtake the UK in 2025, Germany in 2027 and Japan in 2030.

"The UK-based think tank forecast that [2] will in 2028 overtake the US to become the world's biggest economy,
ve years earlier than previously estimated due to the contrasting recoveries of the two countries from the
Covid-19 pandemic," PTI said in a report.

https://www.aspiration.ai/LAW/sis/Solution.jsp?qsetId=JAmrn2p8H1A=&qsetName=LST Mock 27 2021 (CLAT) 102/223


1/17/2021 Mock Analysis

Q.65 [30465398]
In terms of purchasing power parity (PPP), India is the world's ___________ largest economy.

a Second

b Third

c Fourth

d Fifth

Solution:
 Answer key/Solution
Correct Answer : b
Your Answer : b
The economy of India is characterised as a middle income developing market
economy. It is the world's fth-largest economy by nominal GDP and the third-largest by purchasing power
parity (PPP). According to the International Monetary Fund (IMF), on a per capita income basis, India ranked
142nd by GDP (nominal) and 124th by GDP (PPP) in 2020. Historically, India was the largest economy in the
world for most of the two millennia from the 1st until 19th century.

FeedBack Bookmark

https://www.aspiration.ai/LAW/sis/Solution.jsp?qsetId=JAmrn2p8H1A=&qsetName=LST Mock 27 2021 (CLAT) 103/223


1/17/2021 Mock Analysis

Passage-07

By the year 2025, India will once again be part of the ve biggest economies in the world, pushing the UK to the
sixth place once again, a report by a UK-based think tank has predicted. At present, India is the sixth biggest
economy. Last year, it entered the club of top ve economies by pushing the UK to the sixth spot. The report
also forecast that by 2030, India will become the [1] biggest economy in the world.

"India has been knocked off course somewhat through the impact of the coronavirus pandemic. As a result,
after overtaking the UK in 2019, the UK overtakes India again in this year's forecasts and stays ahead till 2024
before India takes over again," the Centre for Economics and Business Research (CEBR) said in its annual report
published.

The report said the UK appears to have overtaken India and raced to the fth spot in 2020 on account of the
weakness of the rupee. Meanwhile, when it comes to India's short-term progress, the CEBR forecasts that the
Indian economy will expand by 9 per cent in 2021 and by 7 per cent in 2022.

"Growth will naturally slow as India becomes more economically developed, with the annual GDP growth
expected to sink to 5.8 per cent in 2035," it said. In a report about the prediction, news agency PTI said CEBR
forecast that this growth trajectory will see India become the world's [1] largest economy by 2030. It will
overtake the UK in 2025, Germany in 2027 and Japan in 2030.

"The UK-based think tank forecast that [2] will in 2028 overtake the US to become the world's biggest economy,
ve years earlier than previously estimated due to the contrasting recoveries of the two countries from the
Covid-19 pandemic," PTI said in a report.

Q.66 [30465398]
In terms of geographical area, the largest country in the world is

a USA

b Canada

c Russia

d China

Solution:
 Answer key/Solution
Correct Answer : c
Your Answer : c
The largest country in the world is Russia with a total area of 17,098,242 Km²
(6,601,665 mi²) and a land area of 16,376,870 Km² (6,323,142 mi²), equivalent to 11% of the total world's
landmass of 148,940,000 Km² (57,510,000 square miles). It is the only country in the world to spread in two
continents i.e. Asia and Europe. India is the seventh largest country in the world.
FeedBack Bookmark

https://www.aspiration.ai/LAW/sis/Solution.jsp?qsetId=JAmrn2p8H1A=&qsetName=LST Mock 27 2021 (CLAT) 104/223


1/17/2021 Mock Analysis

Sec 3
Directions for questions 67 to 105: You have been given some passages followed by questions based on each
passage. You are required to choose the most appropriate option which follows from the passage. Only the
information given in the passage should be used for choosing the answer and no external knowledge of law
howsoever prominent is to be applied.

Passage – 1

The concept of 'bailment' is dealt with in chapter IX of the Indian Contract Act, 1872. 'Bailment' is a technical
common-law term that encompasses change or transfer of possession of goods, upon a contract to return or
dispose of the goods in accordance with the direction of the person's delivering them, once the purpose for
such transfer is completed. The core vital feature of bailment is the change of possession, either physical or
constructive. The Indian Contract Act, 1972, under section 148 de nes the Contract of the Bailment, and in
successive sections deals with certain rights, duties and obligations imposed onto the bailor and bailee, in order
to safeguard their interest.

Essentials of contract of Bailment-


1. The existence of a valid contract- The existence of a valid contract is a foremost condition in bailment which
implies that goods are to be returned when the purpose is ful lled. Finder of lost goods is also known as bailee
although there may not be any existing contract between him and the actual owner.
2. Temporary delivery of goods- The whole concept of bailment revolves around the fact that the goods are
delivered for a temporary period and bailee cannot have permanent possession. Delivery of goods can be done
through actual delivery or through constructive delivery which means doing something which has the effect of
putting the goods in possession of bailee or any other person authorized by him.

3. Return of speci c goods- The bailee is bound to return the goods to the bailor after the purpose for which it
was taken is over. If the person is not returning the goods then it will not be bailment.

Duties and liabilities of a bailor-


1. The bailor is bound to disclose to the bailee, the faults in the goods bailed. which he knows and if he does
not make such disclosures, he is directly responsible for damage arising to the bailee directly from such faults.
2. Bailor is also responsible to the bailee for any loss which the bailee may sustain by reason of the fact that
bailor was not
• To make the bailment
• To receive the goods
• To give directions respecting them
3. Duty to indemnify the bailee- The bailor is duty bound to make good the loss suffered by the bailee where he
was compelled to return the goods before the expiry of the period of bailment
4. Duty to claim back the goods- The bailor is bound to accept the goods upon being returned by the bailee in
accordance with the terms of the agreement. If he refuses to accept it at a proper time, without any reasonable
ground then he will be liable for any loss which may happen to the goods.

Bailee's liabilities -
The bailee is bound to take as much care of goods bailed to him as a man of ordinary prudence would have
under similar circumstances and therefore, he will not be liable for any loss, destruction or deterioration of the
thing bailed if he has taken care. In the case of Calcutta Credit Corporation Ltd v. Prince Peter of Greece, it was
https://www.aspiration.ai/LAW/sis/Solution.jsp?qsetId=JAmrn2p8H1A=&qsetName=LST Mock 27 2021 (CLAT) 105/223
1/17/2021 Mock Analysis

held by Calcutta high court that the defendant has not taken reasonable care to prevent plaintiffs car from
burning.

Bailee is under the duty to return or deliver goods according to the bailor's direction as soon as the time for
which goods were bailed has expired.

In the absence of any agreement, bailee is bound to deliver to the bailor any increase in pro t or any bene t
which may have accrued from the goods bailed (Section 161 of Indian contract act 1872). In the case of
Standard Chartered Bank v. Custodian, it was held by Supreme court that if Shares and debentures are pledged,
bonus shares and dividend are also regarded as Part of it.

Q.67 [30465398]
A, an engineer was hired for a job overseas. While leaving he decided to gift his car to B. A gifted the car to B on
his birthday. Due to the spread of covid-19 pandemic A's overseas job offer was cancelled. A sought back his
car from B stating the car was given in bailment. B challenged this. Decide

a The car is given in bailment as the sole purpose of giving away the car was A leaving the country.

b The car is not given in bailment as there was no valid contract between the parties and there was no
implicit understanding that the car was given to B temporarily.

c A can claim back the car as the very nature of the transaction was bailment and B was expected to return
the car.

d A cannot claim back the car as he gifted the car and gift can be returned only if informed well in advance.

Solution:
 Answer key/Solution
Correct Answer : b
Your Answer : b
Section 148 of Indian contract act 1872 Bailment. The essential requirements of a
bailment are a valid contract and the return of goods when the purpose is accomplished. In the given case A
gifted the car to B and the intention or expectation to return the car was not present.
FeedBack Bookmark

Directions for questions 67 to 105: You have been given some passages followed by questions based on each
passage. You are required to choose the most appropriate option which follows from the passage. Only the
information given in the passage should be used for choosing the answer and no external knowledge of law
howsoever prominent is to be applied.

Passage – 1

The concept of 'bailment' is dealt with in chapter IX of the Indian Contract Act, 1872. 'Bailment' is a technical
common-law term that encompasses change or transfer of possession of goods, upon a contract to return or
dispose of the goods in accordance with the direction of the person's delivering them, once the purpose for

https://www.aspiration.ai/LAW/sis/Solution.jsp?qsetId=JAmrn2p8H1A=&qsetName=LST Mock 27 2021 (CLAT) 106/223


1/17/2021 Mock Analysis

such transfer is completed. The core vital feature of bailment is the change of possession, either physical or
constructive. The Indian Contract Act, 1972, under section 148 de nes the Contract of the Bailment, and in
successive sections deals with certain rights, duties and obligations imposed onto the bailor and bailee, in order
to safeguard their interest.

Essentials of contract of Bailment-


1. The existence of a valid contract- The existence of a valid contract is a foremost condition in bailment which
implies that goods are to be returned when the purpose is ful lled. Finder of lost goods is also known as bailee
although there may not be any existing contract between him and the actual owner.
2. Temporary delivery of goods- The whole concept of bailment revolves around the fact that the goods are
delivered for a temporary period and bailee cannot have permanent possession. Delivery of goods can be done
through actual delivery or through constructive delivery which means doing something which has the effect of
putting the goods in possession of bailee or any other person authorized by him.

3. Return of speci c goods- The bailee is bound to return the goods to the bailor after the purpose for which it
was taken is over. If the person is not returning the goods then it will not be bailment.

Duties and liabilities of a bailor-


1. The bailor is bound to disclose to the bailee, the faults in the goods bailed. which he knows and if he does
not make such disclosures, he is directly responsible for damage arising to the bailee directly from such faults.
2. Bailor is also responsible to the bailee for any loss which the bailee may sustain by reason of the fact that
bailor was not
• To make the bailment
• To receive the goods
• To give directions respecting them
3. Duty to indemnify the bailee- The bailor is duty bound to make good the loss suffered by the bailee where he
was compelled to return the goods before the expiry of the period of bailment
4. Duty to claim back the goods- The bailor is bound to accept the goods upon being returned by the bailee in
accordance with the terms of the agreement. If he refuses to accept it at a proper time, without any reasonable
ground then he will be liable for any loss which may happen to the goods.

Bailee's liabilities -
The bailee is bound to take as much care of goods bailed to him as a man of ordinary prudence would have
under similar circumstances and therefore, he will not be liable for any loss, destruction or deterioration of the
thing bailed if he has taken care. In the case of Calcutta Credit Corporation Ltd v. Prince Peter of Greece, it was
held by Calcutta high court that the defendant has not taken reasonable care to prevent plaintiffs car from
burning.

Bailee is under the duty to return or deliver goods according to the bailor's direction as soon as the time for
which goods were bailed has expired.

In the absence of any agreement, bailee is bound to deliver to the bailor any increase in pro t or any bene t
which may have accrued from the goods bailed (Section 161 of Indian contract act 1872). In the case of
Standard Chartered Bank v. Custodian, it was held by Supreme court that if Shares and debentures are pledged,
bonus shares and dividend are also regarded as Part of it.

https://www.aspiration.ai/LAW/sis/Solution.jsp?qsetId=JAmrn2p8H1A=&qsetName=LST Mock 27 2021 (CLAT) 107/223


1/17/2021 Mock Analysis

Q.68 [30465398]
Natwarlal decided to go on a holy pilgrimage. He asked his friend Chintu to look after his Bull while he was
away. Natwarlal did not disclose to Chintu that his Bull would become violent upon seeing the colour violet. One
day Chintu went to feed the bull wearing a violet-coloured shirt. The Bull attacked and injured him. Chintu sued
Natwarlal. Decide-

a Natwarlal is liable as it is the duty of the bailer to disclose the faults in the goods bailed.

b This is not a case of bailment as a bull cannot be considered as a good.

c Natwarlal is not liable if it is a known fact that bulls turn out ferocious after seeing dark colours.

d Natwarlal is not liable as the reason for ferocious behaviour of the bull cannot be determined by the colour
of the shirt.

Solution:
 Answer key/Solution
Correct Answer : a
Your Answer : a
It is given in the passage that the bailor is duty bound to disclose the faults in the
goods bailed. In the given case Natwarlal knew that the bull would turn ferocious seeing violet colour, but
failed to disclose it. Hence, he will be liable.

FeedBack Bookmark

Directions for questions 67 to 105: You have been given some passages followed by questions based on each
passage. You are required to choose the most appropriate option which follows from the passage. Only the
information given in the passage should be used for choosing the answer and no external knowledge of law
howsoever prominent is to be applied.

Passage – 1

The concept of 'bailment' is dealt with in chapter IX of the Indian Contract Act, 1872. 'Bailment' is a technical
common-law term that encompasses change or transfer of possession of goods, upon a contract to return or
dispose of the goods in accordance with the direction of the person's delivering them, once the purpose for
such transfer is completed. The core vital feature of bailment is the change of possession, either physical or
constructive. The Indian Contract Act, 1972, under section 148 de nes the Contract of the Bailment, and in
successive sections deals with certain rights, duties and obligations imposed onto the bailor and bailee, in order
to safeguard their interest.

Essentials of contract of Bailment-


1. The existence of a valid contract- The existence of a valid contract is a foremost condition in bailment which
implies that goods are to be returned when the purpose is ful lled. Finder of lost goods is also known as bailee
although there may not be any existing contract between him and the actual owner.
2. Temporary delivery of goods- The whole concept of bailment revolves around the fact that the goods are

https://www.aspiration.ai/LAW/sis/Solution.jsp?qsetId=JAmrn2p8H1A=&qsetName=LST Mock 27 2021 (CLAT) 108/223


1/17/2021 Mock Analysis

delivered for a temporary period and bailee cannot have permanent possession. Delivery of goods can be done
through actual delivery or through constructive delivery which means doing something which has the effect of
putting the goods in possession of bailee or any other person authorized by him.

3. Return of speci c goods- The bailee is bound to return the goods to the bailor after the purpose for which it
was taken is over. If the person is not returning the goods then it will not be bailment.

Duties and liabilities of a bailor-


1. The bailor is bound to disclose to the bailee, the faults in the goods bailed. which he knows and if he does
not make such disclosures, he is directly responsible for damage arising to the bailee directly from such faults.
2. Bailor is also responsible to the bailee for any loss which the bailee may sustain by reason of the fact that
bailor was not
• To make the bailment
• To receive the goods
• To give directions respecting them
3. Duty to indemnify the bailee- The bailor is duty bound to make good the loss suffered by the bailee where he
was compelled to return the goods before the expiry of the period of bailment
4. Duty to claim back the goods- The bailor is bound to accept the goods upon being returned by the bailee in
accordance with the terms of the agreement. If he refuses to accept it at a proper time, without any reasonable
ground then he will be liable for any loss which may happen to the goods.

Bailee's liabilities -
The bailee is bound to take as much care of goods bailed to him as a man of ordinary prudence would have
under similar circumstances and therefore, he will not be liable for any loss, destruction or deterioration of the
thing bailed if he has taken care. In the case of Calcutta Credit Corporation Ltd v. Prince Peter of Greece, it was
held by Calcutta high court that the defendant has not taken reasonable care to prevent plaintiffs car from
burning.

Bailee is under the duty to return or deliver goods according to the bailor's direction as soon as the time for
which goods were bailed has expired.

In the absence of any agreement, bailee is bound to deliver to the bailor any increase in pro t or any bene t
which may have accrued from the goods bailed (Section 161 of Indian contract act 1872). In the case of
Standard Chartered Bank v. Custodian, it was held by Supreme court that if Shares and debentures are pledged,
bonus shares and dividend are also regarded as Part of it.

Q.69 [30465398]
A, a farmer stored his grocery in B's warehouse every year after harvesting. The warehouse was nearby a river.
As always A stored his grocery in the warehouse after harvesting and went out of town for a fortnight. During
this period the area experienced unprecedented rainfall which was the highest in last 100 years. People were
expecting the water levels to rise and cause ooding. Due to this heavy rain, water clogged into the warehouse
and destroyed A's groceries. A led a suit against B. B defended saying this had never happened before. Decide-

a B will not be liable as the clogging of water into the warehouse was unexpected and unprecedented.

b B will not be liable as he had taken all su cient care to protect the groceries from seepage of water.

https://www.aspiration.ai/LAW/sis/Solution.jsp?qsetId=JAmrn2p8H1A=&qsetName=LST Mock 27 2021 (CLAT) 109/223


1/17/2021 Mock Analysis

c B will be held liable, as a bailee he had to act as a man of ordinary prudence and should have foreseen the
seepage given the proximity to the river and the amount of rainfall that poured down.

d B will not be held liable as this was not a case of bailment.

Solution:
 Answer key/Solution
Correct Answer : c
Your Answer : d
In the given case it has to be observed that though rainfall was an extraordinary
occurrence and was the highest in the last 100 years, B as a man of ordinary prudence should have acted
reasonably and taken care of the groceries as his warehouse was close to a river. Hence, option c is the
correct answer.
FeedBack Bookmark

Directions for questions 67 to 105: You have been given some passages followed by questions based on each
passage. You are required to choose the most appropriate option which follows from the passage. Only the
information given in the passage should be used for choosing the answer and no external knowledge of law
howsoever prominent is to be applied.

Passage – 1

The concept of 'bailment' is dealt with in chapter IX of the Indian Contract Act, 1872. 'Bailment' is a technical
common-law term that encompasses change or transfer of possession of goods, upon a contract to return or
dispose of the goods in accordance with the direction of the person's delivering them, once the purpose for
such transfer is completed. The core vital feature of bailment is the change of possession, either physical or
constructive. The Indian Contract Act, 1972, under section 148 de nes the Contract of the Bailment, and in
successive sections deals with certain rights, duties and obligations imposed onto the bailor and bailee, in order
to safeguard their interest.

Essentials of contract of Bailment-


1. The existence of a valid contract- The existence of a valid contract is a foremost condition in bailment which
implies that goods are to be returned when the purpose is ful lled. Finder of lost goods is also known as bailee
although there may not be any existing contract between him and the actual owner.
2. Temporary delivery of goods- The whole concept of bailment revolves around the fact that the goods are
delivered for a temporary period and bailee cannot have permanent possession. Delivery of goods can be done
through actual delivery or through constructive delivery which means doing something which has the effect of
putting the goods in possession of bailee or any other person authorized by him.

3. Return of speci c goods- The bailee is bound to return the goods to the bailor after the purpose for which it
was taken is over. If the person is not returning the goods then it will not be bailment.

Duties and liabilities of a bailor-


1. The bailor is bound to disclose to the bailee, the faults in the goods bailed. which he knows and if he does

https://www.aspiration.ai/LAW/sis/Solution.jsp?qsetId=JAmrn2p8H1A=&qsetName=LST Mock 27 2021 (CLAT) 110/223


1/17/2021 Mock Analysis

not make such disclosures, he is directly responsible for damage arising to the bailee directly from such faults.
2. Bailor is also responsible to the bailee for any loss which the bailee may sustain by reason of the fact that
bailor was not
• To make the bailment
• To receive the goods
• To give directions respecting them
3. Duty to indemnify the bailee- The bailor is duty bound to make good the loss suffered by the bailee where he
was compelled to return the goods before the expiry of the period of bailment
4. Duty to claim back the goods- The bailor is bound to accept the goods upon being returned by the bailee in
accordance with the terms of the agreement. If he refuses to accept it at a proper time, without any reasonable
ground then he will be liable for any loss which may happen to the goods.

Bailee's liabilities -
The bailee is bound to take as much care of goods bailed to him as a man of ordinary prudence would have
under similar circumstances and therefore, he will not be liable for any loss, destruction or deterioration of the
thing bailed if he has taken care. In the case of Calcutta Credit Corporation Ltd v. Prince Peter of Greece, it was
held by Calcutta high court that the defendant has not taken reasonable care to prevent plaintiffs car from
burning.

Bailee is under the duty to return or deliver goods according to the bailor's direction as soon as the time for
which goods were bailed has expired.

In the absence of any agreement, bailee is bound to deliver to the bailor any increase in pro t or any bene t
which may have accrued from the goods bailed (Section 161 of Indian contract act 1872). In the case of
Standard Chartered Bank v. Custodian, it was held by Supreme court that if Shares and debentures are pledged,
bonus shares and dividend are also regarded as Part of it.

Q.70 [30465398]
Which of the following is not an example for a contract of bailment?

a A gives his radio to B for repair. B assures to return it within a week.

b Z takes Y's car to pick up Z's relatives from the airport.

c A receives a gift voucher from Zax enterprise worth rupees 5000 rupees.

d N borrows A's dagger to use it in his construction site.

Solution:
 Answer key/Solution
Correct Answer : c
Your Answer : c
In the given case all options except c are examples for bailment. Option c comes
under the concept of gift.

FeedBack Bookmark

https://www.aspiration.ai/LAW/sis/Solution.jsp?qsetId=JAmrn2p8H1A=&qsetName=LST Mock 27 2021 (CLAT) 111/223


1/17/2021 Mock Analysis

Directions for questions 67 to 105: You have been given some passages followed by questions based on each
passage. You are required to choose the most appropriate option which follows from the passage. Only the
information given in the passage should be used for choosing the answer and no external knowledge of law
howsoever prominent is to be applied.

Passage – 1

The concept of 'bailment' is dealt with in chapter IX of the Indian Contract Act, 1872. 'Bailment' is a technical
common-law term that encompasses change or transfer of possession of goods, upon a contract to return or
dispose of the goods in accordance with the direction of the person's delivering them, once the purpose for
such transfer is completed. The core vital feature of bailment is the change of possession, either physical or
constructive. The Indian Contract Act, 1972, under section 148 de nes the Contract of the Bailment, and in
successive sections deals with certain rights, duties and obligations imposed onto the bailor and bailee, in order
to safeguard their interest.

Essentials of contract of Bailment-


1. The existence of a valid contract- The existence of a valid contract is a foremost condition in bailment which
implies that goods are to be returned when the purpose is ful lled. Finder of lost goods is also known as bailee
although there may not be any existing contract between him and the actual owner.
2. Temporary delivery of goods- The whole concept of bailment revolves around the fact that the goods are
delivered for a temporary period and bailee cannot have permanent possession. Delivery of goods can be done
through actual delivery or through constructive delivery which means doing something which has the effect of
putting the goods in possession of bailee or any other person authorized by him.

3. Return of speci c goods- The bailee is bound to return the goods to the bailor after the purpose for which it
was taken is over. If the person is not returning the goods then it will not be bailment.

Duties and liabilities of a bailor-


1. The bailor is bound to disclose to the bailee, the faults in the goods bailed. which he knows and if he does
not make such disclosures, he is directly responsible for damage arising to the bailee directly from such faults.
2. Bailor is also responsible to the bailee for any loss which the bailee may sustain by reason of the fact that
bailor was not
• To make the bailment
• To receive the goods
• To give directions respecting them
3. Duty to indemnify the bailee- The bailor is duty bound to make good the loss suffered by the bailee where he
was compelled to return the goods before the expiry of the period of bailment
4. Duty to claim back the goods- The bailor is bound to accept the goods upon being returned by the bailee in
accordance with the terms of the agreement. If he refuses to accept it at a proper time, without any reasonable
ground then he will be liable for any loss which may happen to the goods.

Bailee's liabilities -
The bailee is bound to take as much care of goods bailed to him as a man of ordinary prudence would have
under similar circumstances and therefore, he will not be liable for any loss, destruction or deterioration of the
thing bailed if he has taken care. In the case of Calcutta Credit Corporation Ltd v. Prince Peter of Greece, it was
held by Calcutta high court that the defendant has not taken reasonable care to prevent plaintiffs car from
burning.

https://www.aspiration.ai/LAW/sis/Solution.jsp?qsetId=JAmrn2p8H1A=&qsetName=LST Mock 27 2021 (CLAT) 112/223


1/17/2021 Mock Analysis

Bailee is under the duty to return or deliver goods according to the bailor's direction as soon as the time for
which goods were bailed has expired.

In the absence of any agreement, bailee is bound to deliver to the bailor any increase in pro t or any bene t
which may have accrued from the goods bailed (Section 161 of Indian contract act 1872). In the case of
Standard Chartered Bank v. Custodian, it was held by Supreme court that if Shares and debentures are pledged,
bonus shares and dividend are also regarded as Part of it.

Q.71 [30465398]
In which of the following scenarios a validcondition/duty of bailment is not taking place?

a A gives his bike to B on bailment and discloses that its front brakes are not working in its full capacity.

b Z gives his watch to R and asks him to keep it for 15 days.

c N the bailee of Z's laptop takes the laptop to the repair shop when he discovers a malware attack on the
system.

d L the bailee of Y's bike rides above the speed limit and commits an accident.

Solution:
 Answer key/Solution
Correct Answer : d
Your Answer : d
In all the options except option d, a valid condition/requirement of contract of
bailment is taking place. Hence option d is the correct answer.
FeedBack Bookmark

Directions for questions 67 to 105: You have been given some passages followed by questions based on each
passage. You are required to choose the most appropriate option which follows from the passage. Only the
information given in the passage should be used for choosing the answer and no external knowledge of law
howsoever prominent is to be applied.

Passage – 2

If we make a critical survey of the leading Constitutions of the world, we nd that theoretically speaking the
Supreme Court of India has wider jurisdiction than any other Supreme Court in any part of the world. The
jurisdiction of the Court can be kept in four categories, viz., original, writ, appellate and advisory.

Original jurisdiction - A court is said to have original jurisdiction when it possesses the authority to hear and
determine the case in the rst instance. This type of jurisdiction has been dealt with in Article 131 of the
Constitution.
The Supreme Court of India possesses original and exclusive jurisdiction in any dispute:
(a) Between the Government of India, and one or more states or

https://www.aspiration.ai/LAW/sis/Solution.jsp?qsetId=JAmrn2p8H1A=&qsetName=LST Mock 27 2021 (CLAT) 113/223


1/17/2021 Mock Analysis

(b) Between the Government of India and any state or states on one side and one or more other states on the
other or
(c) Between two or more states.

The dispute relating to the original jurisdiction of the Court must involve a question of fact or law on which the
existence of a legal right depends. A legal right is de ned "as any advantage or bene t which is in any manner
conferred upon a person by a rule of law". The Supreme Court has no original jurisdiction in disputes between
individuals or between associations or local bodies.

Writ jurisdiction- Under Article 32, the Supreme Court can entertain an application for the issue of a
constitutional writ for the enforcement of Fundamental Rights. This is termed as original jurisdiction as the
aggrieved party can move the Apex court directly through a petition instead of coming through a High Court by
way of an appeal. There are 5 types of writ petitions. They are-

Habeas Corpus- This writ is used to enforce the fundamental right of individual liberty against unlawful
detention. Through Habeas Corpus, Supreme Court/High Court orders one person who has arrested another
person to bring the body of the latter before the court.

Mandamus- This writ is used by the court to order the public o cial who has failed to perform his duty or
refused to do his duty, to resume his work. Besides public o cials, Mandamus can be issued against any public
body, a corporation, an inferior court, a tribunal, or government for the same purpose.

Prohibition- The literal meaning of 'Prohibition' is 'To forbid.' A court that is higher in position issues a Prohibition
writ against a court that is lower in position to prevent the latter from exceeding its jurisdiction or usurping a
jurisdiction that it does not possess. It directs inactivity.

Certiorari - This writ is issued by a court higher in authority to a lower court or tribunal ordering them either to
transfer a case pending with them to itself or squash their order in a case. It is issued on the grounds of an
excess of jurisdiction or lack of jurisdiction or error of law. It not only prevents but also cures for the mistakes in
the judiciary.

Quo Warranto- The literal meaning of the writ of 'Quo-Warranto' is 'By what authority or warrant.' The Supreme
Court or High Court issues this writ to prevent illegal usurpation of a public o ce by a person. Through this writ,
the court enquires into the legality of a claim of a person to a public o ce

Appellate Jurisdiction - The appellate jurisdiction of the Supreme Court can be invoked by a certi cate granted
by the High Court concerned under Article 132(1), 133(1) or 134 of the Constitution in respect of any judgement,
decree or nal order of a High Court in both civil and criminal cases, involving substantial questions of law as to
the interpretation of the Constitution. Appeals also lie to the Supreme Court in civil matters if the High Court
concerned certi es: (a) that the case involves a substantial question of law of general importance, and (b) that,
in the opinion of the High Court, the said question needs to be decided by the Supreme Court. In criminal cases,
an appeal lies to the Supreme Court if the High Court (a) has on appeal reversed an order of acquittal of an
accused person and sentenced him to death or to imprisonment for life or for a period of not less than 10 years,
or (b) has withdrawn for trial before itself any case from any Court subordinate to its authority and has in such
trial convicted the accused and sentenced him to death or to imprisonment for life or for a period of not less
than 10 years, or (c) certi ed that the case is a t one for appeal to the Supreme Court. Parliament is authorised
to confer on the Supreme Court any further powers to entertain and hear appeals from any judgement, nal
order or sentence in a criminal proceeding of a High Court.

https://www.aspiration.ai/LAW/sis/Solution.jsp?qsetId=JAmrn2p8H1A=&qsetName=LST Mock 27 2021 (CLAT) 114/223


1/17/2021 Mock Analysis

Q.72 [30465398]
A, approached his village Tahsildar to get a certi cate for conversion of his agricultural land into a site, as it was
barren and no longer yielded crops. Tahsildar asked A to pay 10 Lakhs failing which his certi cate would not be
issued. A was advised to le a complaint in supreme court of India. Does the supreme court have the
jurisdiction?

a This case can be led under original jurisdiction according to article 131 of the constitution.

b A can le a writ petition of Certiorari under article 32 of the constitution.

c A can le a writ petition of mandamus under article 32 of the constitution.

d A can approach the supreme court only by appellate jurisdiction. Therefore, he has to approach a lower
court before appealing to the supreme court.

Solution:
 Answer key/Solution
Correct Answer : c
Your Answer : c
The writ of mandamus is used by the court to order the public o cial who has
failed to perform his duty or refused to do his duty, to resume his work. In the given case a Tahsildar is a
public o cial and has refused to do his duty. Hence, a writ of mandamus can be led against him. Option c is
the correct answer.
FeedBack Bookmark

Directions for questions 67 to 105: You have been given some passages followed by questions based on each
passage. You are required to choose the most appropriate option which follows from the passage. Only the
information given in the passage should be used for choosing the answer and no external knowledge of law
howsoever prominent is to be applied.

Passage – 2

If we make a critical survey of the leading Constitutions of the world, we nd that theoretically speaking the
Supreme Court of India has wider jurisdiction than any other Supreme Court in any part of the world. The
jurisdiction of the Court can be kept in four categories, viz., original, writ, appellate and advisory.

Original jurisdiction - A court is said to have original jurisdiction when it possesses the authority to hear and
determine the case in the rst instance. This type of jurisdiction has been dealt with in Article 131 of the
Constitution.
The Supreme Court of India possesses original and exclusive jurisdiction in any dispute:
(a) Between the Government of India, and one or more states or
(b) Between the Government of India and any state or states on one side and one or more other states on the
other or
(c) Between two or more states.

https://www.aspiration.ai/LAW/sis/Solution.jsp?qsetId=JAmrn2p8H1A=&qsetName=LST Mock 27 2021 (CLAT) 115/223


1/17/2021 Mock Analysis

The dispute relating to the original jurisdiction of the Court must involve a question of fact or law on which the
existence of a legal right depends. A legal right is de ned "as any advantage or bene t which is in any manner
conferred upon a person by a rule of law". The Supreme Court has no original jurisdiction in disputes between
individuals or between associations or local bodies.

Writ jurisdiction- Under Article 32, the Supreme Court can entertain an application for the issue of a
constitutional writ for the enforcement of Fundamental Rights. This is termed as original jurisdiction as the
aggrieved party can move the Apex court directly through a petition instead of coming through a High Court by
way of an appeal. There are 5 types of writ petitions. They are-

Habeas Corpus- This writ is used to enforce the fundamental right of individual liberty against unlawful
detention. Through Habeas Corpus, Supreme Court/High Court orders one person who has arrested another
person to bring the body of the latter before the court.

Mandamus- This writ is used by the court to order the public o cial who has failed to perform his duty or
refused to do his duty, to resume his work. Besides public o cials, Mandamus can be issued against any public
body, a corporation, an inferior court, a tribunal, or government for the same purpose.

Prohibition- The literal meaning of 'Prohibition' is 'To forbid.' A court that is higher in position issues a Prohibition
writ against a court that is lower in position to prevent the latter from exceeding its jurisdiction or usurping a
jurisdiction that it does not possess. It directs inactivity.

Certiorari - This writ is issued by a court higher in authority to a lower court or tribunal ordering them either to
transfer a case pending with them to itself or squash their order in a case. It is issued on the grounds of an
excess of jurisdiction or lack of jurisdiction or error of law. It not only prevents but also cures for the mistakes in
the judiciary.

Quo Warranto- The literal meaning of the writ of 'Quo-Warranto' is 'By what authority or warrant.' The Supreme
Court or High Court issues this writ to prevent illegal usurpation of a public o ce by a person. Through this writ,
the court enquires into the legality of a claim of a person to a public o ce

Appellate Jurisdiction - The appellate jurisdiction of the Supreme Court can be invoked by a certi cate granted
by the High Court concerned under Article 132(1), 133(1) or 134 of the Constitution in respect of any judgement,
decree or nal order of a High Court in both civil and criminal cases, involving substantial questions of law as to
the interpretation of the Constitution. Appeals also lie to the Supreme Court in civil matters if the High Court
concerned certi es: (a) that the case involves a substantial question of law of general importance, and (b) that,
in the opinion of the High Court, the said question needs to be decided by the Supreme Court. In criminal cases,
an appeal lies to the Supreme Court if the High Court (a) has on appeal reversed an order of acquittal of an
accused person and sentenced him to death or to imprisonment for life or for a period of not less than 10 years,
or (b) has withdrawn for trial before itself any case from any Court subordinate to its authority and has in such
trial convicted the accused and sentenced him to death or to imprisonment for life or for a period of not less
than 10 years, or (c) certi ed that the case is a t one for appeal to the Supreme Court. Parliament is authorised
to confer on the Supreme Court any further powers to entertain and hear appeals from any judgement, nal
order or sentence in a criminal proceeding of a High Court.

https://www.aspiration.ai/LAW/sis/Solution.jsp?qsetId=JAmrn2p8H1A=&qsetName=LST Mock 27 2021 (CLAT) 116/223


1/17/2021 Mock Analysis

Q.73 [30465398]
River Raga was owing between the state of Ranga and Kripa in the country, Sindia. Ranga decided to build a
dam to the river to relieve the farmers from the drought problem during summer. State of Kripa strongly resisted
this action as they were predominantly dependent on the River Raga's water and construction of a dam would
limit their water supply. Both the states approached the supreme court of Sindia. Decide the jurisdiction of the
supreme court to entertain this case. (Laws of Sindia are in pari- materia with the laws of India)

a The states can approach the supreme court through special leave petition under article

b The supreme court has the power to entertain this case under its original jurisdiction.

c The supreme court does not have the power to entertain the given case.

d The states can solve this dispute by approaching an alternate dispute resolution. ADR is fast and time
consuming.

Solution:
 Answer key/Solution
Correct Answer : b
Your Answer : b
The given case comes under the original jurisdiction of the supreme court of
India. Option c under paragraph 'original jurisdiction' of the passage rightly mentions so. Hence, c is the
correct answer.
FeedBack Bookmark

Directions for questions 67 to 105: You have been given some passages followed by questions based on each
passage. You are required to choose the most appropriate option which follows from the passage. Only the
information given in the passage should be used for choosing the answer and no external knowledge of law
howsoever prominent is to be applied.

Passage – 2

If we make a critical survey of the leading Constitutions of the world, we nd that theoretically speaking the
Supreme Court of India has wider jurisdiction than any other Supreme Court in any part of the world. The
jurisdiction of the Court can be kept in four categories, viz., original, writ, appellate and advisory.

Original jurisdiction - A court is said to have original jurisdiction when it possesses the authority to hear and
determine the case in the rst instance. This type of jurisdiction has been dealt with in Article 131 of the
Constitution.
The Supreme Court of India possesses original and exclusive jurisdiction in any dispute:
(a) Between the Government of India, and one or more states or
(b) Between the Government of India and any state or states on one side and one or more other states on the
other or
(c) Between two or more states.

https://www.aspiration.ai/LAW/sis/Solution.jsp?qsetId=JAmrn2p8H1A=&qsetName=LST Mock 27 2021 (CLAT) 117/223


1/17/2021 Mock Analysis

The dispute relating to the original jurisdiction of the Court must involve a question of fact or law on which the
existence of a legal right depends. A legal right is de ned "as any advantage or bene t which is in any manner
conferred upon a person by a rule of law". The Supreme Court has no original jurisdiction in disputes between
individuals or between associations or local bodies.

Writ jurisdiction- Under Article 32, the Supreme Court can entertain an application for the issue of a
constitutional writ for the enforcement of Fundamental Rights. This is termed as original jurisdiction as the
aggrieved party can move the Apex court directly through a petition instead of coming through a High Court by
way of an appeal. There are 5 types of writ petitions. They are-

Habeas Corpus- This writ is used to enforce the fundamental right of individual liberty against unlawful
detention. Through Habeas Corpus, Supreme Court/High Court orders one person who has arrested another
person to bring the body of the latter before the court.

Mandamus- This writ is used by the court to order the public o cial who has failed to perform his duty or
refused to do his duty, to resume his work. Besides public o cials, Mandamus can be issued against any public
body, a corporation, an inferior court, a tribunal, or government for the same purpose.

Prohibition- The literal meaning of 'Prohibition' is 'To forbid.' A court that is higher in position issues a Prohibition
writ against a court that is lower in position to prevent the latter from exceeding its jurisdiction or usurping a
jurisdiction that it does not possess. It directs inactivity.

Certiorari - This writ is issued by a court higher in authority to a lower court or tribunal ordering them either to
transfer a case pending with them to itself or squash their order in a case. It is issued on the grounds of an
excess of jurisdiction or lack of jurisdiction or error of law. It not only prevents but also cures for the mistakes in
the judiciary.

Quo Warranto- The literal meaning of the writ of 'Quo-Warranto' is 'By what authority or warrant.' The Supreme
Court or High Court issues this writ to prevent illegal usurpation of a public o ce by a person. Through this writ,
the court enquires into the legality of a claim of a person to a public o ce

Appellate Jurisdiction - The appellate jurisdiction of the Supreme Court can be invoked by a certi cate granted
by the High Court concerned under Article 132(1), 133(1) or 134 of the Constitution in respect of any judgement,
decree or nal order of a High Court in both civil and criminal cases, involving substantial questions of law as to
the interpretation of the Constitution. Appeals also lie to the Supreme Court in civil matters if the High Court
concerned certi es: (a) that the case involves a substantial question of law of general importance, and (b) that,
in the opinion of the High Court, the said question needs to be decided by the Supreme Court. In criminal cases,
an appeal lies to the Supreme Court if the High Court (a) has on appeal reversed an order of acquittal of an
accused person and sentenced him to death or to imprisonment for life or for a period of not less than 10 years,
or (b) has withdrawn for trial before itself any case from any Court subordinate to its authority and has in such
trial convicted the accused and sentenced him to death or to imprisonment for life or for a period of not less
than 10 years, or (c) certi ed that the case is a t one for appeal to the Supreme Court. Parliament is authorised
to confer on the Supreme Court any further powers to entertain and hear appeals from any judgement, nal
order or sentence in a criminal proceeding of a High Court.

Q.74 [30465398]
Which of the following statements regarding appellate jurisdiction is false?

https://www.aspiration.ai/LAW/sis/Solution.jsp?qsetId=JAmrn2p8H1A=&qsetName=LST Mock 27 2021 (CLAT) 118/223


1/17/2021 Mock Analysis

a The appellate jurisdiction of the Supreme Court can be invoked by a certi cate granted by the High Court
concerned under Article 135(1), 130(1) or 136 of the Constitution

b Appeal can also be made in the cases of civil cases

c Parliament is authorised to confer on the Supreme Court any further powers to entertain and hear appeals
from any judgement, nal order or sentence in a criminal proceeding of a High Court.

d In criminal cases, an appeal lies to the Supreme Court if the High Court has on appeal reversed an order of
acquittal of an accused person and sentenced him to death or to imprisonment for life or for a period of not less
than 10 years.

Solution:
 Answer key/Solution
Correct Answer : a
Your Answer : a
Appellate jurisdiction of the supreme court can be invoked under Article 132(1),
133(1) or 134 of the Constitution and not under Article 135(1), 130(1) or 136 of the Constitution. Therefore,
option a is the correct answer.

FeedBack Bookmark

Directions for questions 67 to 105: You have been given some passages followed by questions based on each
passage. You are required to choose the most appropriate option which follows from the passage. Only the
information given in the passage should be used for choosing the answer and no external knowledge of law
howsoever prominent is to be applied.

Passage – 2

If we make a critical survey of the leading Constitutions of the world, we nd that theoretically speaking the
Supreme Court of India has wider jurisdiction than any other Supreme Court in any part of the world. The
jurisdiction of the Court can be kept in four categories, viz., original, writ, appellate and advisory.

Original jurisdiction - A court is said to have original jurisdiction when it possesses the authority to hear and
determine the case in the rst instance. This type of jurisdiction has been dealt with in Article 131 of the
Constitution.
The Supreme Court of India possesses original and exclusive jurisdiction in any dispute:
(a) Between the Government of India, and one or more states or
(b) Between the Government of India and any state or states on one side and one or more other states on the
other or
(c) Between two or more states.

The dispute relating to the original jurisdiction of the Court must involve a question of fact or law on which the
existence of a legal right depends. A legal right is de ned "as any advantage or bene t which is in any manner
conferred upon a person by a rule of law". The Supreme Court has no original jurisdiction in disputes between

https://www.aspiration.ai/LAW/sis/Solution.jsp?qsetId=JAmrn2p8H1A=&qsetName=LST Mock 27 2021 (CLAT) 119/223


1/17/2021 Mock Analysis

individuals or between associations or local bodies.

Writ jurisdiction- Under Article 32, the Supreme Court can entertain an application for the issue of a
constitutional writ for the enforcement of Fundamental Rights. This is termed as original jurisdiction as the
aggrieved party can move the Apex court directly through a petition instead of coming through a High Court by
way of an appeal. There are 5 types of writ petitions. They are-

Habeas Corpus- This writ is used to enforce the fundamental right of individual liberty against unlawful
detention. Through Habeas Corpus, Supreme Court/High Court orders one person who has arrested another
person to bring the body of the latter before the court.

Mandamus- This writ is used by the court to order the public o cial who has failed to perform his duty or
refused to do his duty, to resume his work. Besides public o cials, Mandamus can be issued against any public
body, a corporation, an inferior court, a tribunal, or government for the same purpose.

Prohibition- The literal meaning of 'Prohibition' is 'To forbid.' A court that is higher in position issues a Prohibition
writ against a court that is lower in position to prevent the latter from exceeding its jurisdiction or usurping a
jurisdiction that it does not possess. It directs inactivity.

Certiorari - This writ is issued by a court higher in authority to a lower court or tribunal ordering them either to
transfer a case pending with them to itself or squash their order in a case. It is issued on the grounds of an
excess of jurisdiction or lack of jurisdiction or error of law. It not only prevents but also cures for the mistakes in
the judiciary.

Quo Warranto- The literal meaning of the writ of 'Quo-Warranto' is 'By what authority or warrant.' The Supreme
Court or High Court issues this writ to prevent illegal usurpation of a public o ce by a person. Through this writ,
the court enquires into the legality of a claim of a person to a public o ce

Appellate Jurisdiction - The appellate jurisdiction of the Supreme Court can be invoked by a certi cate granted
by the High Court concerned under Article 132(1), 133(1) or 134 of the Constitution in respect of any judgement,
decree or nal order of a High Court in both civil and criminal cases, involving substantial questions of law as to
the interpretation of the Constitution. Appeals also lie to the Supreme Court in civil matters if the High Court
concerned certi es: (a) that the case involves a substantial question of law of general importance, and (b) that,
in the opinion of the High Court, the said question needs to be decided by the Supreme Court. In criminal cases,
an appeal lies to the Supreme Court if the High Court (a) has on appeal reversed an order of acquittal of an
accused person and sentenced him to death or to imprisonment for life or for a period of not less than 10 years,
or (b) has withdrawn for trial before itself any case from any Court subordinate to its authority and has in such
trial convicted the accused and sentenced him to death or to imprisonment for life or for a period of not less
than 10 years, or (c) certi ed that the case is a t one for appeal to the Supreme Court. Parliament is authorised
to confer on the Supreme Court any further powers to entertain and hear appeals from any judgement, nal
order or sentence in a criminal proceeding of a High Court.

Q.75 [30465398]
In which of the following cases original jurisdiction under article 131 cannot be invoked?

a States A and B have a dispute over the state boundary and approach the supreme court.

https://www.aspiration.ai/LAW/sis/Solution.jsp?qsetId=JAmrn2p8H1A=&qsetName=LST Mock 27 2021 (CLAT) 120/223


1/17/2021 Mock Analysis

b The Government of India introduced farm bills 2020 and the state of Punjab approached the supreme court
questioning the validity and authority of the government in introduction of the bill.

c Ajmal Guru, a dreaded terrorist involved in bombing of a hotel was awarded the death sentence. He
approached the supreme court challenging the judgement.

d Steel authority of India opened a new factory which dealt with chemicals in a forest area. The factory was
near a wildlife sanctuary. The concerned state along with the national green tribunal approached the supreme
court to stop the factory.

Solution:
 Answer key/Solution
Correct Answer : c
Your Answer : c
In the given case option c is an example of appellate jurisdiction. Apart from that
all others are original jurisdiction. Hence, option c is the correct answer.

FeedBack Bookmark

Directions for questions 67 to 105: You have been given some passages followed by questions based on each
passage. You are required to choose the most appropriate option which follows from the passage. Only the
information given in the passage should be used for choosing the answer and no external knowledge of law
howsoever prominent is to be applied.

Passage – 2

If we make a critical survey of the leading Constitutions of the world, we nd that theoretically speaking the
Supreme Court of India has wider jurisdiction than any other Supreme Court in any part of the world. The
jurisdiction of the Court can be kept in four categories, viz., original, writ, appellate and advisory.

Original jurisdiction - A court is said to have original jurisdiction when it possesses the authority to hear and
determine the case in the rst instance. This type of jurisdiction has been dealt with in Article 131 of the
Constitution.
The Supreme Court of India possesses original and exclusive jurisdiction in any dispute:
(a) Between the Government of India, and one or more states or
(b) Between the Government of India and any state or states on one side and one or more other states on the
other or
(c) Between two or more states.

The dispute relating to the original jurisdiction of the Court must involve a question of fact or law on which the
existence of a legal right depends. A legal right is de ned "as any advantage or bene t which is in any manner
conferred upon a person by a rule of law". The Supreme Court has no original jurisdiction in disputes between
individuals or between associations or local bodies.

Writ jurisdiction- Under Article 32, the Supreme Court can entertain an application for the issue of a

https://www.aspiration.ai/LAW/sis/Solution.jsp?qsetId=JAmrn2p8H1A=&qsetName=LST Mock 27 2021 (CLAT) 121/223


1/17/2021 Mock Analysis

constitutional writ for the enforcement of Fundamental Rights. This is termed as original jurisdiction as the
aggrieved party can move the Apex court directly through a petition instead of coming through a High Court by
way of an appeal. There are 5 types of writ petitions. They are-

Habeas Corpus- This writ is used to enforce the fundamental right of individual liberty against unlawful
detention. Through Habeas Corpus, Supreme Court/High Court orders one person who has arrested another
person to bring the body of the latter before the court.

Mandamus- This writ is used by the court to order the public o cial who has failed to perform his duty or
refused to do his duty, to resume his work. Besides public o cials, Mandamus can be issued against any public
body, a corporation, an inferior court, a tribunal, or government for the same purpose.

Prohibition- The literal meaning of 'Prohibition' is 'To forbid.' A court that is higher in position issues a Prohibition
writ against a court that is lower in position to prevent the latter from exceeding its jurisdiction or usurping a
jurisdiction that it does not possess. It directs inactivity.

Certiorari - This writ is issued by a court higher in authority to a lower court or tribunal ordering them either to
transfer a case pending with them to itself or squash their order in a case. It is issued on the grounds of an
excess of jurisdiction or lack of jurisdiction or error of law. It not only prevents but also cures for the mistakes in
the judiciary.

Quo Warranto- The literal meaning of the writ of 'Quo-Warranto' is 'By what authority or warrant.' The Supreme
Court or High Court issues this writ to prevent illegal usurpation of a public o ce by a person. Through this writ,
the court enquires into the legality of a claim of a person to a public o ce

Appellate Jurisdiction - The appellate jurisdiction of the Supreme Court can be invoked by a certi cate granted
by the High Court concerned under Article 132(1), 133(1) or 134 of the Constitution in respect of any judgement,
decree or nal order of a High Court in both civil and criminal cases, involving substantial questions of law as to
the interpretation of the Constitution. Appeals also lie to the Supreme Court in civil matters if the High Court
concerned certi es: (a) that the case involves a substantial question of law of general importance, and (b) that,
in the opinion of the High Court, the said question needs to be decided by the Supreme Court. In criminal cases,
an appeal lies to the Supreme Court if the High Court (a) has on appeal reversed an order of acquittal of an
accused person and sentenced him to death or to imprisonment for life or for a period of not less than 10 years,
or (b) has withdrawn for trial before itself any case from any Court subordinate to its authority and has in such
trial convicted the accused and sentenced him to death or to imprisonment for life or for a period of not less
than 10 years, or (c) certi ed that the case is a t one for appeal to the Supreme Court. Parliament is authorised
to confer on the Supreme Court any further powers to entertain and hear appeals from any judgement, nal
order or sentence in a criminal proceeding of a High Court.

Q.76 [30465398]
Choose the correct sequence of writ petitions for the below mentioned examples.
(I) A was arrested and was denied bail in a case of pelting stones against a public worker. A led a writ petition.
(II) The court of Z was stopped by the high court in entertaining a case as it was beyond the jurisdiction of court
of Z.
(III) A, the son of a local MP used to attend to his father's work and meetings while his father stayed at home.
The government stopped A from doing so.
(IV) A, a public employee was sent notice to attend his work as he drew salary without attending for months and
faked his presence in the o ce.

https://www.aspiration.ai/LAW/sis/Solution.jsp?qsetId=JAmrn2p8H1A=&qsetName=LST Mock 27 2021 (CLAT) 122/223


1/17/2021 Mock Analysis

a Quo Warranto, Mandamus, Certiorari, Habeas corpus

b Habeas Corpus, Certiorari, Quo warranto, Mandamus

c Habeas Corpus, Mandamus, Mandamus, Quo-warranto

d Quo warranto, Certiorari, certiorari, Mandamus.

Solution:
 Answer key/Solution
Correct Answer : b
Your Answer : b
Option ii is the correct set of writ petitions.

FeedBack Bookmark

https://www.aspiration.ai/LAW/sis/Solution.jsp?qsetId=JAmrn2p8H1A=&qsetName=LST Mock 27 2021 (CLAT) 123/223


1/17/2021 Mock Analysis

Directions for questions 67 to 105: You have been given some passages followed by questions based on each
passage. You are required to choose the most appropriate option which follows from the passage. Only the
information given in the passage should be used for choosing the answer and no external knowledge of law
howsoever prominent is to be applied.

Passage – 3

Section 378 of the IPC de nes theft as, "Whoever, intending to take dishonestly any movable property out of the
possession of any person without that person's consent, moves that property to such taking, is said to commit
theft". We can say that section 378 of the IPC de nes 'theft' as the dishonest removal of moveable property 'out
of the possession of any person' without the consent of that person. The Section further provides in the
explanations given that any object attached to the earth would be considered as an immovable property hence it
could not be a subject of theft but once it is removed from the earth it would become a movable property and
could be stolen. The consent that must be attained for a property to be taken without it being considered theft
may either be express or implied.

Prerequisite for Theft-

Dishonest intention- Section 24 of IPC provides that dishonesty means the intention of wrongful gain or
wrongful loss. Section 23 of the IPC provides the de nition of both wrongful gain and wrongful loss. Wrongful
gain means gaining any property unlawfully, the person who is losing the property is the legal owner of such
property. Wrongful loss means the loss brought about by unlawful means.

Movable property- Section 22 of IPC has provided the de nition of movable property; means that any corporeal
property except land and things permanently attached to the earth. Only movable property can be stolen as it is
impossible to take immovable property away. Immovable property can be converted into movable property and
once it has been converted such property can be stolen.

Property in possession- In order for theft to have occurred, the property being stolen must be taken from the
possession from the owner of such a property. If the property does not have an owner then such property
cannot be said to have been stolen if a person acquires such property. For example; if A nds a gold nugget in a
stream and he takes the gold home, it cannot be considered theft as the gold nugget has no owner.

No consent- The property that is in question must have been taken without the consent of the owner of such a
property. The consent can either be implied or expressed. The consent given must also be free meaning that
such consent must not be acquired through means of coercion or fear of injury or misrepresentation of facts.
Consent, given during state of drunkenness or intoxication as well as consent given by a person of unsound
mind cannot be considered to be a valid consent.

Punishment for theft- Section 379 of the IPC establishes the punishment for theft as imprisonment up to 3
years or ne or both. Other succeeding provisions contain more severe punishment for theft under aggravating
circumstances. For example, Section 380 of IPC punishes the commission of theft in a building, tent or vessel
used for dwelling or residence. The object of this provision is to provide greater security to properties in dwelling
premises. The punishment for this is imprisonment up to 7 years along with ne.

Q.77 [30465398]
A visits B's birthday party. In the party he sees a costly gold ring of A left unattended. A takes the ring and hides
it in such a place that B could never nd out. Will A be held guilty for theft?

https://www.aspiration.ai/LAW/sis/Solution.jsp?qsetId=JAmrn2p8H1A=&qsetName=LST Mock 27 2021 (CLAT) 124/223


1/17/2021 Mock Analysis

a A shall not be held liable for theft as property was not taken out of the possession of B

b A shall be held liable for theft as there was dishonest intention and hiding it in a place never to be found out
is equal to taking out of the possession.

c A shall not be held liable for theft for hiding it, but will be liable once he takes it from that place.

d A shall be held liable for cheating and not for theft as the necessity prerequisites for theft are not ful lled.

Solution:
 Answer key/Solution
Correct Answer : b
Hiding a property in a place never to be found out is equal to taking the property
out of the possession of the owner. There was dishonest intention and no
consent. Hence all the prerequisites for theft are ful lled.
FeedBack Bookmark

https://www.aspiration.ai/LAW/sis/Solution.jsp?qsetId=JAmrn2p8H1A=&qsetName=LST Mock 27 2021 (CLAT) 125/223


1/17/2021 Mock Analysis

Directions for questions 67 to 105: You have been given some passages followed by questions based on each
passage. You are required to choose the most appropriate option which follows from the passage. Only the
information given in the passage should be used for choosing the answer and no external knowledge of law
howsoever prominent is to be applied.

Passage – 3

Section 378 of the IPC de nes theft as, "Whoever, intending to take dishonestly any movable property out of the
possession of any person without that person's consent, moves that property to such taking, is said to commit
theft". We can say that section 378 of the IPC de nes 'theft' as the dishonest removal of moveable property 'out
of the possession of any person' without the consent of that person. The Section further provides in the
explanations given that any object attached to the earth would be considered as an immovable property hence it
could not be a subject of theft but once it is removed from the earth it would become a movable property and
could be stolen. The consent that must be attained for a property to be taken without it being considered theft
may either be express or implied.

Prerequisite for Theft-

Dishonest intention- Section 24 of IPC provides that dishonesty means the intention of wrongful gain or
wrongful loss. Section 23 of the IPC provides the de nition of both wrongful gain and wrongful loss. Wrongful
gain means gaining any property unlawfully, the person who is losing the property is the legal owner of such
property. Wrongful loss means the loss brought about by unlawful means.

Movable property- Section 22 of IPC has provided the de nition of movable property; means that any corporeal
property except land and things permanently attached to the earth. Only movable property can be stolen as it is
impossible to take immovable property away. Immovable property can be converted into movable property and
once it has been converted such property can be stolen.

Property in possession- In order for theft to have occurred, the property being stolen must be taken from the
possession from the owner of such a property. If the property does not have an owner then such property
cannot be said to have been stolen if a person acquires such property. For example; if A nds a gold nugget in a
stream and he takes the gold home, it cannot be considered theft as the gold nugget has no owner.

No consent- The property that is in question must have been taken without the consent of the owner of such a
property. The consent can either be implied or expressed. The consent given must also be free meaning that
such consent must not be acquired through means of coercion or fear of injury or misrepresentation of facts.
Consent, given during state of drunkenness or intoxication as well as consent given by a person of unsound
mind cannot be considered to be a valid consent.

Punishment for theft- Section 379 of the IPC establishes the punishment for theft as imprisonment up to 3
years or ne or both. Other succeeding provisions contain more severe punishment for theft under aggravating
circumstances. For example, Section 380 of IPC punishes the commission of theft in a building, tent or vessel
used for dwelling or residence. The object of this provision is to provide greater security to properties in dwelling
premises. The punishment for this is imprisonment up to 7 years along with ne.

Q.78 [30465398]
Z visited a library in which his friend X was the librarian. Z borrowed a book from the library without the
permission of the librarian hoping to mention it to X later. Z was charged with theft. Decide.

https://www.aspiration.ai/LAW/sis/Solution.jsp?qsetId=JAmrn2p8H1A=&qsetName=LST Mock 27 2021 (CLAT) 126/223


1/17/2021 Mock Analysis

a Z shall be liable for theft as he took the book out of the possession of the concerned person without his
consent.

b Z shall be liable for theft as there was dishonest intention to steal the book and blame it on a friend

c Z shall not be liable for theft as there was no dishonest intention to steal the book, but borrowed it in a good
faith and with a mindset to inform the concerned person.

d Z shall be held liable for theft irrespective of his state of mind as he moved the book without explicit or
implied consent.

Solution:
 Answer key/Solution
Correct Answer : c
Your Answer : c
In the given case Z took the book assuming an implied consent. The dishonest
intention to steal the book was absent. Hence, Z shall not be held liable for theft.
FeedBack Bookmark

https://www.aspiration.ai/LAW/sis/Solution.jsp?qsetId=JAmrn2p8H1A=&qsetName=LST Mock 27 2021 (CLAT) 127/223


1/17/2021 Mock Analysis

Directions for questions 67 to 105: You have been given some passages followed by questions based on each
passage. You are required to choose the most appropriate option which follows from the passage. Only the
information given in the passage should be used for choosing the answer and no external knowledge of law
howsoever prominent is to be applied.

Passage – 3

Section 378 of the IPC de nes theft as, "Whoever, intending to take dishonestly any movable property out of the
possession of any person without that person's consent, moves that property to such taking, is said to commit
theft". We can say that section 378 of the IPC de nes 'theft' as the dishonest removal of moveable property 'out
of the possession of any person' without the consent of that person. The Section further provides in the
explanations given that any object attached to the earth would be considered as an immovable property hence it
could not be a subject of theft but once it is removed from the earth it would become a movable property and
could be stolen. The consent that must be attained for a property to be taken without it being considered theft
may either be express or implied.

Prerequisite for Theft-

Dishonest intention- Section 24 of IPC provides that dishonesty means the intention of wrongful gain or
wrongful loss. Section 23 of the IPC provides the de nition of both wrongful gain and wrongful loss. Wrongful
gain means gaining any property unlawfully, the person who is losing the property is the legal owner of such
property. Wrongful loss means the loss brought about by unlawful means.

Movable property- Section 22 of IPC has provided the de nition of movable property; means that any corporeal
property except land and things permanently attached to the earth. Only movable property can be stolen as it is
impossible to take immovable property away. Immovable property can be converted into movable property and
once it has been converted such property can be stolen.

Property in possession- In order for theft to have occurred, the property being stolen must be taken from the
possession from the owner of such a property. If the property does not have an owner then such property
cannot be said to have been stolen if a person acquires such property. For example; if A nds a gold nugget in a
stream and he takes the gold home, it cannot be considered theft as the gold nugget has no owner.

No consent- The property that is in question must have been taken without the consent of the owner of such a
property. The consent can either be implied or expressed. The consent given must also be free meaning that
such consent must not be acquired through means of coercion or fear of injury or misrepresentation of facts.
Consent, given during state of drunkenness or intoxication as well as consent given by a person of unsound
mind cannot be considered to be a valid consent.

Punishment for theft- Section 379 of the IPC establishes the punishment for theft as imprisonment up to 3
years or ne or both. Other succeeding provisions contain more severe punishment for theft under aggravating
circumstances. For example, Section 380 of IPC punishes the commission of theft in a building, tent or vessel
used for dwelling or residence. The object of this provision is to provide greater security to properties in dwelling
premises. The punishment for this is imprisonment up to 7 years along with ne.

Q.79 [30465398]
Y a furniture maker wanted to buy an oak tree which was in the land of Z, a farmer, Z refused to sell the tree. Y
surreptitiously cut down the tree at night and moved it. Can Y be held liable for theft?

https://www.aspiration.ai/LAW/sis/Solution.jsp?qsetId=JAmrn2p8H1A=&qsetName=LST Mock 27 2021 (CLAT) 128/223


1/17/2021 Mock Analysis

a Y cannot be held liable for theft as a tree is an immovable property and theft can be committed only with
movable property.

b Y cannot be held liable for theft as he had already asked permission denying which Z should have expected
this.

c Y can be held liable for theft from the movement he decides to cut down the tree.

d As soon as the tree is removed from the earth it becomes a movable property, hence Y shall be held liable
for theft.

Solution:
 Answer key/Solution
Correct Answer : d
Your Answer : d
1st paragraph of the passage explains "any object attached to the earth would be
considered as an immovable property hence it could not be a subject of theft but once it is removed from the
earth it would become a movable property and could be stolen. Keeping this in mind Y can be held liable for
theft.
FeedBack Bookmark

https://www.aspiration.ai/LAW/sis/Solution.jsp?qsetId=JAmrn2p8H1A=&qsetName=LST Mock 27 2021 (CLAT) 129/223


1/17/2021 Mock Analysis

Directions for questions 67 to 105: You have been given some passages followed by questions based on each
passage. You are required to choose the most appropriate option which follows from the passage. Only the
information given in the passage should be used for choosing the answer and no external knowledge of law
howsoever prominent is to be applied.

Passage – 3

Section 378 of the IPC de nes theft as, "Whoever, intending to take dishonestly any movable property out of the
possession of any person without that person's consent, moves that property to such taking, is said to commit
theft". We can say that section 378 of the IPC de nes 'theft' as the dishonest removal of moveable property 'out
of the possession of any person' without the consent of that person. The Section further provides in the
explanations given that any object attached to the earth would be considered as an immovable property hence it
could not be a subject of theft but once it is removed from the earth it would become a movable property and
could be stolen. The consent that must be attained for a property to be taken without it being considered theft
may either be express or implied.

Prerequisite for Theft-

Dishonest intention- Section 24 of IPC provides that dishonesty means the intention of wrongful gain or
wrongful loss. Section 23 of the IPC provides the de nition of both wrongful gain and wrongful loss. Wrongful
gain means gaining any property unlawfully, the person who is losing the property is the legal owner of such
property. Wrongful loss means the loss brought about by unlawful means.

Movable property- Section 22 of IPC has provided the de nition of movable property; means that any corporeal
property except land and things permanently attached to the earth. Only movable property can be stolen as it is
impossible to take immovable property away. Immovable property can be converted into movable property and
once it has been converted such property can be stolen.

Property in possession- In order for theft to have occurred, the property being stolen must be taken from the
possession from the owner of such a property. If the property does not have an owner then such property
cannot be said to have been stolen if a person acquires such property. For example; if A nds a gold nugget in a
stream and he takes the gold home, it cannot be considered theft as the gold nugget has no owner.

No consent- The property that is in question must have been taken without the consent of the owner of such a
property. The consent can either be implied or expressed. The consent given must also be free meaning that
such consent must not be acquired through means of coercion or fear of injury or misrepresentation of facts.
Consent, given during state of drunkenness or intoxication as well as consent given by a person of unsound
mind cannot be considered to be a valid consent.

Punishment for theft- Section 379 of the IPC establishes the punishment for theft as imprisonment up to 3
years or ne or both. Other succeeding provisions contain more severe punishment for theft under aggravating
circumstances. For example, Section 380 of IPC punishes the commission of theft in a building, tent or vessel
used for dwelling or residence. The object of this provision is to provide greater security to properties in dwelling
premises. The punishment for this is imprisonment up to 7 years along with ne.

Q.80 [30465398]
G planned to cut down H's tree and take it away. One-night G decided to act upon his plan. G cut H's tree to the
ground but before loading it to the truck was caught by H's guards. Can G be held liable for theft?

https://www.aspiration.ai/LAW/sis/Solution.jsp?qsetId=JAmrn2p8H1A=&qsetName=LST Mock 27 2021 (CLAT) 130/223


1/17/2021 Mock Analysis

a G can be held liable for theft as from the movement the tree has been severed to the ground it has been
moved from its original position.

b G cannot be held liable as he just cut down the tree but did not move it or take it out of the possession of H.

c G cannot be held liable for theft as the tree was not taken out of the possession of H.

d G can be held liable as the presence of a truck to load the tree is su cient evidence to prove the intention
of theft.

Solution:
 Answer key/Solution
Correct Answer : a
Your Answer : a
From the movement the tree is severed it is a movable property. Once the tree is
cut from the original position it means that it has been taken out of the possession of the owner. Hence Y shall
be held liable for theft.
FeedBack Bookmark

https://www.aspiration.ai/LAW/sis/Solution.jsp?qsetId=JAmrn2p8H1A=&qsetName=LST Mock 27 2021 (CLAT) 131/223


1/17/2021 Mock Analysis

Directions for questions 67 to 105: You have been given some passages followed by questions based on each
passage. You are required to choose the most appropriate option which follows from the passage. Only the
information given in the passage should be used for choosing the answer and no external knowledge of law
howsoever prominent is to be applied.

Passage – 3

Section 378 of the IPC de nes theft as, "Whoever, intending to take dishonestly any movable property out of the
possession of any person without that person's consent, moves that property to such taking, is said to commit
theft". We can say that section 378 of the IPC de nes 'theft' as the dishonest removal of moveable property 'out
of the possession of any person' without the consent of that person. The Section further provides in the
explanations given that any object attached to the earth would be considered as an immovable property hence it
could not be a subject of theft but once it is removed from the earth it would become a movable property and
could be stolen. The consent that must be attained for a property to be taken without it being considered theft
may either be express or implied.

Prerequisite for Theft-

Dishonest intention- Section 24 of IPC provides that dishonesty means the intention of wrongful gain or
wrongful loss. Section 23 of the IPC provides the de nition of both wrongful gain and wrongful loss. Wrongful
gain means gaining any property unlawfully, the person who is losing the property is the legal owner of such
property. Wrongful loss means the loss brought about by unlawful means.

Movable property- Section 22 of IPC has provided the de nition of movable property; means that any corporeal
property except land and things permanently attached to the earth. Only movable property can be stolen as it is
impossible to take immovable property away. Immovable property can be converted into movable property and
once it has been converted such property can be stolen.

Property in possession- In order for theft to have occurred, the property being stolen must be taken from the
possession from the owner of such a property. If the property does not have an owner then such property
cannot be said to have been stolen if a person acquires such property. For example; if A nds a gold nugget in a
stream and he takes the gold home, it cannot be considered theft as the gold nugget has no owner.

No consent- The property that is in question must have been taken without the consent of the owner of such a
property. The consent can either be implied or expressed. The consent given must also be free meaning that
such consent must not be acquired through means of coercion or fear of injury or misrepresentation of facts.
Consent, given during state of drunkenness or intoxication as well as consent given by a person of unsound
mind cannot be considered to be a valid consent.

Punishment for theft- Section 379 of the IPC establishes the punishment for theft as imprisonment up to 3
years or ne or both. Other succeeding provisions contain more severe punishment for theft under aggravating
circumstances. For example, Section 380 of IPC punishes the commission of theft in a building, tent or vessel
used for dwelling or residence. The object of this provision is to provide greater security to properties in dwelling
premises. The punishment for this is imprisonment up to 7 years along with ne.

Q.81 [30465398]
A gave his watch to Z for repair. After 3 days, A came back and took the watch out of Z's possession without his
consent or paying him the money that was due for repair. Is A guilty of theft?

https://www.aspiration.ai/LAW/sis/Solution.jsp?qsetId=JAmrn2p8H1A=&qsetName=LST Mock 27 2021 (CLAT) 132/223


1/17/2021 Mock Analysis

a A is not guilty as he took back his own watch, hence it is not theft.

b A is guilty of committing theft.

c A is not guilty as it is understood that the payment will be done later.

d A is guilty of criminal misappropriation of property.

Solution:
 Answer key/Solution
Correct Answer : b
Your Answer : b
In the given case A had to pay the money that was due failing which he has no
right to take away the property. As soon as he takes it away without paying the money and without Z's consent
it becomes a theft.

FeedBack Bookmark

Directions for questions 67 to 105: You have been given some passages followed by questions based on each
passage. You are required to choose the most appropriate option which follows from the passage. Only the
information given in the passage should be used for choosing the answer and no external knowledge of law
howsoever prominent is to be applied.

Passage – 4

In the law of torts, if any person commits any wrongful act which causes injury to another person, he is held
liable and has to pay damages or provide some other remedy which the Court determines, to the victim of such
an act. But in some cases even if a person suffers some loss because of the act of another person, he cannot
claim damages from that person because of the operation of defences of tort. One such defence available to a
defendant is the defence of volenti non t injuria in which the plaintiff is not entitled to damages because he
consents to the act which has caused injury to him.

In the law of torts, there is a duty on every person to act with reasonable care in order to avoid any harm which
may occur due to their failure of taking such care. For e.g -If a person is driving his car, he has a duty to drive the
car safely and within speed limits so that no accident occurs which can also harm any other person. This is the
general rule in torts but there are certain exceptions which are allowed in these cases and these called as
defences to tort. Under these defences, a defendant can escape liability and volenti non- t injuria is also one
such defence which is available for the defendant. In case a person gives his consent to doing an act which
leads to him getting injured, then even if an injury is caused by the other person, he cannot claim any damages
from that person because the act was one for which he voluntarily consented. The consent of the plaintiff acts
as a defence and this defence is called volenti non t injuria which means to a willing person no injury happens.

For the application of the defence of volenti non t injuria there are some essential elements or conditions
which should be present in a case and only when they are ful lled, this defence can be taken to prevent liability.

https://www.aspiration.ai/LAW/sis/Solution.jsp?qsetId=JAmrn2p8H1A=&qsetName=LST Mock 27 2021 (CLAT) 133/223


1/17/2021 Mock Analysis

There are 2 essential elements in this defence:


1. The plaintiff has the knowledge of the risk
2. The plaintiff with the knowledge of risk has voluntarily agreed to suffer the harm.

Thus, whenever the plaintiff is aware of the possibility of harm which is likely to be caused by an act and when
he still accepts to do that act and therefore agrees to suffer the injury, a defendant is relieved of his liability.

Burden of proof- In the cases where the defendant is taking the defence of volenti non t injuria, the burden of
proof is on him to show that the plaintiff had full knowledge of the act and he had consented to the risk involved
in the act and the defendant has to show that the plaintiff was also aware of the extent of risk which was
involved in the act for successfully taking this defence.

The consent of the plaintiff is very important in the defence of volenti non t injuria because only when he
voluntarily gives his consent to an act, the defendant can take this defence. In the cases of this defence, the
consent of the defendant is not required to be expressly given and even by his conduct, his consent can be
taken. When a plaintiff gives his consent for an act such consent should be free from any coercion, fraud or any
other such means by which the free consent can be affected. In cases of consent having been obtained by
fraud, the defence of volenti non t injuria will not apply and the defendant will be held liable for the wrong by
him. There are certain limitations under which the defence of volenti non t injuria cannot be taken by a
defendant even if the essentials of this defence are present in the case. They are rescue cases, illegal acts,
negligence of the defendant etc.

Q.82 [30465398]
A was a singing coach who convinced a 16year-old student to have sexual intercourse with him by telling her
that it will help her in improving her voice and singing. While he was charged for the same, he took the defence
of volenti non t injuria. Decide.

a A shall not be held liable as a 16year old girl is mature enough to give her consent for any act she
undertakes.

b A shall be held liable as he fraudulently convinced a 16year old under the pretence of improving her voice
and singing.

c A shall not be held liable as he did the act in a good faith to help the girl to improve her singing and voice.

d A shall be held liable as he had to take permission of the girl's parents before committing the act.

Solution:
 Answer key/Solution
Correct Answer : b
Your Answer : b
The defence of 'volenti non t injuria' is not considered for a fraudulent act. In this
case A committed a fraud by convincing her to improve her voice and singing in order to have sex with her.
Hence, option A is the correct answer.

FeedBack Bookmark

https://www.aspiration.ai/LAW/sis/Solution.jsp?qsetId=JAmrn2p8H1A=&qsetName=LST Mock 27 2021 (CLAT) 134/223


1/17/2021 Mock Analysis

Directions for questions 67 to 105: You have been given some passages followed by questions based on each
passage. You are required to choose the most appropriate option which follows from the passage. Only the
information given in the passage should be used for choosing the answer and no external knowledge of law
howsoever prominent is to be applied.

Passage – 4

In the law of torts, if any person commits any wrongful act which causes injury to another person, he is held
liable and has to pay damages or provide some other remedy which the Court determines, to the victim of such
an act. But in some cases even if a person suffers some loss because of the act of another person, he cannot
claim damages from that person because of the operation of defences of tort. One such defence available to a
defendant is the defence of volenti non t injuria in which the plaintiff is not entitled to damages because he
consents to the act which has caused injury to him.

In the law of torts, there is a duty on every person to act with reasonable care in order to avoid any harm which
may occur due to their failure of taking such care. For e.g -If a person is driving his car, he has a duty to drive the
car safely and within speed limits so that no accident occurs which can also harm any other person. This is the
general rule in torts but there are certain exceptions which are allowed in these cases and these called as
defences to tort. Under these defences, a defendant can escape liability and volenti non- t injuria is also one
such defence which is available for the defendant. In case a person gives his consent to doing an act which
leads to him getting injured, then even if an injury is caused by the other person, he cannot claim any damages
from that person because the act was one for which he voluntarily consented. The consent of the plaintiff acts
as a defence and this defence is called volenti non t injuria which means to a willing person no injury happens.

For the application of the defence of volenti non t injuria there are some essential elements or conditions
which should be present in a case and only when they are ful lled, this defence can be taken to prevent liability.

There are 2 essential elements in this defence:


1. The plaintiff has the knowledge of the risk
2. The plaintiff with the knowledge of risk has voluntarily agreed to suffer the harm.

Thus, whenever the plaintiff is aware of the possibility of harm which is likely to be caused by an act and when
he still accepts to do that act and therefore agrees to suffer the injury, a defendant is relieved of his liability.

Burden of proof- In the cases where the defendant is taking the defence of volenti non t injuria, the burden of
proof is on him to show that the plaintiff had full knowledge of the act and he had consented to the risk involved
in the act and the defendant has to show that the plaintiff was also aware of the extent of risk which was
involved in the act for successfully taking this defence.

The consent of the plaintiff is very important in the defence of volenti non t injuria because only when he
voluntarily gives his consent to an act, the defendant can take this defence. In the cases of this defence, the
consent of the defendant is not required to be expressly given and even by his conduct, his consent can be
taken. When a plaintiff gives his consent for an act such consent should be free from any coercion, fraud or any
other such means by which the free consent can be affected. In cases of consent having been obtained by
fraud, the defence of volenti non t injuria will not apply and the defendant will be held liable for the wrong by
him. There are certain limitations under which the defence of volenti non t injuria cannot be taken by a

https://www.aspiration.ai/LAW/sis/Solution.jsp?qsetId=JAmrn2p8H1A=&qsetName=LST Mock 27 2021 (CLAT) 135/223


1/17/2021 Mock Analysis

defendant even if the essentials of this defence are present in the case. They are rescue cases, illegal acts,
negligence of the defendant etc.

Q.83 [30465398]
A and B were friends. A has a bike whose brakes do not work and B knowing about the conditions of the bike
still chooses to sit on it with A driving. There was an accident and A got injured. A sued B. Decide.

a B shall be held liable as A gave the consent to sit behind the bike and not to get hurt by accident.

b B shall be held liable as he should have warned A and not asked him to sit with him.

c B shall not be held liable as A had the knowledge of the risk and still sitting implies his acceptance to
undergo that risk.

d B shall not be held liable as B himself was injured along with A in the accident.

Solution:
 Answer key/Solution
Correct Answer : c
Your Answer : a
In the given case. The defence of 'volenti non t injuria' applies. After knowing
that brakes were faulty, A sat along with B which shows the willingness to undergo that risk. Hence, option c is
the correct answer.

FeedBack Bookmark

Directions for questions 67 to 105: You have been given some passages followed by questions based on each
passage. You are required to choose the most appropriate option which follows from the passage. Only the
information given in the passage should be used for choosing the answer and no external knowledge of law
howsoever prominent is to be applied.

Passage – 4

In the law of torts, if any person commits any wrongful act which causes injury to another person, he is held
liable and has to pay damages or provide some other remedy which the Court determines, to the victim of such
an act. But in some cases even if a person suffers some loss because of the act of another person, he cannot
claim damages from that person because of the operation of defences of tort. One such defence available to a
defendant is the defence of volenti non t injuria in which the plaintiff is not entitled to damages because he
consents to the act which has caused injury to him.

In the law of torts, there is a duty on every person to act with reasonable care in order to avoid any harm which
may occur due to their failure of taking such care. For e.g -If a person is driving his car, he has a duty to drive the
car safely and within speed limits so that no accident occurs which can also harm any other person. This is the
general rule in torts but there are certain exceptions which are allowed in these cases and these called as
defences to tort. Under these defences, a defendant can escape liability and volenti non- t injuria is also one
such defence which is available for the defendant. In case a person gives his consent to doing an act which

https://www.aspiration.ai/LAW/sis/Solution.jsp?qsetId=JAmrn2p8H1A=&qsetName=LST Mock 27 2021 (CLAT) 136/223


1/17/2021 Mock Analysis

leads to him getting injured, then even if an injury is caused by the other person, he cannot claim any damages
from that person because the act was one for which he voluntarily consented. The consent of the plaintiff acts
as a defence and this defence is called volenti non t injuria which means to a willing person no injury happens.

For the application of the defence of volenti non t injuria there are some essential elements or conditions
which should be present in a case and only when they are ful lled, this defence can be taken to prevent liability.

There are 2 essential elements in this defence:


1. The plaintiff has the knowledge of the risk
2. The plaintiff with the knowledge of risk has voluntarily agreed to suffer the harm.

Thus, whenever the plaintiff is aware of the possibility of harm which is likely to be caused by an act and when
he still accepts to do that act and therefore agrees to suffer the injury, a defendant is relieved of his liability.

Burden of proof- In the cases where the defendant is taking the defence of volenti non t injuria, the burden of
proof is on him to show that the plaintiff had full knowledge of the act and he had consented to the risk involved
in the act and the defendant has to show that the plaintiff was also aware of the extent of risk which was
involved in the act for successfully taking this defence.

The consent of the plaintiff is very important in the defence of volenti non t injuria because only when he
voluntarily gives his consent to an act, the defendant can take this defence. In the cases of this defence, the
consent of the defendant is not required to be expressly given and even by his conduct, his consent can be
taken. When a plaintiff gives his consent for an act such consent should be free from any coercion, fraud or any
other such means by which the free consent can be affected. In cases of consent having been obtained by
fraud, the defence of volenti non t injuria will not apply and the defendant will be held liable for the wrong by
him. There are certain limitations under which the defence of volenti non t injuria cannot be taken by a
defendant even if the essentials of this defence are present in the case. They are rescue cases, illegal acts,
negligence of the defendant etc.

Q.84 [30465398]
A was walking alongside Rangaswami stadium. A knew a cricket match was going on inside the stadium. Chris
Wale, the batsman hit a six which came out of the stadium and hit A's head. That was the biggest six in the
history of cricket. A led a case. Can the defence of volenti non t injuria' taken by the defendants?

a Defence of 'volenti non t injuria' can be taken as by walking alongside the stadium A had consented for
the risk of being hit.

b Defence of 'volenti non t injuria' can be taken as A had the knowledge of cricket match inside stadium
which shows he had consented for the risk/

c Defence of 'volenti non t injuria' cannot be taken by the defendants.

d The defendants shall not be liable as it is an act of god.

https://www.aspiration.ai/LAW/sis/Solution.jsp?qsetId=JAmrn2p8H1A=&qsetName=LST Mock 27 2021 (CLAT) 137/223


1/17/2021 Mock Analysis

Solution:
 Answer key/Solution
Correct Answer : c
Your Answer : c
In the given case A was neither inside the stadium nor had he accepted the risk of
being hit. Mere knowledge of a cricket match inside the stadium does not mean A had accepted the risk. The
ball hitting A was unexpected and unprecedented as it was the biggest six in the history of the game. Hence
the defence will not be valid.
FeedBack Bookmark

https://www.aspiration.ai/LAW/sis/Solution.jsp?qsetId=JAmrn2p8H1A=&qsetName=LST Mock 27 2021 (CLAT) 138/223


1/17/2021 Mock Analysis

Directions for questions 67 to 105: You have been given some passages followed by questions based on each
passage. You are required to choose the most appropriate option which follows from the passage. Only the
information given in the passage should be used for choosing the answer and no external knowledge of law
howsoever prominent is to be applied.

Passage – 4

In the law of torts, if any person commits any wrongful act which causes injury to another person, he is held
liable and has to pay damages or provide some other remedy which the Court determines, to the victim of such
an act. But in some cases even if a person suffers some loss because of the act of another person, he cannot
claim damages from that person because of the operation of defences of tort. One such defence available to a
defendant is the defence of volenti non t injuria in which the plaintiff is not entitled to damages because he
consents to the act which has caused injury to him.

In the law of torts, there is a duty on every person to act with reasonable care in order to avoid any harm which
may occur due to their failure of taking such care. For e.g -If a person is driving his car, he has a duty to drive the
car safely and within speed limits so that no accident occurs which can also harm any other person. This is the
general rule in torts but there are certain exceptions which are allowed in these cases and these called as
defences to tort. Under these defences, a defendant can escape liability and volenti non- t injuria is also one
such defence which is available for the defendant. In case a person gives his consent to doing an act which
leads to him getting injured, then even if an injury is caused by the other person, he cannot claim any damages
from that person because the act was one for which he voluntarily consented. The consent of the plaintiff acts
as a defence and this defence is called volenti non t injuria which means to a willing person no injury happens.

For the application of the defence of volenti non t injuria there are some essential elements or conditions
which should be present in a case and only when they are ful lled, this defence can be taken to prevent liability.

There are 2 essential elements in this defence:


1. The plaintiff has the knowledge of the risk
2. The plaintiff with the knowledge of risk has voluntarily agreed to suffer the harm.

Thus, whenever the plaintiff is aware of the possibility of harm which is likely to be caused by an act and when
he still accepts to do that act and therefore agrees to suffer the injury, a defendant is relieved of his liability.

Burden of proof- In the cases where the defendant is taking the defence of volenti non t injuria, the burden of
proof is on him to show that the plaintiff had full knowledge of the act and he had consented to the risk involved
in the act and the defendant has to show that the plaintiff was also aware of the extent of risk which was
involved in the act for successfully taking this defence.

The consent of the plaintiff is very important in the defence of volenti non t injuria because only when he
voluntarily gives his consent to an act, the defendant can take this defence. In the cases of this defence, the
consent of the defendant is not required to be expressly given and even by his conduct, his consent can be
taken. When a plaintiff gives his consent for an act such consent should be free from any coercion, fraud or any
other such means by which the free consent can be affected. In cases of consent having been obtained by
fraud, the defence of volenti non t injuria will not apply and the defendant will be held liable for the wrong by
him. There are certain limitations under which the defence of volenti non t injuria cannot be taken by a
defendant even if the essentials of this defence are present in the case. They are rescue cases, illegal acts,
negligence of the defendant etc.

https://www.aspiration.ai/LAW/sis/Solution.jsp?qsetId=JAmrn2p8H1A=&qsetName=LST Mock 27 2021 (CLAT) 139/223


1/17/2021 Mock Analysis

Q.85 [30465398]
Which of the following is not an example for implied consent of 'volenti non t injuria'?

a S enters into a boxing match with K.

b A sit's dangerously near the formula one race track to witness his favourite racer Miguel Shoemaker.

c A sign a contract expressing his acceptance to undergo economic risk before buying mutual funds.

d C takes up cold bath challenge in -15 degrees in order to appear in 'Wadia's got talent'.

Solution:
 Answer key/Solution
Correct Answer : c
Your Answer : c
In all cases except option c the consent for risk is implied.

FeedBack Bookmark

Directions for questions 67 to 105: You have been given some passages followed by questions based on each
passage. You are required to choose the most appropriate option which follows from the passage. Only the
information given in the passage should be used for choosing the answer and no external knowledge of law
howsoever prominent is to be applied.

Passage – 4

In the law of torts, if any person commits any wrongful act which causes injury to another person, he is held
liable and has to pay damages or provide some other remedy which the Court determines, to the victim of such
an act. But in some cases even if a person suffers some loss because of the act of another person, he cannot
claim damages from that person because of the operation of defences of tort. One such defence available to a
defendant is the defence of volenti non t injuria in which the plaintiff is not entitled to damages because he
consents to the act which has caused injury to him.

In the law of torts, there is a duty on every person to act with reasonable care in order to avoid any harm which
may occur due to their failure of taking such care. For e.g -If a person is driving his car, he has a duty to drive the
car safely and within speed limits so that no accident occurs which can also harm any other person. This is the
general rule in torts but there are certain exceptions which are allowed in these cases and these called as
defences to tort. Under these defences, a defendant can escape liability and volenti non- t injuria is also one
such defence which is available for the defendant. In case a person gives his consent to doing an act which
leads to him getting injured, then even if an injury is caused by the other person, he cannot claim any damages
from that person because the act was one for which he voluntarily consented. The consent of the plaintiff acts
as a defence and this defence is called volenti non t injuria which means to a willing person no injury happens.

For the application of the defence of volenti non t injuria there are some essential elements or conditions
which should be present in a case and only when they are ful lled, this defence can be taken to prevent liability.

https://www.aspiration.ai/LAW/sis/Solution.jsp?qsetId=JAmrn2p8H1A=&qsetName=LST Mock 27 2021 (CLAT) 140/223


1/17/2021 Mock Analysis

There are 2 essential elements in this defence:


1. The plaintiff has the knowledge of the risk
2. The plaintiff with the knowledge of risk has voluntarily agreed to suffer the harm.

Thus, whenever the plaintiff is aware of the possibility of harm which is likely to be caused by an act and when
he still accepts to do that act and therefore agrees to suffer the injury, a defendant is relieved of his liability.

Burden of proof- In the cases where the defendant is taking the defence of volenti non t injuria, the burden of
proof is on him to show that the plaintiff had full knowledge of the act and he had consented to the risk involved
in the act and the defendant has to show that the plaintiff was also aware of the extent of risk which was
involved in the act for successfully taking this defence.

The consent of the plaintiff is very important in the defence of volenti non t injuria because only when he
voluntarily gives his consent to an act, the defendant can take this defence. In the cases of this defence, the
consent of the defendant is not required to be expressly given and even by his conduct, his consent can be
taken. When a plaintiff gives his consent for an act such consent should be free from any coercion, fraud or any
other such means by which the free consent can be affected. In cases of consent having been obtained by
fraud, the defence of volenti non t injuria will not apply and the defendant will be held liable for the wrong by
him. There are certain limitations under which the defence of volenti non t injuria cannot be taken by a
defendant even if the essentials of this defence are present in the case. They are rescue cases, illegal acts,
negligence of the defendant etc.

Q.86 [30465398]
A, B and C are best friends who go hiking in 'Mount Ever-rest'. The mountain was 5000 metres in hight and had
extremely dangerous terrain. A decided to go after a staunch persuasion from B and C. While hiking A fell into a
gorge and broke his legs. He led a suit. The defendants pleaded 'volenti non t injuria'. Who has the burden of
proof in the given case?

a Burden of proof is on A to show that he had not consented for the injuries and risks.

b Burden of proof is on C as he was the one who persuaded others to go on hiking.

c Burden of proof is on the state as they negligently allowed the individuals to go hikingin dangerous terrain.

d B and C the defendants in this case share the burden of proof to show that A consented for the risk.

Solution:
 Answer key/Solution
Correct Answer : d
Your Answer : d
It is given in the passage that the burden of proof lies on the defendants. In the
given case B and C are the defendants and hence they have the burden of proof.

FeedBack Bookmark

https://www.aspiration.ai/LAW/sis/Solution.jsp?qsetId=JAmrn2p8H1A=&qsetName=LST Mock 27 2021 (CLAT) 141/223


1/17/2021 Mock Analysis

Directions for questions 67 to 105: You have been given some passages followed by questions based on each
passage. You are required to choose the most appropriate option which follows from the passage. Only the
information given in the passage should be used for choosing the answer and no external knowledge of law
howsoever prominent is to be applied.

Passage – 5

According to Section 2(f) of Domestic violence Act 2005, "domestic relationship" means a relationship between
two persons living in a shared household. Domestic relationships can be through marriage such as wives,
daughtersin- law, sisters-in-law, widows and any other members of the family; or blood relationships such as
mothers, sisters or daughters; and other domestic relationships including through adoption, live-in relationships,
and women in bigamous relationships or victims of legally invalid marriages. The law addresses the concerns
of women of all ages irrespective of their marital status. The de nition of "domestic relationship" under the
Domestic Violence Act is exhaustive: when a de nition clause is de ned to "mean" such and such, the de nition
is prima facie restrictive and exhaustive. The Supreme Court has further stated that the word domestic
relationship means a relationship that has some inherent or essential characteristics of marriage though not a
marriage that is legally recognized. Expression "relationship in the nature of marriage" cannot be construed in
the abstract. It is to be taken in the context in which it appears and to be applied bearing in mind the purpose
and object of DV Act as well as meaning of the expression "in the nature of marriage", (Indra Sarma v. V.K.V
Sarma, (2013))

Shared household- According to Section 2(s)of DV Act 2005, a shared household is where the aggrieved person
or a woman lives in a domestic relationship, either singly, or along with the man against whom the complaint is
led. It may also imply a household where a woman has lived in a domestic relationship but has been thrown
out. This may include all kinds of situations whether the household is owned by the respondent or it is rented
accommodation. It also includes a house either owned jointly by the aggrieved person and the respondent or
both may have jointly or singly, any rights, titles or interests. The DV Act recognizes a woman's right to reside in
a shared household. This means a woman cannot be thrown out of such a household except through the
procedure established by the law. In case she is thrown out she can be brought back again after obtaining the
order from the court. A woman to claim the protection of right in "shared household" has to establish
(a) that the relationship with the opposite party is "domestic relationship", and
(b) that the house in respect of which she seeks to enforce the right is "shared household".

Domestic violence- "Domestic violence" is a broad term that entails not only physical beating but also other
forms of violence such as emotional violence, mental violence, sexual violence, nancial violence and other
forms of cruelty that may occur within a household. The de nition provided in Section 3 of the DV Act includes
the following as acts of domestic violence. The Section also de nes the meaning of terms physical abuse,
sexual abuse, verbal and emotional abuse, and economic abuse. It further enunciates that the overall facts and
circumstances of the case shall be taken into consideration in order to determine whether any act, omission,
commission or conduct of the respondent constitutes "domestic violence" under the said section.

Q.87 [30465398]
A used to live along with her husband B, in her matrimonial home. A was being harassed by her husband and her
in laws. One-day, she was thrown out of the house alleging her of committing adultery. During this period, A
moved out of the house (which was registered under his parents, name) and settled at a new place. A brought
action against B for the right to shared household to the old home where he used to say. A contended that he no
longer stayed and that was not the shared household.

https://www.aspiration.ai/LAW/sis/Solution.jsp?qsetId=JAmrn2p8H1A=&qsetName=LST Mock 27 2021 (CLAT) 142/223


1/17/2021 Mock Analysis

a A can successfully seek 'right to shared household' irrespective of whether A stayed there or not.

b A cannot seek 'right to shared household' as A no longer stayed there and had no right in that house.

c A cannot seek 'right to shared household' as she committed adultery and deserved the right punishment.

d A can successfully seek 'right to shared household' as she was thrown out of the house without her
consent and she has every right to live there.

Solution:
 Answer key/Solution
Correct Answer : b
Your Answer : d
In the given case keen observation of the facts is necessary. During the period A
was out of the house, her husband shifted to a different house. It is also said that the house was not registered
in his name. Therefore, the husband has no right on the house in turn the wife fails in her claim for shared
household. But she can claim the rightto a shared household for the new house where her husband has been
settled. Hence, option b is the correct answer.
FeedBack Bookmark

https://www.aspiration.ai/LAW/sis/Solution.jsp?qsetId=JAmrn2p8H1A=&qsetName=LST Mock 27 2021 (CLAT) 143/223


1/17/2021 Mock Analysis

Directions for questions 67 to 105: You have been given some passages followed by questions based on each
passage. You are required to choose the most appropriate option which follows from the passage. Only the
information given in the passage should be used for choosing the answer and no external knowledge of law
howsoever prominent is to be applied.

Passage – 5

According to Section 2(f) of Domestic violence Act 2005, "domestic relationship" means a relationship between
two persons living in a shared household. Domestic relationships can be through marriage such as wives,
daughtersin- law, sisters-in-law, widows and any other members of the family; or blood relationships such as
mothers, sisters or daughters; and other domestic relationships including through adoption, live-in relationships,
and women in bigamous relationships or victims of legally invalid marriages. The law addresses the concerns
of women of all ages irrespective of their marital status. The de nition of "domestic relationship" under the
Domestic Violence Act is exhaustive: when a de nition clause is de ned to "mean" such and such, the de nition
is prima facie restrictive and exhaustive. The Supreme Court has further stated that the word domestic
relationship means a relationship that has some inherent or essential characteristics of marriage though not a
marriage that is legally recognized. Expression "relationship in the nature of marriage" cannot be construed in
the abstract. It is to be taken in the context in which it appears and to be applied bearing in mind the purpose
and object of DV Act as well as meaning of the expression "in the nature of marriage", (Indra Sarma v. V.K.V
Sarma, (2013))

Shared household- According to Section 2(s)of DV Act 2005, a shared household is where the aggrieved person
or a woman lives in a domestic relationship, either singly, or along with the man against whom the complaint is
led. It may also imply a household where a woman has lived in a domestic relationship but has been thrown
out. This may include all kinds of situations whether the household is owned by the respondent or it is rented
accommodation. It also includes a house either owned jointly by the aggrieved person and the respondent or
both may have jointly or singly, any rights, titles or interests. The DV Act recognizes a woman's right to reside in
a shared household. This means a woman cannot be thrown out of such a household except through the
procedure established by the law. In case she is thrown out she can be brought back again after obtaining the
order from the court. A woman to claim the protection of right in "shared household" has to establish
(a) that the relationship with the opposite party is "domestic relationship", and
(b) that the house in respect of which she seeks to enforce the right is "shared household".

Domestic violence- "Domestic violence" is a broad term that entails not only physical beating but also other
forms of violence such as emotional violence, mental violence, sexual violence, nancial violence and other
forms of cruelty that may occur within a household. The de nition provided in Section 3 of the DV Act includes
the following as acts of domestic violence. The Section also de nes the meaning of terms physical abuse,
sexual abuse, verbal and emotional abuse, and economic abuse. It further enunciates that the overall facts and
circumstances of the case shall be taken into consideration in order to determine whether any act, omission,
commission or conduct of the respondent constitutes "domestic violence" under the said section.

Q.88 [30465398]
Rhea used to stay in her husband Suhant's house. Sushant had an adopted sister named Alia. Alia used to
harass and beat Rhea. Rhea led a case for domestic violence. Does the relationship of Rhea and Alia come
under domestic relationship?

a No, as Alia is an adopted daughter and not an inherent member of the family, she doesn't come under
domestic relationship.

https://www.aspiration.ai/LAW/sis/Solution.jsp?qsetId=JAmrn2p8H1A=&qsetName=LST Mock 27 2021 (CLAT) 144/223


1/17/2021 Mock Analysis

b Domestic relationship is the relationship between the husband, wife and the husband's parents. None of the
others are included.

c Yes, the relationship comes under domestic relationship.

d Can't say, It depends on the property rights shared between Sushant and Alia.

Solution:
 Answer key/Solution
Correct Answer : c
Your Answer : c
Taking reference of the passage- 'Domestic relationship can be through marriage
such as wives, daughters-in-law, sisters-in-law, widows and any other members of the family; or blood
relationship such as mothers, sisters or daughters; and other domestic relationships including through
adoption, live-in relationships, and women in bigamous relationship or victims of legally invalid marriages.
Thus, we can arrive at the answer.

FeedBack Bookmark

https://www.aspiration.ai/LAW/sis/Solution.jsp?qsetId=JAmrn2p8H1A=&qsetName=LST Mock 27 2021 (CLAT) 145/223


1/17/2021 Mock Analysis

Directions for questions 67 to 105: You have been given some passages followed by questions based on each
passage. You are required to choose the most appropriate option which follows from the passage. Only the
information given in the passage should be used for choosing the answer and no external knowledge of law
howsoever prominent is to be applied.

Passage – 5

According to Section 2(f) of Domestic violence Act 2005, "domestic relationship" means a relationship between
two persons living in a shared household. Domestic relationships can be through marriage such as wives,
daughtersin- law, sisters-in-law, widows and any other members of the family; or blood relationships such as
mothers, sisters or daughters; and other domestic relationships including through adoption, live-in relationships,
and women in bigamous relationships or victims of legally invalid marriages. The law addresses the concerns
of women of all ages irrespective of their marital status. The de nition of "domestic relationship" under the
Domestic Violence Act is exhaustive: when a de nition clause is de ned to "mean" such and such, the de nition
is prima facie restrictive and exhaustive. The Supreme Court has further stated that the word domestic
relationship means a relationship that has some inherent or essential characteristics of marriage though not a
marriage that is legally recognized. Expression "relationship in the nature of marriage" cannot be construed in
the abstract. It is to be taken in the context in which it appears and to be applied bearing in mind the purpose
and object of DV Act as well as meaning of the expression "in the nature of marriage", (Indra Sarma v. V.K.V
Sarma, (2013))

Shared household- According to Section 2(s)of DV Act 2005, a shared household is where the aggrieved person
or a woman lives in a domestic relationship, either singly, or along with the man against whom the complaint is
led. It may also imply a household where a woman has lived in a domestic relationship but has been thrown
out. This may include all kinds of situations whether the household is owned by the respondent or it is rented
accommodation. It also includes a house either owned jointly by the aggrieved person and the respondent or
both may have jointly or singly, any rights, titles or interests. The DV Act recognizes a woman's right to reside in
a shared household. This means a woman cannot be thrown out of such a household except through the
procedure established by the law. In case she is thrown out she can be brought back again after obtaining the
order from the court. A woman to claim the protection of right in "shared household" has to establish
(a) that the relationship with the opposite party is "domestic relationship", and
(b) that the house in respect of which she seeks to enforce the right is "shared household".

Domestic violence- "Domestic violence" is a broad term that entails not only physical beating but also other
forms of violence such as emotional violence, mental violence, sexual violence, nancial violence and other
forms of cruelty that may occur within a household. The de nition provided in Section 3 of the DV Act includes
the following as acts of domestic violence. The Section also de nes the meaning of terms physical abuse,
sexual abuse, verbal and emotional abuse, and economic abuse. It further enunciates that the overall facts and
circumstances of the case shall be taken into consideration in order to determine whether any act, omission,
commission or conduct of the respondent constitutes "domestic violence" under the said section.

Q.89 [30465398]
Abhi and Ashu, two bachelors, were in a living relationship for 4 years. One ne morning Ashu left a letter asking
Abhi to empty the place immediately saying he was no longer interested in the relationship and paid back her
share of rent. As Abhi failed to comply Ashu forcefully removed her from the house. Ashu led a complaint
under domestic violence act for the rightto a shared household. Decide.

a Abhi cannot enforce the rightto a shared household as both are bachelors and not married.

https://www.aspiration.ai/LAW/sis/Solution.jsp?qsetId=JAmrn2p8H1A=&qsetName=LST Mock 27 2021 (CLAT) 146/223


1/17/2021 Mock Analysis

b Abhi can enforce the rightto a shared household irrespective of their marital status.

c Abhi cannot le a complaint under domestic violence act but can approach the court under IPC 1860.

d Abhi can return back to her parent's place or stay at a different place. Living relationshipsend in vain.

Solution:
 Answer key/Solution
Correct Answer : b
Your Answer : b
It is clear from the passage that right to shared household can be enforced by non
-married couples too. In the given case Abhi and Ashu are non- married couples staying together for a
considerably long time. Right to a shared household can be obtained.

FeedBack Bookmark

https://www.aspiration.ai/LAW/sis/Solution.jsp?qsetId=JAmrn2p8H1A=&qsetName=LST Mock 27 2021 (CLAT) 147/223


1/17/2021 Mock Analysis

Directions for questions 67 to 105: You have been given some passages followed by questions based on each
passage. You are required to choose the most appropriate option which follows from the passage. Only the
information given in the passage should be used for choosing the answer and no external knowledge of law
howsoever prominent is to be applied.

Passage – 5

According to Section 2(f) of Domestic violence Act 2005, "domestic relationship" means a relationship between
two persons living in a shared household. Domestic relationships can be through marriage such as wives,
daughtersin- law, sisters-in-law, widows and any other members of the family; or blood relationships such as
mothers, sisters or daughters; and other domestic relationships including through adoption, live-in relationships,
and women in bigamous relationships or victims of legally invalid marriages. The law addresses the concerns
of women of all ages irrespective of their marital status. The de nition of "domestic relationship" under the
Domestic Violence Act is exhaustive: when a de nition clause is de ned to "mean" such and such, the de nition
is prima facie restrictive and exhaustive. The Supreme Court has further stated that the word domestic
relationship means a relationship that has some inherent or essential characteristics of marriage though not a
marriage that is legally recognized. Expression "relationship in the nature of marriage" cannot be construed in
the abstract. It is to be taken in the context in which it appears and to be applied bearing in mind the purpose
and object of DV Act as well as meaning of the expression "in the nature of marriage", (Indra Sarma v. V.K.V
Sarma, (2013))

Shared household- According to Section 2(s)of DV Act 2005, a shared household is where the aggrieved person
or a woman lives in a domestic relationship, either singly, or along with the man against whom the complaint is
led. It may also imply a household where a woman has lived in a domestic relationship but has been thrown
out. This may include all kinds of situations whether the household is owned by the respondent or it is rented
accommodation. It also includes a house either owned jointly by the aggrieved person and the respondent or
both may have jointly or singly, any rights, titles or interests. The DV Act recognizes a woman's right to reside in
a shared household. This means a woman cannot be thrown out of such a household except through the
procedure established by the law. In case she is thrown out she can be brought back again after obtaining the
order from the court. A woman to claim the protection of right in "shared household" has to establish
(a) that the relationship with the opposite party is "domestic relationship", and
(b) that the house in respect of which she seeks to enforce the right is "shared household".

Domestic violence- "Domestic violence" is a broad term that entails not only physical beating but also other
forms of violence such as emotional violence, mental violence, sexual violence, nancial violence and other
forms of cruelty that may occur within a household. The de nition provided in Section 3 of the DV Act includes
the following as acts of domestic violence. The Section also de nes the meaning of terms physical abuse,
sexual abuse, verbal and emotional abuse, and economic abuse. It further enunciates that the overall facts and
circumstances of the case shall be taken into consideration in order to determine whether any act, omission,
commission or conduct of the respondent constitutes "domestic violence" under the said section.

Q.90 [30465398]
Pick the wrong statement regarding domestic violence act 2005 from the following?

a Domestic violence includes physical and sexual violence but not mental violence.

b Section 3 of domestic violence act 2005 de nes physical abuse.

https://www.aspiration.ai/LAW/sis/Solution.jsp?qsetId=JAmrn2p8H1A=&qsetName=LST Mock 27 2021 (CLAT) 148/223


1/17/2021 Mock Analysis

c According to Section 2(f) of Domestic violence Act 2005, "domestic relationship" means a relationship
between two persons living in a shared household

d According to Section 2(s)of DV Act 2005, a shared household is where the aggrieved person or a woman
lives in a domestic relationship, either singly, or along with the man against whom the complaint is led.

Solution:
 Answer key/Solution
Correct Answer : a
Your Answer : a
All statements except option a are true and valid.
FeedBack Bookmark

https://www.aspiration.ai/LAW/sis/Solution.jsp?qsetId=JAmrn2p8H1A=&qsetName=LST Mock 27 2021 (CLAT) 149/223


1/17/2021 Mock Analysis

Directions for questions 67 to 105: You have been given some passages followed by questions based on each
passage. You are required to choose the most appropriate option which follows from the passage. Only the
information given in the passage should be used for choosing the answer and no external knowledge of law
howsoever prominent is to be applied.

Passage – 5

According to Section 2(f) of Domestic violence Act 2005, "domestic relationship" means a relationship between
two persons living in a shared household. Domestic relationships can be through marriage such as wives,
daughtersin- law, sisters-in-law, widows and any other members of the family; or blood relationships such as
mothers, sisters or daughters; and other domestic relationships including through adoption, live-in relationships,
and women in bigamous relationships or victims of legally invalid marriages. The law addresses the concerns
of women of all ages irrespective of their marital status. The de nition of "domestic relationship" under the
Domestic Violence Act is exhaustive: when a de nition clause is de ned to "mean" such and such, the de nition
is prima facie restrictive and exhaustive. The Supreme Court has further stated that the word domestic
relationship means a relationship that has some inherent or essential characteristics of marriage though not a
marriage that is legally recognized. Expression "relationship in the nature of marriage" cannot be construed in
the abstract. It is to be taken in the context in which it appears and to be applied bearing in mind the purpose
and object of DV Act as well as meaning of the expression "in the nature of marriage", (Indra Sarma v. V.K.V
Sarma, (2013))

Shared household- According to Section 2(s)of DV Act 2005, a shared household is where the aggrieved person
or a woman lives in a domestic relationship, either singly, or along with the man against whom the complaint is
led. It may also imply a household where a woman has lived in a domestic relationship but has been thrown
out. This may include all kinds of situations whether the household is owned by the respondent or it is rented
accommodation. It also includes a house either owned jointly by the aggrieved person and the respondent or
both may have jointly or singly, any rights, titles or interests. The DV Act recognizes a woman's right to reside in
a shared household. This means a woman cannot be thrown out of such a household except through the
procedure established by the law. In case she is thrown out she can be brought back again after obtaining the
order from the court. A woman to claim the protection of right in "shared household" has to establish
(a) that the relationship with the opposite party is "domestic relationship", and
(b) that the house in respect of which she seeks to enforce the right is "shared household".

Domestic violence- "Domestic violence" is a broad term that entails not only physical beating but also other
forms of violence such as emotional violence, mental violence, sexual violence, nancial violence and other
forms of cruelty that may occur within a household. The de nition provided in Section 3 of the DV Act includes
the following as acts of domestic violence. The Section also de nes the meaning of terms physical abuse,
sexual abuse, verbal and emotional abuse, and economic abuse. It further enunciates that the overall facts and
circumstances of the case shall be taken into consideration in order to determine whether any act, omission,
commission or conduct of the respondent constitutes "domestic violence" under the said section.

Q.91 [30465398]
Which of the following statements regarding shared household is false?

a A woman to claim the protection of rights in "shared household" has to establish that the relationship with
the opposite party is "domestic relationship.

https://www.aspiration.ai/LAW/sis/Solution.jsp?qsetId=JAmrn2p8H1A=&qsetName=LST Mock 27 2021 (CLAT) 150/223


1/17/2021 Mock Analysis

b A woman to claim the protection of rights in "shared household" has to establish that the house in respect
of which she seeks to enforce the right is "shared household".

c A woman cannot be thrown out of such a household except through the procedure established by the law

d Shared household law exists only for the initial 10 years of the marriage.

Solution:
 Answer key/Solution
Correct Answer : d
Your Answer : d
All the statements except option d are true regarding shared household.

FeedBack Bookmark

Directions for questions 67 to 105: You have been given some passages followed by questions based on each
passage. You are required to choose the most appropriate option which follows from the passage. Only the
information given in the passage should be used for choosing the answer and no external knowledge of law
howsoever prominent is to be applied.

Passage – 6

On September 27, 2020, the president of India Mr. Ram Nath Kovind gave his assent to the three farm reform
bills which are touted to play a major role in revolutionising the development of farmers in India. Before
understanding the bills, a brief look into the past introductions is necessary. APMC (Agriculture Produce Market
Committee) Act was introduced in 1960's at the very same time when green revolution started in India. APMCs
set up Mandis or Markets across India where farmer's produce was sold. There are around 7000 APMCs in India
at present. Now, the process of selling the produce is that after harvesting crops are brought to the Mandis or
Markets where they sell the produce through auctioning or price discovery. Whom are the farmers selling the
crops? Not to the government but the middlemen or Arhatiyas. Middlemen are people between the farmer and
the retailer or big traders. For example, farmers sell their vegetables to the middlemen and then the vegetable
vendor buys vegetable from the middleman, vegetable vendor will not buy directly from the farmers.
Government gives license to these Middlemen; shops, storage facilities etc. are provided to them in APMC
markets. Many people work in these APMCs, there is storage of grains, so it requires laborers, accountants so
overall it is a self-thriving ecosystem. One thing which should be noted here is these APMC markets are
regulated by state governments, a tax is charged on each transaction so in a way government knows at price
produce is being sold.

Produces which are not being sold to the middle man are being bought by the government at MSP (Minimum
Support Price). MSP is constant throughout the country. MSP also ensured that produce bought be the
middlemen were not below a certain price. This system was good seeing 1960's but with time we need to evolve
similarly, not much was done to APMCs and some problems popped up. Middlemen started exploiting farmers,
they formed cartels or an understanding among themselves and started buying the produce at MSP only and
sold to traders at a high rate. Voice arose from time to time to remove these defects and in response
government brought the three Acts in 2020. These three farm Acts seek to replace ordinances issued in June

https://www.aspiration.ai/LAW/sis/Solution.jsp?qsetId=JAmrn2p8H1A=&qsetName=LST Mock 27 2021 (CLAT) 151/223


1/17/2021 Mock Analysis

2020.

1) The Farmers Produce Trade and Commerce (Promotion and Facilitation) Bill, 2020 (FPTC)- The FTPC seeks
to break the monopoly of government-regulated mandis, reduce reliance on APMCs (disintermediation or
reducing the in uence of 'middlemen' or Angadias) and promote 'One India, One Agriculture Market'. Intrastate
and Inter-state trade are now facilitated freely through this and governments cannot levy barriers like market fee
outside APMC (Agriculture Produce Market Committee) areas.
2) The Farmers (Empowerment and Protection) Agreement of Price Assurance and Farm Services Bill, 2020
(FAPAFS)- The FAPAFS bill provides a legal framework for farmers to enter into written contracts with
companies and produce for them. Combined with the rst bill, it leaves the market open for private players to
join. As per its Preamble, it allows "national framework on farming agreements that empowers farmers to
engage with agri-business rms, processors, wholesalers, exporters or large retailers for farm services and sale
of future farming produce at a mutually agreed 'remunerative price'.
3) The Essential Commodities (Amendment) Bill, 2020- It takes away cereals, pulses, oilseeds, edible oils, onion
and potatoes from the list of essential commodities. Therefore, these commodities are now free of the
Essential Commodities Act restrictions and stand deregulated. However, the central government has retained
the right to regulate them under extraordinary circumstances, such as in case of a war, famine, natural calamity,
and impose stock limits if there is a steep rise in prices. It enables hoarding and free trade now.

Most important concern of the farmers is the revoking Minimum Selling Price (MSP). Very understandably, the
farmers are concerned about why the bill is silent on MSP even as the Prime Minister adamantly guarantees
that MSP and oor price will very much be a part of the modi ed system. Absence of regulation of the newly
involved corporate players is another major area of concern for farmers.Being big private companies, exporters,
wholesalers, and processors, they will always have an edge in disputes. Written contract is not mandatory which
means farmer will never be able to prove violation of terms of contract. Loss of revenue to the state government
and loss of jobs for middle man are other concerns over the changes. But with every new introduction questions
and concerns are customary. Only time will tell whether Farm Bills can live up to its agendas or not.

Q.92 [30465398]
Z, a farmer cultivated tomatoes. At the end of a good tomato season a number of private companies
approached Z to buy his tomatoes at various rates. Never had Z sold his tomatoes anywhere apart from the
state run APMC mandis which provided with a minimum selling price (MSP). Z was informed by a friend, now
that the Farm bill 2020 was passed MSP and government procurement no longer existed. Help Z-

a With the introduction of private sector to buy products from farmers, MSP and government procurement no
longer existed.

b MSP and government procurement will continue same way as before, and Z can sell his tomatoes in local
APMC mandi.

c Z can sell his tomatoes in APMC mandis, but MSP no longer exists and the price is decided by average
demand rate.

d Z has the choice of selling his tomatoes to the highest bidder. Therefore, approaching mandi for MSP is
useless.

https://www.aspiration.ai/LAW/sis/Solution.jsp?qsetId=JAmrn2p8H1A=&qsetName=LST Mock 27 2021 (CLAT) 152/223


1/17/2021 Mock Analysis

Solution:
 Answer key/Solution
Correct Answer : b
In the last paragraph of the comprehension (2nd and 3rd line) it is mentioned that
the prime minster has assured that MSP and oor price will very much be party of
the modi ed system. Therefore, option b is the correct answer.
FeedBack Bookmark

Directions for questions 67 to 105: You have been given some passages followed by questions based on each
passage. You are required to choose the most appropriate option which follows from the passage. Only the
information given in the passage should be used for choosing the answer and no external knowledge of law
howsoever prominent is to be applied.

Passage – 6

On September 27, 2020, the president of India Mr. Ram Nath Kovind gave his assent to the three farm reform
bills which are touted to play a major role in revolutionising the development of farmers in India. Before
understanding the bills, a brief look into the past introductions is necessary. APMC (Agriculture Produce Market
Committee) Act was introduced in 1960's at the very same time when green revolution started in India. APMCs
set up Mandis or Markets across India where farmer's produce was sold. There are around 7000 APMCs in India
at present. Now, the process of selling the produce is that after harvesting crops are brought to the Mandis or
Markets where they sell the produce through auctioning or price discovery. Whom are the farmers selling the
crops? Not to the government but the middlemen or Arhatiyas. Middlemen are people between the farmer and
the retailer or big traders. For example, farmers sell their vegetables to the middlemen and then the vegetable
vendor buys vegetable from the middleman, vegetable vendor will not buy directly from the farmers.
Government gives license to these Middlemen; shops, storage facilities etc. are provided to them in APMC
markets. Many people work in these APMCs, there is storage of grains, so it requires laborers, accountants so
overall it is a self-thriving ecosystem. One thing which should be noted here is these APMC markets are
regulated by state governments, a tax is charged on each transaction so in a way government knows at price
produce is being sold.

Produces which are not being sold to the middle man are being bought by the government at MSP (Minimum
Support Price). MSP is constant throughout the country. MSP also ensured that produce bought be the
middlemen were not below a certain price. This system was good seeing 1960's but with time we need to evolve
similarly, not much was done to APMCs and some problems popped up. Middlemen started exploiting farmers,
they formed cartels or an understanding among themselves and started buying the produce at MSP only and
sold to traders at a high rate. Voice arose from time to time to remove these defects and in response
government brought the three Acts in 2020. These three farm Acts seek to replace ordinances issued in June
2020.

1) The Farmers Produce Trade and Commerce (Promotion and Facilitation) Bill, 2020 (FPTC)- The FTPC seeks
to break the monopoly of government-regulated mandis, reduce reliance on APMCs (disintermediation or
reducing the in uence of 'middlemen' or Angadias) and promote 'One India, One Agriculture Market'. Intrastate
and Inter-state trade are now facilitated freely through this and governments cannot levy barriers like market fee
outside APMC (Agriculture Produce Market Committee) areas.
2) The Farmers (Empowerment and Protection) Agreement of Price Assurance and Farm Services Bill, 2020
(FAPAFS)- The FAPAFS bill provides a legal framework for farmers to enter into written contracts with
https://www.aspiration.ai/LAW/sis/Solution.jsp?qsetId=JAmrn2p8H1A=&qsetName=LST Mock 27 2021 (CLAT) 153/223
1/17/2021 Mock Analysis

companies and produce for them. Combined with the rst bill, it leaves the market open for private players to
join. As per its Preamble, it allows "national framework on farming agreements that empowers farmers to
engage with agri-business rms, processors, wholesalers, exporters or large retailers for farm services and sale
of future farming produce at a mutually agreed 'remunerative price'.
3) The Essential Commodities (Amendment) Bill, 2020- It takes away cereals, pulses, oilseeds, edible oils, onion
and potatoes from the list of essential commodities. Therefore, these commodities are now free of the
Essential Commodities Act restrictions and stand deregulated. However, the central government has retained
the right to regulate them under extraordinary circumstances, such as in case of a war, famine, natural calamity,
and impose stock limits if there is a steep rise in prices. It enables hoarding and free trade now.

Most important concern of the farmers is the revoking Minimum Selling Price (MSP). Very understandably, the
farmers are concerned about why the bill is silent on MSP even as the Prime Minister adamantly guarantees
that MSP and oor price will very much be a part of the modi ed system. Absence of regulation of the newly
involved corporate players is another major area of concern for farmers.Being big private companies, exporters,
wholesalers, and processors, they will always have an edge in disputes. Written contract is not mandatory which
means farmer will never be able to prove violation of terms of contract. Loss of revenue to the state government
and loss of jobs for middle man are other concerns over the changes. But with every new introduction questions
and concerns are customary. Only time will tell whether Farm Bills can live up to its agendas or not.

Q.93 [30465398]
A, a farmer used to sell his products in APMC mandi. The price of onion in the mandi was 8 Rs/-KG. A was
utterly disappointed. What other possible ways can A seek? (Assuming that Farm Bills 2020 are passed and
implemented).
(I) A can seek to sell his products to any private buyer outside the mandi without paying market fee.
(II) A can seek to sell his goods across the border of his state/district without any special charges.
(III) A can bargain and ask the mandi to increase the price. If not satis ed A can le a complaint in farmer
dispute redressal forum.
(IV) A can seek to sell his products outside the mandi, but should obtain a special permission from the
concerned government authorities.

a A only

b A and D only

c A B and c only

d A and B only

Solution:
 Answer key/Solution
Correct Answer : d
Your Answer : d
The Farmers Produce Trade and Commerce (Promotion and Facilitation) Bill,
2020 (FPTC) allows farmers to trade outside the mandi and with private entities without paying any market
charges. It has been explained in the above- mentioned passage.

FeedBack Bookmark

https://www.aspiration.ai/LAW/sis/Solution.jsp?qsetId=JAmrn2p8H1A=&qsetName=LST Mock 27 2021 (CLAT) 154/223


1/17/2021 Mock Analysis

Directions for questions 67 to 105: You have been given some passages followed by questions based on each
passage. You are required to choose the most appropriate option which follows from the passage. Only the
information given in the passage should be used for choosing the answer and no external knowledge of law
howsoever prominent is to be applied.

Passage – 6

On September 27, 2020, the president of India Mr. Ram Nath Kovind gave his assent to the three farm reform
bills which are touted to play a major role in revolutionising the development of farmers in India. Before
understanding the bills, a brief look into the past introductions is necessary. APMC (Agriculture Produce Market
Committee) Act was introduced in 1960's at the very same time when green revolution started in India. APMCs
set up Mandis or Markets across India where farmer's produce was sold. There are around 7000 APMCs in India
at present. Now, the process of selling the produce is that after harvesting crops are brought to the Mandis or
Markets where they sell the produce through auctioning or price discovery. Whom are the farmers selling the
crops? Not to the government but the middlemen or Arhatiyas. Middlemen are people between the farmer and
the retailer or big traders. For example, farmers sell their vegetables to the middlemen and then the vegetable
vendor buys vegetable from the middleman, vegetable vendor will not buy directly from the farmers.
Government gives license to these Middlemen; shops, storage facilities etc. are provided to them in APMC
markets. Many people work in these APMCs, there is storage of grains, so it requires laborers, accountants so
overall it is a self-thriving ecosystem. One thing which should be noted here is these APMC markets are
regulated by state governments, a tax is charged on each transaction so in a way government knows at price
produce is being sold.

Produces which are not being sold to the middle man are being bought by the government at MSP (Minimum
Support Price). MSP is constant throughout the country. MSP also ensured that produce bought be the
middlemen were not below a certain price. This system was good seeing 1960's but with time we need to evolve
similarly, not much was done to APMCs and some problems popped up. Middlemen started exploiting farmers,
they formed cartels or an understanding among themselves and started buying the produce at MSP only and
sold to traders at a high rate. Voice arose from time to time to remove these defects and in response
government brought the three Acts in 2020. These three farm Acts seek to replace ordinances issued in June
2020.

1) The Farmers Produce Trade and Commerce (Promotion and Facilitation) Bill, 2020 (FPTC)- The FTPC seeks
to break the monopoly of government-regulated mandis, reduce reliance on APMCs (disintermediation or
reducing the in uence of 'middlemen' or Angadias) and promote 'One India, One Agriculture Market'. Intrastate
and Inter-state trade are now facilitated freely through this and governments cannot levy barriers like market fee
outside APMC (Agriculture Produce Market Committee) areas.
2) The Farmers (Empowerment and Protection) Agreement of Price Assurance and Farm Services Bill, 2020
(FAPAFS)- The FAPAFS bill provides a legal framework for farmers to enter into written contracts with
companies and produce for them. Combined with the rst bill, it leaves the market open for private players to
join. As per its Preamble, it allows "national framework on farming agreements that empowers farmers to
engage with agri-business rms, processors, wholesalers, exporters or large retailers for farm services and sale
of future farming produce at a mutually agreed 'remunerative price'.
3) The Essential Commodities (Amendment) Bill, 2020- It takes away cereals, pulses, oilseeds, edible oils, onion
and potatoes from the list of essential commodities. Therefore, these commodities are now free of the
Essential Commodities Act restrictions and stand deregulated. However, the central government has retained
the right to regulate them under extraordinary circumstances, such as in case of a war, famine, natural calamity,

https://www.aspiration.ai/LAW/sis/Solution.jsp?qsetId=JAmrn2p8H1A=&qsetName=LST Mock 27 2021 (CLAT) 155/223


1/17/2021 Mock Analysis

and impose stock limits if there is a steep rise in prices. It enables hoarding and free trade now.

Most important concern of the farmers is the revoking Minimum Selling Price (MSP). Very understandably, the
farmers are concerned about why the bill is silent on MSP even as the Prime Minister adamantly guarantees
that MSP and oor price will very much be a part of the modi ed system. Absence of regulation of the newly
involved corporate players is another major area of concern for farmers.Being big private companies, exporters,
wholesalers, and processors, they will always have an edge in disputes. Written contract is not mandatory which
means farmer will never be able to prove violation of terms of contract. Loss of revenue to the state government
and loss of jobs for middle man are other concerns over the changes. But with every new introduction questions
and concerns are customary. Only time will tell whether Farm Bills can live up to its agendas or not.

Q.94 [30465398]
What aspect of MSP has changed from 1960 to present day according to the comprehension?

a The effectivity of MSP has seen reached a plateau during last 20 years.

b Middlemen started exploiting farmers, they formed cartels or an understanding among themselves and
started buying the produce at MSP only and sold to traders at a high rate.

c Middleman started exploiting the farmers, they formed cartels or an understanding among themselves and
starting charging extra taxes secretly above the MSP.

d MSP has improved the state of farmers to great extent and had brought health, wealth and prosperity in the
recent years. Thus, farmers in Haryana and Rajasthan are protesting.

Solution:
 Answer key/Solution
Correct Answer : b
Your Answer : b
Evolution of MSP has been explained in the second paragraph of the passage. 5th
line of second paragraph reads "Middleman started exploiting the farmers, they formed cartels or an
understanding among themselves and started buying the product at MSP only and sold to traders at a high
rate".

FeedBack Bookmark

Directions for questions 67 to 105: You have been given some passages followed by questions based on each
passage. You are required to choose the most appropriate option which follows from the passage. Only the
information given in the passage should be used for choosing the answer and no external knowledge of law
howsoever prominent is to be applied.

Passage – 6

On September 27, 2020, the president of India Mr. Ram Nath Kovind gave his assent to the three farm reform
bills which are touted to play a major role in revolutionising the development of farmers in India. Before
understanding the bills, a brief look into the past introductions is necessary. APMC (Agriculture Produce Market

https://www.aspiration.ai/LAW/sis/Solution.jsp?qsetId=JAmrn2p8H1A=&qsetName=LST Mock 27 2021 (CLAT) 156/223


1/17/2021 Mock Analysis

Committee) Act was introduced in 1960's at the very same time when green revolution started in India. APMCs
set up Mandis or Markets across India where farmer's produce was sold. There are around 7000 APMCs in India
at present. Now, the process of selling the produce is that after harvesting crops are brought to the Mandis or
Markets where they sell the produce through auctioning or price discovery. Whom are the farmers selling the
crops? Not to the government but the middlemen or Arhatiyas. Middlemen are people between the farmer and
the retailer or big traders. For example, farmers sell their vegetables to the middlemen and then the vegetable
vendor buys vegetable from the middleman, vegetable vendor will not buy directly from the farmers.
Government gives license to these Middlemen; shops, storage facilities etc. are provided to them in APMC
markets. Many people work in these APMCs, there is storage of grains, so it requires laborers, accountants so
overall it is a self-thriving ecosystem. One thing which should be noted here is these APMC markets are
regulated by state governments, a tax is charged on each transaction so in a way government knows at price
produce is being sold.

Produces which are not being sold to the middle man are being bought by the government at MSP (Minimum
Support Price). MSP is constant throughout the country. MSP also ensured that produce bought be the
middlemen were not below a certain price. This system was good seeing 1960's but with time we need to evolve
similarly, not much was done to APMCs and some problems popped up. Middlemen started exploiting farmers,
they formed cartels or an understanding among themselves and started buying the produce at MSP only and
sold to traders at a high rate. Voice arose from time to time to remove these defects and in response
government brought the three Acts in 2020. These three farm Acts seek to replace ordinances issued in June
2020.

1) The Farmers Produce Trade and Commerce (Promotion and Facilitation) Bill, 2020 (FPTC)- The FTPC seeks
to break the monopoly of government-regulated mandis, reduce reliance on APMCs (disintermediation or
reducing the in uence of 'middlemen' or Angadias) and promote 'One India, One Agriculture Market'. Intrastate
and Inter-state trade are now facilitated freely through this and governments cannot levy barriers like market fee
outside APMC (Agriculture Produce Market Committee) areas.
2) The Farmers (Empowerment and Protection) Agreement of Price Assurance and Farm Services Bill, 2020
(FAPAFS)- The FAPAFS bill provides a legal framework for farmers to enter into written contracts with
companies and produce for them. Combined with the rst bill, it leaves the market open for private players to
join. As per its Preamble, it allows "national framework on farming agreements that empowers farmers to
engage with agri-business rms, processors, wholesalers, exporters or large retailers for farm services and sale
of future farming produce at a mutually agreed 'remunerative price'.
3) The Essential Commodities (Amendment) Bill, 2020- It takes away cereals, pulses, oilseeds, edible oils, onion
and potatoes from the list of essential commodities. Therefore, these commodities are now free of the
Essential Commodities Act restrictions and stand deregulated. However, the central government has retained
the right to regulate them under extraordinary circumstances, such as in case of a war, famine, natural calamity,
and impose stock limits if there is a steep rise in prices. It enables hoarding and free trade now.

Most important concern of the farmers is the revoking Minimum Selling Price (MSP). Very understandably, the
farmers are concerned about why the bill is silent on MSP even as the Prime Minister adamantly guarantees
that MSP and oor price will very much be a part of the modi ed system. Absence of regulation of the newly
involved corporate players is another major area of concern for farmers.Being big private companies, exporters,
wholesalers, and processors, they will always have an edge in disputes. Written contract is not mandatory which
means farmer will never be able to prove violation of terms of contract. Loss of revenue to the state government
and loss of jobs for middle man are other concerns over the changes. But with every new introduction questions
and concerns are customary. Only time will tell whether Farm Bills can live up to its agendas or not.

https://www.aspiration.ai/LAW/sis/Solution.jsp?qsetId=JAmrn2p8H1A=&qsetName=LST Mock 27 2021 (CLAT) 157/223


1/17/2021 Mock Analysis

Q.95 [30465398]
Monsoon rains across India lead to ooding in various states. This led to dearth of potatoes and the price of
potatoes skyrocketed. Decide the role of central government in controlling the price and situation. (Assuming
that farm bill 2020 is passed and implemented)

a Central government should ask the state governments to set a bracket amount for the selling potatoes.

b Central government under The Essential Commodities (Amendment) Bill, 2020 has the right to decrease the
prices of potatoes to the bene t of the customers.

c Central government under The Essential Commodities (Amendment) Bill, 2020 has right to stock limits if
there is steep rise in prices.

d Central government has no power under The Essential Commodities (Amendment) Bill, 2020 but state
governments do.

Solution:
 Answer key/Solution
Correct Answer : c
Your Answer : c
It is given in the 2nd last paragraph of the passage that during extraordinary
circumstances like famine, natural calamities, war central government has the power to take over the
regulation of essential commodities.

FeedBack Bookmark

Directions for questions 67 to 105: You have been given some passages followed by questions based on each
passage. You are required to choose the most appropriate option which follows from the passage. Only the
information given in the passage should be used for choosing the answer and no external knowledge of law
howsoever prominent is to be applied.

Passage – 6

On September 27, 2020, the president of India Mr. Ram Nath Kovind gave his assent to the three farm reform
bills which are touted to play a major role in revolutionising the development of farmers in India. Before
understanding the bills, a brief look into the past introductions is necessary. APMC (Agriculture Produce Market
Committee) Act was introduced in 1960's at the very same time when green revolution started in India. APMCs
set up Mandis or Markets across India where farmer's produce was sold. There are around 7000 APMCs in India
at present. Now, the process of selling the produce is that after harvesting crops are brought to the Mandis or
Markets where they sell the produce through auctioning or price discovery. Whom are the farmers selling the
crops? Not to the government but the middlemen or Arhatiyas. Middlemen are people between the farmer and
the retailer or big traders. For example, farmers sell their vegetables to the middlemen and then the vegetable
vendor buys vegetable from the middleman, vegetable vendor will not buy directly from the farmers.
Government gives license to these Middlemen; shops, storage facilities etc. are provided to them in APMC
markets. Many people work in these APMCs, there is storage of grains, so it requires laborers, accountants so

https://www.aspiration.ai/LAW/sis/Solution.jsp?qsetId=JAmrn2p8H1A=&qsetName=LST Mock 27 2021 (CLAT) 158/223


1/17/2021 Mock Analysis

overall it is a self-thriving ecosystem. One thing which should be noted here is these APMC markets are
regulated by state governments, a tax is charged on each transaction so in a way government knows at price
produce is being sold.

Produces which are not being sold to the middle man are being bought by the government at MSP (Minimum
Support Price). MSP is constant throughout the country. MSP also ensured that produce bought be the
middlemen were not below a certain price. This system was good seeing 1960's but with time we need to evolve
similarly, not much was done to APMCs and some problems popped up. Middlemen started exploiting farmers,
they formed cartels or an understanding among themselves and started buying the produce at MSP only and
sold to traders at a high rate. Voice arose from time to time to remove these defects and in response
government brought the three Acts in 2020. These three farm Acts seek to replace ordinances issued in June
2020.

1) The Farmers Produce Trade and Commerce (Promotion and Facilitation) Bill, 2020 (FPTC)- The FTPC seeks
to break the monopoly of government-regulated mandis, reduce reliance on APMCs (disintermediation or
reducing the in uence of 'middlemen' or Angadias) and promote 'One India, One Agriculture Market'. Intrastate
and Inter-state trade are now facilitated freely through this and governments cannot levy barriers like market fee
outside APMC (Agriculture Produce Market Committee) areas.
2) The Farmers (Empowerment and Protection) Agreement of Price Assurance and Farm Services Bill, 2020
(FAPAFS)- The FAPAFS bill provides a legal framework for farmers to enter into written contracts with
companies and produce for them. Combined with the rst bill, it leaves the market open for private players to
join. As per its Preamble, it allows "national framework on farming agreements that empowers farmers to
engage with agri-business rms, processors, wholesalers, exporters or large retailers for farm services and sale
of future farming produce at a mutually agreed 'remunerative price'.
3) The Essential Commodities (Amendment) Bill, 2020- It takes away cereals, pulses, oilseeds, edible oils, onion
and potatoes from the list of essential commodities. Therefore, these commodities are now free of the
Essential Commodities Act restrictions and stand deregulated. However, the central government has retained
the right to regulate them under extraordinary circumstances, such as in case of a war, famine, natural calamity,
and impose stock limits if there is a steep rise in prices. It enables hoarding and free trade now.

Most important concern of the farmers is the revoking Minimum Selling Price (MSP). Very understandably, the
farmers are concerned about why the bill is silent on MSP even as the Prime Minister adamantly guarantees
that MSP and oor price will very much be a part of the modi ed system. Absence of regulation of the newly
involved corporate players is another major area of concern for farmers.Being big private companies, exporters,
wholesalers, and processors, they will always have an edge in disputes. Written contract is not mandatory which
means farmer will never be able to prove violation of terms of contract. Loss of revenue to the state government
and loss of jobs for middle man are other concerns over the changes. But with every new introduction questions
and concerns are customary. Only time will tell whether Farm Bills can live up to its agendas or not.

Q.96 [30465398]
Which of the following are the correct set of reasons for the concern/ dissatisfaction of farmers with the
introduction of farm bills according to the paragraph?

a MSP, Introduction of corporate/private entities, more power to central government, Loss of revenue for the
state

b MSP, Loss of revenue for the state, Arbitrary control by central government, Loss of revenue to the state,
Loss of jobs for the middle man

https://www.aspiration.ai/LAW/sis/Solution.jsp?qsetId=JAmrn2p8H1A=&qsetName=LST Mock 27 2021 (CLAT) 159/223


1/17/2021 Mock Analysis

c MSP, Introduction of corporate/private entities, Loss of revenue to the state, loss of job for middle man

d Introduction of corporate/private sector, Loss of job for middleman, Burden on farmers, Arbitrary control by
the centre.

Solution:
 Answer key/Solution
Correct Answer : c
Your Answer : a
In the last paragraph of the passage all the four reasons has been mentioned.
Other reasons given are not mentioned in the passage hence option c is the correct option.

FeedBack Bookmark

https://www.aspiration.ai/LAW/sis/Solution.jsp?qsetId=JAmrn2p8H1A=&qsetName=LST Mock 27 2021 (CLAT) 160/223


1/17/2021 Mock Analysis

Directions for questions 67 to 105: You have been given some passages followed by questions based on each
passage. You are required to choose the most appropriate option which follows from the passage. Only the
information given in the passage should be used for choosing the answer and no external knowledge of law
howsoever prominent is to be applied.

Passage – 7

A contract is an agreement between two or more parties creating obligations that are enforceable or otherwise
recognizable at law. Execution of these obligations may be affected by unforeseen or supervening events, i.e.,
events which are unexpected or incapable of being known in advance by either of the parties and which
ultimately discharge the parties from their contractual obligations. The doctrine of frustration is a "doctrine" of
special case of the discharge of contract by an impossibility to perform it. The Indian Contract Act, 1872
("Contract Act) does not de ne the term frustration. The Black's Law Dictionary de nes frustration in relation to
contracts as "the doctrine that if a party's principal purpose is substantially frustrated by unanticipated changed
circumstances, that party's duties are discharged and the contract is considered terminated," also termed as the
frustration of purpose. In India, courts nuanced: "The expression 'frustration of the contract' is an elliptical
expression. The fuller and more accurate expression is 'frustration of the adventure or of the commercial or
practical purpose of contract'". This doctrine is a device to reconcile the rule of absolute contracts with a special
exception which is demanded in certain circumstances in the name of justice.

The doctrine comes within the purview of section 56 of the Contract Act as it discharges the contract by reason
of supervening impossibility or illegality of the act agreed to be done. A contract is also frustrated under section
32 when the condition, on which the contract is contingent, is not ful lled or cannot be ful lled because of
impossibility (paragraph 1 and 2 of section 32, respectively). Nevertheless, the doctrine under Indian law is
associated with section 56. As section 32 only applies when contracts are discharged and parties absolved of
their obligations as per terms already contained in the relevant contract. Section 56 applies when contracts are
discharged and parties absolved of their obligations as a result of subsequent impossibility due to outside
forces and factors. Section 56 states that an agreement to do an act which becomes impossible or unlawful is
void. The word "impossible" has not been used in the sense of physical or literal impossibility. The performance
of an act may not be literally impossible but it may be impracticable and useless from the point of view of the
object and purpose which the parties had in view. Therefore, if an untoward event or change of circumstances
totally upsets the very foundation upon which the parties rested their bargain, it can very well be said that the
promisor nds it impossible to do the act which he promised to do.

Q.97 [30465398]
A, the defendant was keen on watching coronation ceremony of the king. Therefore, he hired a at for two days
as he could watch the ceremony from the balcony of the at. He informed B about the reason for the renting the
at. But due to ill health of the king, the ceremony was cancelled. A refused to pay the rent. Decide-

a A is liable to pay the rent irrespective of the coronation as the object of the contract was to reside and
residence was possible.

b A is not liable to pay the rent as the doctrine of frustration applies, the object of contract as recognised by
both the parties was to witness coronation ceremony and it was impossible.

c A is liable to pay the rent and the doctrine of frustration does not apply as the plaintiff rented the at with
the view of residence.

https://www.aspiration.ai/LAW/sis/Solution.jsp?qsetId=JAmrn2p8H1A=&qsetName=LST Mock 27 2021 (CLAT) 161/223


1/17/2021 Mock Analysis

d A is not liable to pay the rent as he did not reside in the at, but doctrine of impossibility does not apply.

Solution:
 Answer key/Solution
Correct Answer : b
The doctrine of frustration applies as the very object of the contract, as
recognised by both the contracting parties, was to have a view of the coronation
process. The happening of the coronation hence forms the heart of the contract. The very object of the
contract was frustrated by the non-happening of the coronation and the plaintiff is held not entitled to receive
money as rent fees.
FeedBack Bookmark

Directions for questions 67 to 105: You have been given some passages followed by questions based on each
passage. You are required to choose the most appropriate option which follows from the passage. Only the
information given in the passage should be used for choosing the answer and no external knowledge of law
howsoever prominent is to be applied.

Passage – 7

A contract is an agreement between two or more parties creating obligations that are enforceable or otherwise
recognizable at law. Execution of these obligations may be affected by unforeseen or supervening events, i.e.,
events which are unexpected or incapable of being known in advance by either of the parties and which
ultimately discharge the parties from their contractual obligations. The doctrine of frustration is a "doctrine" of
special case of the discharge of contract by an impossibility to perform it. The Indian Contract Act, 1872
("Contract Act) does not de ne the term frustration. The Black's Law Dictionary de nes frustration in relation to
contracts as "the doctrine that if a party's principal purpose is substantially frustrated by unanticipated changed
circumstances, that party's duties are discharged and the contract is considered terminated," also termed as the
frustration of purpose. In India, courts nuanced: "The expression 'frustration of the contract' is an elliptical
expression. The fuller and more accurate expression is 'frustration of the adventure or of the commercial or
practical purpose of contract'". This doctrine is a device to reconcile the rule of absolute contracts with a special
exception which is demanded in certain circumstances in the name of justice.

The doctrine comes within the purview of section 56 of the Contract Act as it discharges the contract by reason
of supervening impossibility or illegality of the act agreed to be done. A contract is also frustrated under section
32 when the condition, on which the contract is contingent, is not ful lled or cannot be ful lled because of
impossibility (paragraph 1 and 2 of section 32, respectively). Nevertheless, the doctrine under Indian law is
associated with section 56. As section 32 only applies when contracts are discharged and parties absolved of
their obligations as per terms already contained in the relevant contract. Section 56 applies when contracts are
discharged and parties absolved of their obligations as a result of subsequent impossibility due to outside
forces and factors. Section 56 states that an agreement to do an act which becomes impossible or unlawful is
void. The word "impossible" has not been used in the sense of physical or literal impossibility. The performance
of an act may not be literally impossible but it may be impracticable and useless from the point of view of the
object and purpose which the parties had in view. Therefore, if an untoward event or change of circumstances
totally upsets the very foundation upon which the parties rested their bargain, it can very well be said that the
promisor nds it impossible to do the act which he promised to do.

https://www.aspiration.ai/LAW/sis/Solution.jsp?qsetId=JAmrn2p8H1A=&qsetName=LST Mock 27 2021 (CLAT) 162/223


1/17/2021 Mock Analysis

Q.98 [30465398]
A entered into contract with B to supply 100 litres of petrol. Due to some mishappening in the middle east, the
price of petrol rose 400%. A could not supply petrol. B sued A for speci c performance. A pleaded doctrine of
impossibility. Decide-

a A can plead doctrine of impossibility as the price escalated tremendously high and subsequently making it
impossible for the defendant to supply.

b A cannot plead doctrine of impossibility as rise and fall of prices is normal and expected in market.

c A can supply petrol once the prices come down and there is no need to go to court.

d A can plead the court to delay the date of supply but cannot withdraw himself from the contract.

Solution:
 Answer key/Solution
Correct Answer : a
Your Answer : a
The rise of price by 400% is ridiculously high and is unexpected. It is impossible
to supply the product with price being 400% higher than at the time of entering contract. Thus, A can plead the
defence of doctrine of impossibility.
FeedBack Bookmark

https://www.aspiration.ai/LAW/sis/Solution.jsp?qsetId=JAmrn2p8H1A=&qsetName=LST Mock 27 2021 (CLAT) 163/223


1/17/2021 Mock Analysis

Directions for questions 67 to 105: You have been given some passages followed by questions based on each
passage. You are required to choose the most appropriate option which follows from the passage. Only the
information given in the passage should be used for choosing the answer and no external knowledge of law
howsoever prominent is to be applied.

Passage – 7

A contract is an agreement between two or more parties creating obligations that are enforceable or otherwise
recognizable at law. Execution of these obligations may be affected by unforeseen or supervening events, i.e.,
events which are unexpected or incapable of being known in advance by either of the parties and which
ultimately discharge the parties from their contractual obligations. The doctrine of frustration is a "doctrine" of
special case of the discharge of contract by an impossibility to perform it. The Indian Contract Act, 1872
("Contract Act) does not de ne the term frustration. The Black's Law Dictionary de nes frustration in relation to
contracts as "the doctrine that if a party's principal purpose is substantially frustrated by unanticipated changed
circumstances, that party's duties are discharged and the contract is considered terminated," also termed as the
frustration of purpose. In India, courts nuanced: "The expression 'frustration of the contract' is an elliptical
expression. The fuller and more accurate expression is 'frustration of the adventure or of the commercial or
practical purpose of contract'". This doctrine is a device to reconcile the rule of absolute contracts with a special
exception which is demanded in certain circumstances in the name of justice.

The doctrine comes within the purview of section 56 of the Contract Act as it discharges the contract by reason
of supervening impossibility or illegality of the act agreed to be done. A contract is also frustrated under section
32 when the condition, on which the contract is contingent, is not ful lled or cannot be ful lled because of
impossibility (paragraph 1 and 2 of section 32, respectively). Nevertheless, the doctrine under Indian law is
associated with section 56. As section 32 only applies when contracts are discharged and parties absolved of
their obligations as per terms already contained in the relevant contract. Section 56 applies when contracts are
discharged and parties absolved of their obligations as a result of subsequent impossibility due to outside
forces and factors. Section 56 states that an agreement to do an act which becomes impossible or unlawful is
void. The word "impossible" has not been used in the sense of physical or literal impossibility. The performance
of an act may not be literally impossible but it may be impracticable and useless from the point of view of the
object and purpose which the parties had in view. Therefore, if an untoward event or change of circumstances
totally upsets the very foundation upon which the parties rested their bargain, it can very well be said that the
promisor nds it impossible to do the act which he promised to do.

Q.99 [30465398]
Z joined as a manager for ten years, and undertook a vote not to enter into any professional engagement
without the consent of the employer. Before the completion of the said term he was asked to join military
service. After the war Z undertook professional engagement and was sued by his employer. Z pleaded
frustration of contract. Decide

a Z cannot plead frustration as until the completion of 10 years he was under the contract and was
supposed to abide by it.

b Z cannot plead frustration as enrolling in military is no ground for frustration of contract.

c Z can plead frustration as from the time he joined military he was no longer a manager and was not bound
to the contract. Thus, the contract was frustrated.

https://www.aspiration.ai/LAW/sis/Solution.jsp?qsetId=JAmrn2p8H1A=&qsetName=LST Mock 27 2021 (CLAT) 164/223


1/17/2021 Mock Analysis

d All contracts stand rejected when one joins military as it is for the service of the nation.

Solution:
 Answer key/Solution
Correct Answer : c
Your Answer : c
In the given case, Z can successfully plead frustration. Once he joined military his
old employment is automatically terminated thus releasing him from the contract.

FeedBack Bookmark

Directions for questions 67 to 105: You have been given some passages followed by questions based on each
passage. You are required to choose the most appropriate option which follows from the passage. Only the
information given in the passage should be used for choosing the answer and no external knowledge of law
howsoever prominent is to be applied.

Passage – 7

A contract is an agreement between two or more parties creating obligations that are enforceable or otherwise
recognizable at law. Execution of these obligations may be affected by unforeseen or supervening events, i.e.,
events which are unexpected or incapable of being known in advance by either of the parties and which
ultimately discharge the parties from their contractual obligations. The doctrine of frustration is a "doctrine" of
special case of the discharge of contract by an impossibility to perform it. The Indian Contract Act, 1872
("Contract Act) does not de ne the term frustration. The Black's Law Dictionary de nes frustration in relation to
contracts as "the doctrine that if a party's principal purpose is substantially frustrated by unanticipated changed
circumstances, that party's duties are discharged and the contract is considered terminated," also termed as the
frustration of purpose. In India, courts nuanced: "The expression 'frustration of the contract' is an elliptical
expression. The fuller and more accurate expression is 'frustration of the adventure or of the commercial or
practical purpose of contract'". This doctrine is a device to reconcile the rule of absolute contracts with a special
exception which is demanded in certain circumstances in the name of justice.

The doctrine comes within the purview of section 56 of the Contract Act as it discharges the contract by reason
of supervening impossibility or illegality of the act agreed to be done. A contract is also frustrated under section
32 when the condition, on which the contract is contingent, is not ful lled or cannot be ful lled because of
impossibility (paragraph 1 and 2 of section 32, respectively). Nevertheless, the doctrine under Indian law is
associated with section 56. As section 32 only applies when contracts are discharged and parties absolved of
their obligations as per terms already contained in the relevant contract. Section 56 applies when contracts are
discharged and parties absolved of their obligations as a result of subsequent impossibility due to outside
forces and factors. Section 56 states that an agreement to do an act which becomes impossible or unlawful is
void. The word "impossible" has not been used in the sense of physical or literal impossibility. The performance
of an act may not be literally impossible but it may be impracticable and useless from the point of view of the
object and purpose which the parties had in view. Therefore, if an untoward event or change of circumstances
totally upsets the very foundation upon which the parties rested their bargain, it can very well be said that the
promisor nds it impossible to do the act which he promised to do.

https://www.aspiration.ai/LAW/sis/Solution.jsp?qsetId=JAmrn2p8H1A=&qsetName=LST Mock 27 2021 (CLAT) 165/223


1/17/2021 Mock Analysis

Q.100 [30465398]
According to the passage, why doctrine of frustration comes only under section 56 and not under section 32 of
Indian contract act 1872?

a Section 56 gives speci c explanation about impossibility while section 32 discusses about only one
particular condition.

b Section 32 applies only when the contracts are not discharged

c Section 32 only applies when contracts are discharged and parties absolved of their obligations as per
terms already contained in the relevant contract.

d Doctrine of frustration applies both under section 32 and 56. The above given statement is incorrect.

Solution:
 Answer key/Solution
Correct Answer : c
Your Answer : c
Section 32 talks about contingent contract. Section 32 can be applied only when
the contingent event has happened. In the last paragraph of the passage it is given 'Section 32 only applies
when contracts are discharged and parties absolved of their obligations as per terms already contained in the
relevant contract'.

FeedBack Bookmark

https://www.aspiration.ai/LAW/sis/Solution.jsp?qsetId=JAmrn2p8H1A=&qsetName=LST Mock 27 2021 (CLAT) 166/223


1/17/2021 Mock Analysis

Directions for questions 67 to 105: You have been given some passages followed by questions based on each
passage. You are required to choose the most appropriate option which follows from the passage. Only the
information given in the passage should be used for choosing the answer and no external knowledge of law
howsoever prominent is to be applied.

Passage – 7

A contract is an agreement between two or more parties creating obligations that are enforceable or otherwise
recognizable at law. Execution of these obligations may be affected by unforeseen or supervening events, i.e.,
events which are unexpected or incapable of being known in advance by either of the parties and which
ultimately discharge the parties from their contractual obligations. The doctrine of frustration is a "doctrine" of
special case of the discharge of contract by an impossibility to perform it. The Indian Contract Act, 1872
("Contract Act) does not de ne the term frustration. The Black's Law Dictionary de nes frustration in relation to
contracts as "the doctrine that if a party's principal purpose is substantially frustrated by unanticipated changed
circumstances, that party's duties are discharged and the contract is considered terminated," also termed as the
frustration of purpose. In India, courts nuanced: "The expression 'frustration of the contract' is an elliptical
expression. The fuller and more accurate expression is 'frustration of the adventure or of the commercial or
practical purpose of contract'". This doctrine is a device to reconcile the rule of absolute contracts with a special
exception which is demanded in certain circumstances in the name of justice.

The doctrine comes within the purview of section 56 of the Contract Act as it discharges the contract by reason
of supervening impossibility or illegality of the act agreed to be done. A contract is also frustrated under section
32 when the condition, on which the contract is contingent, is not ful lled or cannot be ful lled because of
impossibility (paragraph 1 and 2 of section 32, respectively). Nevertheless, the doctrine under Indian law is
associated with section 56. As section 32 only applies when contracts are discharged and parties absolved of
their obligations as per terms already contained in the relevant contract. Section 56 applies when contracts are
discharged and parties absolved of their obligations as a result of subsequent impossibility due to outside
forces and factors. Section 56 states that an agreement to do an act which becomes impossible or unlawful is
void. The word "impossible" has not been used in the sense of physical or literal impossibility. The performance
of an act may not be literally impossible but it may be impracticable and useless from the point of view of the
object and purpose which the parties had in view. Therefore, if an untoward event or change of circumstances
totally upsets the very foundation upon which the parties rested their bargain, it can very well be said that the
promisor nds it impossible to do the act which he promised to do.

Q.101 [30465398]
Which of the following is an impossible event under section 56 of the Indian contract act 1972?

a Y, invites A to play guitar at his ceremony. A has a bad ght with his girlfriend on the day of ceremony and
fails to attend the programme. While sued, A pleads doctrine of impossibility.

b T orders a brand -new car which was to be imported from America. On the day of export from America, the
ights get cancelled due to thunderstorm. The car company cancels the order and pleads doctrine of
impossibility in court while sued.

c A, signs a contract with singer Gatlin Beiber to perform at his place. On the day of programme Beiber suffer
a mild cardiac attack and is admitted to the hospital. A sues Beiber, Beiber pleads impossibility.

https://www.aspiration.ai/LAW/sis/Solution.jsp?qsetId=JAmrn2p8H1A=&qsetName=LST Mock 27 2021 (CLAT) 167/223


1/17/2021 Mock Analysis

d A failed to attend all import project as his car broke down. While sued, he pleads frustration under section
56.

Solution:
 Answer key/Solution
Correct Answer : c
Your Answer : c
In rst option ght with girlfriend cannot be a ground for impossibility. In 2nd the
car company could have sent the car once the thunderstorm was cleared. In 4th car break down is not a
reason as there are other ways to attend the presentation. But in 3rd option cardiac arrest puts the risk of
health of the defendant making it impossible for him to sing immediately after an attack. Thus, C is the right
answer.
FeedBack Bookmark

https://www.aspiration.ai/LAW/sis/Solution.jsp?qsetId=JAmrn2p8H1A=&qsetName=LST Mock 27 2021 (CLAT) 168/223


1/17/2021 Mock Analysis

Directions for questions 67 to 105: You have been given some passages followed by questions based on each
passage. You are required to choose the most appropriate option which follows from the passage. Only the
information given in the passage should be used for choosing the answer and no external knowledge of law
howsoever prominent is to be applied.

Passage – 8

In common law, assault is a tort, an act of the defendant which causes to the plaintiff reasonable apprehension
of the in iction of a battery on him by the defendant. When the defendant creates his act by an apprehension in
the mind of the plaintiff that he is going to commit battery against the plaintiff, the wrong of assault is
completed. The wrong consists of an attempt to do harm rather than the harm being caused thereby. In assault
charges must include conduct that is offensive which is offensive or causes another person to the fear of their
safety. This clearly means that one can be guilty of assault even if he/she did not physically harm the victim. In
the case of R. v. S George, it was held that the pointing of loaded gun to another is an assault. If the pistol is not
loaded, then even it may be an assault, if pointed at such a distance that it may cause injury. if a person
advances the manner of threatening to use force, then there is assault. This was decided in the case of
Stephens v. Myers.

If one or more elements have not been satis ed then It can be a defence to an assault charge. Elements of the
crime of assault are:

An act or conduct intended to created: To prove a criminal attack, the defendants' behaviour must be motivated
to create a situation of fear or danger in the victim's mind. Accident acts do not include allegations of assault.

A reasonable apprehension: Further, the victim must reasonably believe that the defendant's conduct will harm
or humiliate him. The victim must understand the defendant's potentially harmful or offensive acts.

Of imminent harm: The victim's fear must be a direct response to a threat that is imminent. Future threats, such
as "I will beat you tomorrow, will not result in assault charges. In addition, there must be some kind of perceived
physical threat to the victim in the loss; For this reason, words by themselves generally do not constitute an
attack.

It is believed that the defendant's actions would cause physical danger or abusive behaviour to the victim. Thus,
the pretence of kicking or punching the victim may be an attack, as will attempt to spit on the victim (aggressive
behaviour). All of the above elements must be present and the evidence must be supported with evidence if
found guilty for the attack. It can be di cult to prove whether the defendant actually intended the attack.
Similarly, judges often spend a lot of time determining whether a defendant's actions are considered harmful or
abusive. In determining this, they will consider what an average person may perceive as harmfulor aggressive.

Q.102 [30465398]
A was sitting in his backyard. B who was inside a passing train shoved his sts towards A citing he will bash
him. Did B commit assault on A.

a B did not commit assault as he was inside the passing train.

b B did not commit assault as there was no reasonable apprehension of attack nor an imminent threat.

c B committed assault on A irrespective of whether he was inside the train or not

https://www.aspiration.ai/LAW/sis/Solution.jsp?qsetId=JAmrn2p8H1A=&qsetName=LST Mock 27 2021 (CLAT) 169/223


1/17/2021 Mock Analysis

d B committed assault as he committed an act intending to cause hurt to A

Solution:
 Answer key/Solution
Correct Answer : b
Your Answer : b
Both a and b are correct answers, but b gives a more reasonable explanation. In
this case there was no reasonable apprehension as one could not assault a person from a moving train.

FeedBack Bookmark

https://www.aspiration.ai/LAW/sis/Solution.jsp?qsetId=JAmrn2p8H1A=&qsetName=LST Mock 27 2021 (CLAT) 170/223


1/17/2021 Mock Analysis

Directions for questions 67 to 105: You have been given some passages followed by questions based on each
passage. You are required to choose the most appropriate option which follows from the passage. Only the
information given in the passage should be used for choosing the answer and no external knowledge of law
howsoever prominent is to be applied.

Passage – 8

In common law, assault is a tort, an act of the defendant which causes to the plaintiff reasonable apprehension
of the in iction of a battery on him by the defendant. When the defendant creates his act by an apprehension in
the mind of the plaintiff that he is going to commit battery against the plaintiff, the wrong of assault is
completed. The wrong consists of an attempt to do harm rather than the harm being caused thereby. In assault
charges must include conduct that is offensive which is offensive or causes another person to the fear of their
safety. This clearly means that one can be guilty of assault even if he/she did not physically harm the victim. In
the case of R. v. S George, it was held that the pointing of loaded gun to another is an assault. If the pistol is not
loaded, then even it may be an assault, if pointed at such a distance that it may cause injury. if a person
advances the manner of threatening to use force, then there is assault. This was decided in the case of
Stephens v. Myers.

If one or more elements have not been satis ed then It can be a defence to an assault charge. Elements of the
crime of assault are:

An act or conduct intended to created: To prove a criminal attack, the defendants' behaviour must be motivated
to create a situation of fear or danger in the victim's mind. Accident acts do not include allegations of assault.

A reasonable apprehension: Further, the victim must reasonably believe that the defendant's conduct will harm
or humiliate him. The victim must understand the defendant's potentially harmful or offensive acts.

Of imminent harm: The victim's fear must be a direct response to a threat that is imminent. Future threats, such
as "I will beat you tomorrow, will not result in assault charges. In addition, there must be some kind of perceived
physical threat to the victim in the loss; For this reason, words by themselves generally do not constitute an
attack.

It is believed that the defendant's actions would cause physical danger or abusive behaviour to the victim. Thus,
the pretence of kicking or punching the victim may be an attack, as will attempt to spit on the victim (aggressive
behaviour). All of the above elements must be present and the evidence must be supported with evidence if
found guilty for the attack. It can be di cult to prove whether the defendant actually intended the attack.
Similarly, judges often spend a lot of time determining whether a defendant's actions are considered harmful or
abusive. In determining this, they will consider what an average person may perceive as harmfulor aggressive.

Q.103 [30465398]
A was walking in a street. Z starting running at A with an axe in his hand. A, escaped and led a suit for assault.
Decide

a Z will be liable for assault as all the requirements for assault are ful lled.

b Z will be not be liable for assault as there was no imminent harm present.

c Z will not be liable for assault but for battery.

https://www.aspiration.ai/LAW/sis/Solution.jsp?qsetId=JAmrn2p8H1A=&qsetName=LST Mock 27 2021 (CLAT) 171/223


1/17/2021 Mock Analysis

d This case cannot be tried under common law/law of torts.

Solution:
 Answer key/Solution
Correct Answer : a
Your Answer : a
In the given case all the necessary requirement of assault is ful lled. There is an
act, a reasonable apprehension and a threat of imminent harm. Therefore, option a is the correct answer.
FeedBack Bookmark

https://www.aspiration.ai/LAW/sis/Solution.jsp?qsetId=JAmrn2p8H1A=&qsetName=LST Mock 27 2021 (CLAT) 172/223


1/17/2021 Mock Analysis

Directions for questions 67 to 105: You have been given some passages followed by questions based on each
passage. You are required to choose the most appropriate option which follows from the passage. Only the
information given in the passage should be used for choosing the answer and no external knowledge of law
howsoever prominent is to be applied.

Passage – 8

In common law, assault is a tort, an act of the defendant which causes to the plaintiff reasonable apprehension
of the in iction of a battery on him by the defendant. When the defendant creates his act by an apprehension in
the mind of the plaintiff that he is going to commit battery against the plaintiff, the wrong of assault is
completed. The wrong consists of an attempt to do harm rather than the harm being caused thereby. In assault
charges must include conduct that is offensive which is offensive or causes another person to the fear of their
safety. This clearly means that one can be guilty of assault even if he/she did not physically harm the victim. In
the case of R. v. S George, it was held that the pointing of loaded gun to another is an assault. If the pistol is not
loaded, then even it may be an assault, if pointed at such a distance that it may cause injury. if a person
advances the manner of threatening to use force, then there is assault. This was decided in the case of
Stephens v. Myers.

If one or more elements have not been satis ed then It can be a defence to an assault charge. Elements of the
crime of assault are:

An act or conduct intended to created: To prove a criminal attack, the defendants' behaviour must be motivated
to create a situation of fear or danger in the victim's mind. Accident acts do not include allegations of assault.

A reasonable apprehension: Further, the victim must reasonably believe that the defendant's conduct will harm
or humiliate him. The victim must understand the defendant's potentially harmful or offensive acts.

Of imminent harm: The victim's fear must be a direct response to a threat that is imminent. Future threats, such
as "I will beat you tomorrow, will not result in assault charges. In addition, there must be some kind of perceived
physical threat to the victim in the loss; For this reason, words by themselves generally do not constitute an
attack.

It is believed that the defendant's actions would cause physical danger or abusive behaviour to the victim. Thus,
the pretence of kicking or punching the victim may be an attack, as will attempt to spit on the victim (aggressive
behaviour). All of the above elements must be present and the evidence must be supported with evidence if
found guilty for the attack. It can be di cult to prove whether the defendant actually intended the attack.
Similarly, judges often spend a lot of time determining whether a defendant's actions are considered harmful or
abusive. In determining this, they will consider what an average person may perceive as harmfulor aggressive.

Q.104 [30465398]
Puttu and Ramu were neighbours. They shared a common boundary. One day they had a dispute regarding the
land border. The matter accelerated and Puttu said he will bring a knife to kill Ramu and started walking towards
his house. Fearing of assault Ramu hit Puttu with a stone and ran away. Whether the fear of assault was
present.

a The fear of assault was present as Puttu was going to bring a knife. Ramu's act was to defend himself.

b The fear of assault was present as Puttu was about to commit a battery at Ramu.

https://www.aspiration.ai/LAW/sis/Solution.jsp?qsetId=JAmrn2p8H1A=&qsetName=LST Mock 27 2021 (CLAT) 173/223


1/17/2021 Mock Analysis

c The fear of assault was absent as there was no fear of imminent harm.

d Ramu instead of hitting with a stone should have approached a police station and led a complaint.

Solution:
 Answer key/Solution
Correct Answer : c
Your Answer : c
In the given case there was no imminent harm nor a reasonable apprehension of
fear of battery. Puttu was returning towards his house and was not attacking Ramu. Hence, there is no assault.

FeedBack Bookmark

https://www.aspiration.ai/LAW/sis/Solution.jsp?qsetId=JAmrn2p8H1A=&qsetName=LST Mock 27 2021 (CLAT) 174/223


1/17/2021 Mock Analysis

Directions for questions 67 to 105: You have been given some passages followed by questions based on each
passage. You are required to choose the most appropriate option which follows from the passage. Only the
information given in the passage should be used for choosing the answer and no external knowledge of law
howsoever prominent is to be applied.

Passage – 8

In common law, assault is a tort, an act of the defendant which causes to the plaintiff reasonable apprehension
of the in iction of a battery on him by the defendant. When the defendant creates his act by an apprehension in
the mind of the plaintiff that he is going to commit battery against the plaintiff, the wrong of assault is
completed. The wrong consists of an attempt to do harm rather than the harm being caused thereby. In assault
charges must include conduct that is offensive which is offensive or causes another person to the fear of their
safety. This clearly means that one can be guilty of assault even if he/she did not physically harm the victim. In
the case of R. v. S George, it was held that the pointing of loaded gun to another is an assault. If the pistol is not
loaded, then even it may be an assault, if pointed at such a distance that it may cause injury. if a person
advances the manner of threatening to use force, then there is assault. This was decided in the case of
Stephens v. Myers.

If one or more elements have not been satis ed then It can be a defence to an assault charge. Elements of the
crime of assault are:

An act or conduct intended to created: To prove a criminal attack, the defendants' behaviour must be motivated
to create a situation of fear or danger in the victim's mind. Accident acts do not include allegations of assault.

A reasonable apprehension: Further, the victim must reasonably believe that the defendant's conduct will harm
or humiliate him. The victim must understand the defendant's potentially harmful or offensive acts.

Of imminent harm: The victim's fear must be a direct response to a threat that is imminent. Future threats, such
as "I will beat you tomorrow, will not result in assault charges. In addition, there must be some kind of perceived
physical threat to the victim in the loss; For this reason, words by themselves generally do not constitute an
attack.

It is believed that the defendant's actions would cause physical danger or abusive behaviour to the victim. Thus,
the pretence of kicking or punching the victim may be an attack, as will attempt to spit on the victim (aggressive
behaviour). All of the above elements must be present and the evidence must be supported with evidence if
found guilty for the attack. It can be di cult to prove whether the defendant actually intended the attack.
Similarly, judges often spend a lot of time determining whether a defendant's actions are considered harmful or
abusive. In determining this, they will consider what an average person may perceive as harmfulor aggressive.

Q.105 [30465398]
What nature of the tort 'Assault' can be deduced from the case of R v. S George? (Above mentioned in passage)

a One can be charged with assault only when his act is capable of committing hurt to the other person.

b To charge a person with assault it is not necessary that his act may possibly lead to hurting others.
Reasonable apprehension is enough

https://www.aspiration.ai/LAW/sis/Solution.jsp?qsetId=JAmrn2p8H1A=&qsetName=LST Mock 27 2021 (CLAT) 175/223


1/17/2021 Mock Analysis

c To charge a person with assault he or she should have the intention to hurt. Without intention it is not
assault.

d Person who commits battery will necessarily also commit assault. Assault is a part of the battery.

Solution:
 Answer key/Solution
Correct Answer : b
Your Answer : b
In the case of R v. S George it is said pointing a gun which is not loaded will also
lead to an assault. But it should be at a reasonable distance from where a person can fear of getting injured.
Thus, 2nd option is correct.
FeedBack Bookmark

Sec 4
Directions for questions 106 to 130: Each set of questions in this section is based on the reasoning and
arguments set out in the preceding passage. Please answer each question on the basis of what is stated or
implied in the corresponding passage. Do not rely on any information or facts other than the ones supplied to
you. In some instances, more than one option may be the answer to the question; in such a case, please choose
the option that most accurately and comprehensively answers the question.

Passage – 1

The standoff between farmers and the government continues even after a few rounds of discussion and more
than a fortnight of protest at the Delhi border by the farmers. The latest proposal by the government indicates
its willingness to amend the three agriculture-related Acts passed in September. On the contentious issues of
registration of private traders, levy of taxes on trade outside the Agricultural Produce Market Committee (APMC)
mandis, the government has proposed amendments which will empower the States to frame rules on these
issues. Similar assurances have been given on access to the judiciary for dispute resolution and continuation of
the Minimum Support Price (MSP) mechanism. However, farmer unions have rejected the proposal and
continue to demand complete withdrawal of the three Acts along with making MSP a guarantee.

It is likely that the issue may ultimately get resolved and an amicable solution is found to resolve the impasse.
However, this will only be a temporary reprieve from the vexatious issue of declining farmer income and the
nature of state support to agriculture.

The real issue is the lack of remunerative prices for a majority of agricultural commodities, a sharp increase in
price variability in recent years, and an unpredictable and arbitrary government policy regime, none of which is
likely to change in the near future. It is these which have led to a recurrence of distress in the agricultural sector
with regular farmers' protests which have only grown in frequency in recent years.

https://www.aspiration.ai/LAW/sis/Solution.jsp?qsetId=JAmrn2p8H1A=&qsetName=LST Mock 27 2021 (CLAT) 176/223


1/17/2021 Mock Analysis

Q.106 [30465398]
What is the main idea of the given passage?

a The government has suggested the amendment of the three agricultural-related Acts passed in September.

b Farmers in India demand for the withdrawal of the agricultural-related Acts passed in September and the
guarantee of MSP.

c Any solution to the stalemate between farmers and the government might only be a temporary one.

d A multiple rounds of talks have taken place over the issues of the agricultural-related Acts passed in
September and MSP.

Solution:
 Answer key/Solution
Correct Answer : c
Your Answer : b
Options (a), (b) and (d) are all factually true according to the given passage,
however, the main idea of the passage is that even if a solution is found to the farmers' issues it is likely to be
a temporary reprieve from the vexatious issue of declining farmer income and the nature of state support to
agriculture. Hence, option (c) is the answer.

FeedBack Bookmark

https://www.aspiration.ai/LAW/sis/Solution.jsp?qsetId=JAmrn2p8H1A=&qsetName=LST Mock 27 2021 (CLAT) 177/223


1/17/2021 Mock Analysis

Directions for questions 106 to 130: Each set of questions in this section is based on the reasoning and
arguments set out in the preceding passage. Please answer each question on the basis of what is stated or
implied in the corresponding passage. Do not rely on any information or facts other than the ones supplied to
you. In some instances, more than one option may be the answer to the question; in such a case, please choose
the option that most accurately and comprehensively answers the question.

Passage – 1

The standoff between farmers and the government continues even after a few rounds of discussion and more
than a fortnight of protest at the Delhi border by the farmers. The latest proposal by the government indicates
its willingness to amend the three agriculture-related Acts passed in September. On the contentious issues of
registration of private traders, levy of taxes on trade outside the Agricultural Produce Market Committee (APMC)
mandis, the government has proposed amendments which will empower the States to frame rules on these
issues. Similar assurances have been given on access to the judiciary for dispute resolution and continuation of
the Minimum Support Price (MSP) mechanism. However, farmer unions have rejected the proposal and
continue to demand complete withdrawal of the three Acts along with making MSP a guarantee.

It is likely that the issue may ultimately get resolved and an amicable solution is found to resolve the impasse.
However, this will only be a temporary reprieve from the vexatious issue of declining farmer income and the
nature of state support to agriculture.

The real issue is the lack of remunerative prices for a majority of agricultural commodities, a sharp increase in
price variability in recent years, and an unpredictable and arbitrary government policy regime, none of which is
likely to change in the near future. It is these which have led to a recurrence of distress in the agricultural sector
with regular farmers' protests which have only grown in frequency in recent years.

Q.107 [30465398]
Which of the following statements would weaken the argument that "The real issue is the lack of remunerative
prices for a majority of agricultural commodities…"?

a The farmers have failed to gain much pro t in almost all the agricultural commodities they sell to the
government.

b The government has been buying all agricultural commodities at high prices from the farmers.

c The government's quoted prices for agricultural commodities have been lower than the farmers' quoted
prices.

d The farmers have been selling major agricultural commodities to the government at the prices set by the
government.

https://www.aspiration.ai/LAW/sis/Solution.jsp?qsetId=JAmrn2p8H1A=&qsetName=LST Mock 27 2021 (CLAT) 178/223


1/17/2021 Mock Analysis

Solution:
 Answer key/Solution
Correct Answer : b
Your Answer : a
Option (a) would strengthen the argument in the quoted sentence. Since it said in
the quoted sentence that the real issue is the lack of lucrative prices for a majority of agricultural
commodities, option (b) would weaken the argument provided in the quoted sentence. Options (c) and (d)
would strengthen the argument provided in the quoted sentence.
FeedBack Bookmark

Directions for questions 106 to 130: Each set of questions in this section is based on the reasoning and
arguments set out in the preceding passage. Please answer each question on the basis of what is stated or
implied in the corresponding passage. Do not rely on any information or facts other than the ones supplied to
you. In some instances, more than one option may be the answer to the question; in such a case, please choose
the option that most accurately and comprehensively answers the question.

Passage – 1

The standoff between farmers and the government continues even after a few rounds of discussion and more
than a fortnight of protest at the Delhi border by the farmers. The latest proposal by the government indicates
its willingness to amend the three agriculture-related Acts passed in September. On the contentious issues of
registration of private traders, levy of taxes on trade outside the Agricultural Produce Market Committee (APMC)
mandis, the government has proposed amendments which will empower the States to frame rules on these
issues. Similar assurances have been given on access to the judiciary for dispute resolution and continuation of
the Minimum Support Price (MSP) mechanism. However, farmer unions have rejected the proposal and
continue to demand complete withdrawal of the three Acts along with making MSP a guarantee.

It is likely that the issue may ultimately get resolved and an amicable solution is found to resolve the impasse.
However, this will only be a temporary reprieve from the vexatious issue of declining farmer income and the
nature of state support to agriculture.

The real issue is the lack of remunerative prices for a majority of agricultural commodities, a sharp increase in
price variability in recent years, and an unpredictable and arbitrary government policy regime, none of which is
likely to change in the near future. It is these which have led to a recurrence of distress in the agricultural sector
with regular farmers' protests which have only grown in frequency in recent years.

Q.108 [30465398]
Which of the following cannot be inferred from the passage above?

a The government had consulted the farmers before the agriculture-related Acts was passed in September.

b The farmers have felt that they have been neglected all these years.

c The government has not been able to make the farmers understand the new agriculture-related Acts
passed in September.

https://www.aspiration.ai/LAW/sis/Solution.jsp?qsetId=JAmrn2p8H1A=&qsetName=LST Mock 27 2021 (CLAT) 179/223


1/17/2021 Mock Analysis

d The government needs to nd a permanent solution to the problem that is collectively being faced by the
farmers in India.

Solution:
 Answer key/Solution
Correct Answer : a
Your Answer : a
Although it is mentioned in the passage that several rounds of talks/discussions
have taken place between the government and the farmers after the agriculture-related Act was passed in
September, nothing suggests that the government consulted the farmers before this Act was passed. Hence,
option (a) is the answer. The remaining options can be inferred from the given passage.
FeedBack Bookmark

Directions for questions 106 to 130: Each set of questions in this section is based on the reasoning and
arguments set out in the preceding passage. Please answer each question on the basis of what is stated or
implied in the corresponding passage. Do not rely on any information or facts other than the ones supplied to
you. In some instances, more than one option may be the answer to the question; in such a case, please choose
the option that most accurately and comprehensively answers the question.

Passage – 1

The standoff between farmers and the government continues even after a few rounds of discussion and more
than a fortnight of protest at the Delhi border by the farmers. The latest proposal by the government indicates
its willingness to amend the three agriculture-related Acts passed in September. On the contentious issues of
registration of private traders, levy of taxes on trade outside the Agricultural Produce Market Committee (APMC)
mandis, the government has proposed amendments which will empower the States to frame rules on these
issues. Similar assurances have been given on access to the judiciary for dispute resolution and continuation of
the Minimum Support Price (MSP) mechanism. However, farmer unions have rejected the proposal and
continue to demand complete withdrawal of the three Acts along with making MSP a guarantee.

It is likely that the issue may ultimately get resolved and an amicable solution is found to resolve the impasse.
However, this will only be a temporary reprieve from the vexatious issue of declining farmer income and the
nature of state support to agriculture.

The real issue is the lack of remunerative prices for a majority of agricultural commodities, a sharp increase in
price variability in recent years, and an unpredictable and arbitrary government policy regime, none of which is
likely to change in the near future. It is these which have led to a recurrence of distress in the agricultural sector
with regular farmers' protests which have only grown in frequency in recent years.

Q.109 [30465398]
Which of the following can be inferred from the passage above?

a A permanent solution to the farmers' problem is likely to be found at the earliest.

https://www.aspiration.ai/LAW/sis/Solution.jsp?qsetId=JAmrn2p8H1A=&qsetName=LST Mock 27 2021 (CLAT) 180/223


1/17/2021 Mock Analysis

b The farmers' problem in India is an inherent problem of the country.

c The farmers are not prepared to give in to the government's request to call off the protest that is being held
now.

d The government in the past was able to address the farmers' grievances.

Solution:
 Answer key/Solution
Correct Answer : b
Option(a) cannot be inferred. Option (b) can be inferred from the fact that the
farmers' grievances in India have not been addressed e ciently in India for a long
time. Hence, option (b) is the answer. Options (c) and (d) cannot be inferred.
FeedBack Bookmark

Directions for questions 106 to 130: Each set of questions in this section is based on the reasoning and
arguments set out in the preceding passage. Please answer each question on the basis of what is stated or
implied in the corresponding passage. Do not rely on any information or facts other than the ones supplied to
you. In some instances, more than one option may be the answer to the question; in such a case, please choose
the option that most accurately and comprehensively answers the question.

Passage – 1

The standoff between farmers and the government continues even after a few rounds of discussion and more
than a fortnight of protest at the Delhi border by the farmers. The latest proposal by the government indicates
its willingness to amend the three agriculture-related Acts passed in September. On the contentious issues of
registration of private traders, levy of taxes on trade outside the Agricultural Produce Market Committee (APMC)
mandis, the government has proposed amendments which will empower the States to frame rules on these
issues. Similar assurances have been given on access to the judiciary for dispute resolution and continuation of
the Minimum Support Price (MSP) mechanism. However, farmer unions have rejected the proposal and
continue to demand complete withdrawal of the three Acts along with making MSP a guarantee.

It is likely that the issue may ultimately get resolved and an amicable solution is found to resolve the impasse.
However, this will only be a temporary reprieve from the vexatious issue of declining farmer income and the
nature of state support to agriculture.

The real issue is the lack of remunerative prices for a majority of agricultural commodities, a sharp increase in
price variability in recent years, and an unpredictable and arbitrary government policy regime, none of which is
likely to change in the near future. It is these which have led to a recurrence of distress in the agricultural sector
with regular farmers' protests which have only grown in frequency in recent years.

Q.110 [30465398]
What can be inferred from the last paragraph of the given passage?

a Farmers are protesting as their distress has become completely unbearable.

https://www.aspiration.ai/LAW/sis/Solution.jsp?qsetId=JAmrn2p8H1A=&qsetName=LST Mock 27 2021 (CLAT) 181/223


1/17/2021 Mock Analysis

b The issues that farmers are facing can never be solved.

c The prices of many agricultural commodities have not been uctuating.

d Unless there is a rm agricultural policy that favours the farmers the issues of the farmers cannot be
resolved.

Solution:
 Answer key/Solution
Correct Answer : d
Your Answer : a
Only option (d) can be inferred from the last paragraph of the passage. Option (a)
is not suggested by anything in the given passage. Option (b) is extreme and cannot be inferred from the given
passage either. Option (c) is factually incorrect. Hence, option (d) is the answer.

FeedBack Bookmark

Directions for questions 106 to 130: Each set of questions in this section is based on the reasoning and
arguments set out in the preceding passage. Please answer each question on the basis of what is stated or
implied in the corresponding passage. Do not rely on any information or facts other than the ones supplied to
you. In some instances, more than one option may be the answer to the question; in such a case, please choose
the option that most accurately and comprehensively answers the question.

Passage – 2

With the surge of mechanisation, sanitation workers need to be reorganised on a large scale to reinstate a
permanent structure in the system. This labour concentration is required to weaken the solidarity of the
oppressors who also need severe mechanisation to x their caste bias. The rst question is whose capacity will
the Centre build when it seems they don't even know who these sanitation workers are? Their lack of
sociological understanding is evident in their failure of inclusion of sanitation workers in proposed plans
devised for their bene t. Another aspect is that if the number of workmen is unknown, any plans of direct
monetary support is far away from reality. On the ground the states have always refused to recognise the
existing categorisation of sanitation workers, therefore, when it comes to the work of scavenging only sewer
cleaners come to mind which is not always true. Therefore, the missing link between manhole to machine hole
transformation is a need for a mobilisation movement so that sanitation workers of different categories can be
at the front and centre of decision making. An alliance of all categories will not only help in their realistic
recognition but also protect their interests.

Q.111 [30465398]
In the light of the paragraph, it can be inferred that:

a The writer believes in exclusivity.

b The writer is a revolutionary.

https://www.aspiration.ai/LAW/sis/Solution.jsp?qsetId=JAmrn2p8H1A=&qsetName=LST Mock 27 2021 (CLAT) 182/223


1/17/2021 Mock Analysis

c The writer is a spiritually inclined man.

d The writer believes in inclusivity.

Solution:
 Answer key/Solution
Correct Answer : d
Your Answer : b
Refer to the second last sentence of the passage where the author mentions the
need to include sanitation workers in decision making. Therefore, option (d) is the most appropriate answer.
The remaining options are out of context.
FeedBack Bookmark

Directions for questions 106 to 130: Each set of questions in this section is based on the reasoning and
arguments set out in the preceding passage. Please answer each question on the basis of what is stated or
implied in the corresponding passage. Do not rely on any information or facts other than the ones supplied to
you. In some instances, more than one option may be the answer to the question; in such a case, please choose
the option that most accurately and comprehensively answers the question.

Passage – 2

With the surge of mechanisation, sanitation workers need to be reorganised on a large scale to reinstate a
permanent structure in the system. This labour concentration is required to weaken the solidarity of the
oppressors who also need severe mechanisation to x their caste bias. The rst question is whose capacity will
the Centre build when it seems they don't even know who these sanitation workers are? Their lack of
sociological understanding is evident in their failure of inclusion of sanitation workers in proposed plans
devised for their bene t. Another aspect is that if the number of workmen is unknown, any plans of direct
monetary support is far away from reality. On the ground the states have always refused to recognise the
existing categorisation of sanitation workers, therefore, when it comes to the work of scavenging only sewer
cleaners come to mind which is not always true. Therefore, the missing link between manhole to machine hole
transformation is a need for a mobilisation movement so that sanitation workers of different categories can be
at the front and centre of decision making. An alliance of all categories will not only help in their realistic
recognition but also protect their interests.

Q.112 [30465398]
Out of the following options, which one captures the central idea of the given paragraph?

a Sanitation workers and their little joys

b The family dynamics of sanitation workers

c Ways to improve the lot of sanitation workers

d Sanitation workers and scavengers

https://www.aspiration.ai/LAW/sis/Solution.jsp?qsetId=JAmrn2p8H1A=&qsetName=LST Mock 27 2021 (CLAT) 183/223


1/17/2021 Mock Analysis

Solution:
 Answer key/Solution
Correct Answer : c
Your Answer : c
The passage talks about the ways in which the lives of sanitation workers can be
improved. Hence, option (c) is the best possible answer.
FeedBack Bookmark

Directions for questions 106 to 130: Each set of questions in this section is based on the reasoning and
arguments set out in the preceding passage. Please answer each question on the basis of what is stated or
implied in the corresponding passage. Do not rely on any information or facts other than the ones supplied to
you. In some instances, more than one option may be the answer to the question; in such a case, please choose
the option that most accurately and comprehensively answers the question.

Passage – 2

With the surge of mechanisation, sanitation workers need to be reorganised on a large scale to reinstate a
permanent structure in the system. This labour concentration is required to weaken the solidarity of the
oppressors who also need severe mechanisation to x their caste bias. The rst question is whose capacity will
the Centre build when it seems they don't even know who these sanitation workers are? Their lack of
sociological understanding is evident in their failure of inclusion of sanitation workers in proposed plans
devised for their bene t. Another aspect is that if the number of workmen is unknown, any plans of direct
monetary support is far away from reality. On the ground the states have always refused to recognise the
existing categorisation of sanitation workers, therefore, when it comes to the work of scavenging only sewer
cleaners come to mind which is not always true. Therefore, the missing link between manhole to machine hole
transformation is a need for a mobilisation movement so that sanitation workers of different categories can be
at the front and centre of decision making. An alliance of all categories will not only help in their realistic
recognition but also protect their interests.

Q.113 [30465398]
The author of the passage is most likely to agree with which of the following statements:

a The caste biases in India were present in India even during the British times.

b The government needs to possess data before it can think of improving the lives of sanitation workers.

c The government needs to take the help of NGOs in order to ameliorate the conditions of the sanitation
workers.

d The government needs to involve social activists in order to ameliorate the conditions of the sanitation
workers.

https://www.aspiration.ai/LAW/sis/Solution.jsp?qsetId=JAmrn2p8H1A=&qsetName=LST Mock 27 2021 (CLAT) 184/223


1/17/2021 Mock Analysis

Solution:
 Answer key/Solution
Correct Answer : b
Your Answer : d
Refer to the sentence, "The rst question is whose capacity will the Centre build
when it seems they don't even know who these sanitation workers are?" Therefore, it is evident that without
data, any programme will be unsuccessful.
FeedBack Bookmark

Directions for questions 106 to 130: Each set of questions in this section is based on the reasoning and
arguments set out in the preceding passage. Please answer each question on the basis of what is stated or
implied in the corresponding passage. Do not rely on any information or facts other than the ones supplied to
you. In some instances, more than one option may be the answer to the question; in such a case, please choose
the option that most accurately and comprehensively answers the question.

Passage – 2

With the surge of mechanisation, sanitation workers need to be reorganised on a large scale to reinstate a
permanent structure in the system. This labour concentration is required to weaken the solidarity of the
oppressors who also need severe mechanisation to x their caste bias. The rst question is whose capacity will
the Centre build when it seems they don't even know who these sanitation workers are? Their lack of
sociological understanding is evident in their failure of inclusion of sanitation workers in proposed plans
devised for their bene t. Another aspect is that if the number of workmen is unknown, any plans of direct
monetary support is far away from reality. On the ground the states have always refused to recognise the
existing categorisation of sanitation workers, therefore, when it comes to the work of scavenging only sewer
cleaners come to mind which is not always true. Therefore, the missing link between manhole to machine hole
transformation is a need for a mobilisation movement so that sanitation workers of different categories can be
at the front and centre of decision making. An alliance of all categories will not only help in their realistic
recognition but also protect their interests.

Q.114 [30465398]
The author appears to believe that:

a Financial help can be one of the ways to help a marginalised individual.

b The condition of sanitation workers worsened recently because of economic recession.

c Sanitation workers have to join politics so that they can live a digni ed life.

d Sanitation workers have to be sensitised so that they don't discriminate against each other.

https://www.aspiration.ai/LAW/sis/Solution.jsp?qsetId=JAmrn2p8H1A=&qsetName=LST Mock 27 2021 (CLAT) 185/223


1/17/2021 Mock Analysis

Solution:
 Answer key/Solution
Correct Answer : a
Your Answer : c
Refer to the sentence, "Another aspect is that if the number of workmen is
unknown, any plans of direct monetary support is far away from reality."
FeedBack Bookmark

Directions for questions 106 to 130: Each set of questions in this section is based on the reasoning and
arguments set out in the preceding passage. Please answer each question on the basis of what is stated or
implied in the corresponding passage. Do not rely on any information or facts other than the ones supplied to
you. In some instances, more than one option may be the answer to the question; in such a case, please choose
the option that most accurately and comprehensively answers the question.

Passage – 2

With the surge of mechanisation, sanitation workers need to be reorganised on a large scale to reinstate a
permanent structure in the system. This labour concentration is required to weaken the solidarity of the
oppressors who also need severe mechanisation to x their caste bias. The rst question is whose capacity will
the Centre build when it seems they don't even know who these sanitation workers are? Their lack of
sociological understanding is evident in their failure of inclusion of sanitation workers in proposed plans
devised for their bene t. Another aspect is that if the number of workmen is unknown, any plans of direct
monetary support is far away from reality. On the ground the states have always refused to recognise the
existing categorisation of sanitation workers, therefore, when it comes to the work of scavenging only sewer
cleaners come to mind which is not always true. Therefore, the missing link between manhole to machine hole
transformation is a need for a mobilisation movement so that sanitation workers of different categories can be
at the front and centre of decision making. An alliance of all categories will not only help in their realistic
recognition but also protect their interests.

Q.115 [30465398]
In the light of the paragraph, it can be understood that:

a Hundreds of people perish while cleaning sewer tanks in India.

b Sewer cleaning is the most horri c form of manual scavenging.

c Other than sewer cleaners, there are other types of manual scavengers.

d Many kids are involved in manual scavenging in India.

https://www.aspiration.ai/LAW/sis/Solution.jsp?qsetId=JAmrn2p8H1A=&qsetName=LST Mock 27 2021 (CLAT) 186/223


1/17/2021 Mock Analysis

Solution:
 Answer key/Solution
Correct Answer : c
Your Answer : c
Refer to the sentence, "... when it comes to the work of scavenging only sewer
cleaners come to mind which is not always true." We don't know if sewer cleaning is the most horri c form of
manual scavenging. The remaining options are also untrue.
FeedBack Bookmark

Directions for questions 106 to 130: Each set of questions in this section is based on the reasoning and
arguments set out in the preceding passage. Please answer each question on the basis of what is stated or
implied in the corresponding passage. Do not rely on any information or facts other than the ones supplied to
you. In some instances, more than one option may be the answer to the question; in such a case, please choose
the option that most accurately and comprehensively answers the question.

Passage – 3

It is di cult to end this year on a celebratory note. If this year has taught us anything, it is to be conscious of our
mutual vulnerability. The COVID-19 pandemic has been primarily seen as a public health crisis. However, it is an
opportune moment to re ect upon the forgotten constitutional ideal of fraternity which is enshrined in the
Preamble. Fraternity may well be the only treatment for the multitude of challenges that this pandemic has
brought with itself.

So far, the policy response to the pandemic entails standard measures such as strengthening the public health
system and targeted economic support to the vulnerable sections. Fortunately, the pandemic has offered an
opportunity to rediscover our political and constitutional order by invoking the forgotten principle of fraternity. In
his closing speech to the Constituent Assembly, Bhimrao Ambedkar underlined the speci c purpose of inserting
"fraternity" in the constitutional text saying: "Liberty cannot be divorced from equality, equality cannot be
divorced from liberty. Nor can liberty and equality be divorced from fraternity." Unwittingly, Ambedkar may have
also predicted the fate of the fraternity as a principle. If we take the rst Republic day as our historical starting
point, there is little doubt that the Indian republic has made signi cant progress in its struggle for more liberty
and equality, but that progress towards more fraternity has been, to put it mildly, disappointing.

Q.116 [30465398]
The author of the paragraph is of the opinion that:

a Fraternity among Indians is not possible because of linguistic differences.

b Companionship among Indians is possible only if caste divisions are removed.

c True kinship among Indians is still wanting.

d Without B.R. Ambedkar, drafting the Indian Constitution would not have been possible.

https://www.aspiration.ai/LAW/sis/Solution.jsp?qsetId=JAmrn2p8H1A=&qsetName=LST Mock 27 2021 (CLAT) 187/223


1/17/2021 Mock Analysis

Solution:
 Answer key/Solution
Correct Answer : c
Your Answer : c
Refer to the last sentence of the passage where the author mentions that the
progress towards fraternity has been disappointing. The other options cannot be inferred from the given
paragraph.

FeedBack Bookmark

Directions for questions 106 to 130: Each set of questions in this section is based on the reasoning and
arguments set out in the preceding passage. Please answer each question on the basis of what is stated or
implied in the corresponding passage. Do not rely on any information or facts other than the ones supplied to
you. In some instances, more than one option may be the answer to the question; in such a case, please choose
the option that most accurately and comprehensively answers the question.

Passage – 3

It is di cult to end this year on a celebratory note. If this year has taught us anything, it is to be conscious of our
mutual vulnerability. The COVID-19 pandemic has been primarily seen as a public health crisis. However, it is an
opportune moment to re ect upon the forgotten constitutional ideal of fraternity which is enshrined in the
Preamble. Fraternity may well be the only treatment for the multitude of challenges that this pandemic has
brought with itself.

So far, the policy response to the pandemic entails standard measures such as strengthening the public health
system and targeted economic support to the vulnerable sections. Fortunately, the pandemic has offered an
opportunity to rediscover our political and constitutional order by invoking the forgotten principle of fraternity. In
his closing speech to the Constituent Assembly, Bhimrao Ambedkar underlined the speci c purpose of inserting
"fraternity" in the constitutional text saying: "Liberty cannot be divorced from equality, equality cannot be
divorced from liberty. Nor can liberty and equality be divorced from fraternity." Unwittingly, Ambedkar may have
also predicted the fate of the fraternity as a principle. If we take the rst Republic day as our historical starting
point, there is little doubt that the Indian republic has made signi cant progress in its struggle for more liberty
and equality, but that progress towards more fraternity has been, to put it mildly, disappointing.

Q.117 [30465398]
Out of the following options,which one captures the essence of the given paragraph?

a The importance of brotherhood

b The importance of helping the poor

c The importance of providing livelihood to the poor

d The importance of trust and belief

https://www.aspiration.ai/LAW/sis/Solution.jsp?qsetId=JAmrn2p8H1A=&qsetName=LST Mock 27 2021 (CLAT) 188/223


1/17/2021 Mock Analysis

Solution:
 Answer key/Solution
Correct Answer : a
Your Answer : a
The remaining options narrow in scope. The passage is about the importance of
fraternity or brotherhood.
FeedBack Bookmark

Directions for questions 106 to 130: Each set of questions in this section is based on the reasoning and
arguments set out in the preceding passage. Please answer each question on the basis of what is stated or
implied in the corresponding passage. Do not rely on any information or facts other than the ones supplied to
you. In some instances, more than one option may be the answer to the question; in such a case, please choose
the option that most accurately and comprehensively answers the question.

Passage – 3

It is di cult to end this year on a celebratory note. If this year has taught us anything, it is to be conscious of our
mutual vulnerability. The COVID-19 pandemic has been primarily seen as a public health crisis. However, it is an
opportune moment to re ect upon the forgotten constitutional ideal of fraternity which is enshrined in the
Preamble. Fraternity may well be the only treatment for the multitude of challenges that this pandemic has
brought with itself.

So far, the policy response to the pandemic entails standard measures such as strengthening the public health
system and targeted economic support to the vulnerable sections. Fortunately, the pandemic has offered an
opportunity to rediscover our political and constitutional order by invoking the forgotten principle of fraternity. In
his closing speech to the Constituent Assembly, Bhimrao Ambedkar underlined the speci c purpose of inserting
"fraternity" in the constitutional text saying: "Liberty cannot be divorced from equality, equality cannot be
divorced from liberty. Nor can liberty and equality be divorced from fraternity." Unwittingly, Ambedkar may have
also predicted the fate of the fraternity as a principle. If we take the rst Republic day as our historical starting
point, there is little doubt that the Indian republic has made signi cant progress in its struggle for more liberty
and equality, but that progress towards more fraternity has been, to put it mildly, disappointing.

Q.118 [30465398]
In the light of the paragraph, it can be inferred that:

a The marginalised sections of the society were rendered homeless during the pandemic.

b The marginalised sections of the society were provided assistance during the pandemic.

c The marginalised sections of the society were forced to return to their villages.

d The marginalised sections of the society were exploited by the administration.

https://www.aspiration.ai/LAW/sis/Solution.jsp?qsetId=JAmrn2p8H1A=&qsetName=LST Mock 27 2021 (CLAT) 189/223


1/17/2021 Mock Analysis

Solution:
 Answer key/Solution
Correct Answer : b
Your Answer : b
Refer to the sentence, "So far, the policy response to the pandemic entails
standard measures such as strengthening the public health system and targeted economic support to the
vulnerable sections."

FeedBack Bookmark

Directions for questions 106 to 130: Each set of questions in this section is based on the reasoning and
arguments set out in the preceding passage. Please answer each question on the basis of what is stated or
implied in the corresponding passage. Do not rely on any information or facts other than the ones supplied to
you. In some instances, more than one option may be the answer to the question; in such a case, please choose
the option that most accurately and comprehensively answers the question.

Passage – 3

It is di cult to end this year on a celebratory note. If this year has taught us anything, it is to be conscious of our
mutual vulnerability. The COVID-19 pandemic has been primarily seen as a public health crisis. However, it is an
opportune moment to re ect upon the forgotten constitutional ideal of fraternity which is enshrined in the
Preamble. Fraternity may well be the only treatment for the multitude of challenges that this pandemic has
brought with itself.

So far, the policy response to the pandemic entails standard measures such as strengthening the public health
system and targeted economic support to the vulnerable sections. Fortunately, the pandemic has offered an
opportunity to rediscover our political and constitutional order by invoking the forgotten principle of fraternity. In
his closing speech to the Constituent Assembly, Bhimrao Ambedkar underlined the speci c purpose of inserting
"fraternity" in the constitutional text saying: "Liberty cannot be divorced from equality, equality cannot be
divorced from liberty. Nor can liberty and equality be divorced from fraternity." Unwittingly, Ambedkar may have
also predicted the fate of the fraternity as a principle. If we take the rst Republic day as our historical starting
point, there is little doubt that the Indian republic has made signi cant progress in its struggle for more liberty
and equality, but that progress towards more fraternity has been, to put it mildly, disappointing.

Q.119 [30465398]
Which of the following options, if true, could challenge the author's belief as expressed in the paragraph?

a Indians, in general, love and worship nature.

b Indians are spiritual by nature and are often found worshipping different Gods and Goddesses.

c The Indian society is riddled with casteist politics and rifts.

d Whenever a natural crisis happens in India, people across social strata join hands in supporting one
another.

https://www.aspiration.ai/LAW/sis/Solution.jsp?qsetId=JAmrn2p8H1A=&qsetName=LST Mock 27 2021 (CLAT) 190/223


1/17/2021 Mock Analysis

Solution:
 Answer key/Solution
Correct Answer : d
Refer to the last sentence of the paragraph where the author mentions that
fraternity is disappointing. Therefore, if option (d) were true, it would challenge
the author's belief as expressed in the paragraph.
FeedBack Bookmark

Directions for questions 106 to 130: Each set of questions in this section is based on the reasoning and
arguments set out in the preceding passage. Please answer each question on the basis of what is stated or
implied in the corresponding passage. Do not rely on any information or facts other than the ones supplied to
you. In some instances, more than one option may be the answer to the question; in such a case, please choose
the option that most accurately and comprehensively answers the question.

Passage – 3

It is di cult to end this year on a celebratory note. If this year has taught us anything, it is to be conscious of our
mutual vulnerability. The COVID-19 pandemic has been primarily seen as a public health crisis. However, it is an
opportune moment to re ect upon the forgotten constitutional ideal of fraternity which is enshrined in the
Preamble. Fraternity may well be the only treatment for the multitude of challenges that this pandemic has
brought with itself.

So far, the policy response to the pandemic entails standard measures such as strengthening the public health
system and targeted economic support to the vulnerable sections. Fortunately, the pandemic has offered an
opportunity to rediscover our political and constitutional order by invoking the forgotten principle of fraternity. In
his closing speech to the Constituent Assembly, Bhimrao Ambedkar underlined the speci c purpose of inserting
"fraternity" in the constitutional text saying: "Liberty cannot be divorced from equality, equality cannot be
divorced from liberty. Nor can liberty and equality be divorced from fraternity." Unwittingly, Ambedkar may have
also predicted the fate of the fraternity as a principle. If we take the rst Republic day as our historical starting
point, there is little doubt that the Indian republic has made signi cant progress in its struggle for more liberty
and equality, but that progress towards more fraternity has been, to put it mildly, disappointing.

Q.120 [30465398]
In the light of the passage, it can be understood that:

a Liberty is more important than fraternity.

b Fraternity can pose a threat to regional political parties.

c Liberty, equality and fraternity are interdependent.

d Liberty, equality and fraternity are exclusive entities.

https://www.aspiration.ai/LAW/sis/Solution.jsp?qsetId=JAmrn2p8H1A=&qsetName=LST Mock 27 2021 (CLAT) 191/223


1/17/2021 Mock Analysis

Solution:
 Answer key/Solution
Correct Answer : c
Your Answer : d
Refer to the sentence, "Liberty cannot be divorced from equality, equality cannot
be divorced from liberty. Nor can liberty and equality be divorced from fraternity."
FeedBack Bookmark

Directions for questions 106 to 130: Each set of questions in this section is based on the reasoning and
arguments set out in the preceding passage. Please answer each question on the basis of what is stated or
implied in the corresponding passage. Do not rely on any information or facts other than the ones supplied to
you. In some instances, more than one option may be the answer to the question; in such a case, please choose
the option that most accurately and comprehensively answers the question.

Passage – 4

The electric vehicles market in India appears to be gaining traction. Companies have not only started to
manufacture electric scooters and bikes in states like Uttar Pradesh but have also begun to research for
alternatives to lithiumion batteries, which are mostly imported from China. EV major Tesla is expected to launch
its India operations early next year, signalling the market's attractiveness. In due course of time, with more
money owing into research, India could emerge as the number one manufacturer of electric vehicles like cars,
bikes, and tractors, said Nitin Gadkari, Minister of Road Transport and Highways, at the Indian Express Idea
Exchange programme. These are welcome developments and signal the long-term attractiveness of the EV
market segment in India. The arguments in favour of a shift towards electric vehicles are straightforward,
arising largely as a response to the challenges of pollution and energy security. India has 21 of the world's 30
cities with the worst air pollution as per data from IQAir AirVisual's 2019 World Air Quality Report. And much of
the pollution load can be traced to vehicular emissions. As such, the adoption of electric vehicles will reduce
overall emissions and also help meet the targets under the Paris agreement. Such a shift would also help lower
the country's dependence on oil imports. A NITI Aayog report had earlier pegged the savings in the oil import bill
at Rs 1.2 lakh crore Considering such a shift will lead to enormous socio-economic and environmental bene ts,
a prudent approach would be to channel investment towards facilitating the shift of various modes of public
transport, two-wheelers and taxis to electric vehicles. Covering these modes of transport itself would include
nearly 80 per cent of vehicles on Indian roads as per the NITI Aayog study.

Q.121 [30465398]
Which of the following is the main idea of the given passage?

a Several companies have begun to manufacture electric vehicles in India.

b Vehicles that run on fossil fuel are the major cause for carbon emission in India.

c India is a potential market for electric vehicles.

d Switching over to electric vehicles will help in reducing the level of carbon emission.

https://www.aspiration.ai/LAW/sis/Solution.jsp?qsetId=JAmrn2p8H1A=&qsetName=LST Mock 27 2021 (CLAT) 192/223


1/17/2021 Mock Analysis

Solution:
 Answer key/Solution
Correct Answer : c
Your Answer : c
Option (a) is mentioned in the given passage but it is one aspect of the given
passage and cannot be the main idea of it. Option (b) is factually correct according to the given passage,
however, it cannot be the main idea of the passage either. Option (c) is the main idea of the passage. Option
(d) is also just an aspect of the given passage.
FeedBack Bookmark

Directions for questions 106 to 130: Each set of questions in this section is based on the reasoning and
arguments set out in the preceding passage. Please answer each question on the basis of what is stated or
implied in the corresponding passage. Do not rely on any information or facts other than the ones supplied to
you. In some instances, more than one option may be the answer to the question; in such a case, please choose
the option that most accurately and comprehensively answers the question.

Passage – 4

The electric vehicles market in India appears to be gaining traction. Companies have not only started to
manufacture electric scooters and bikes in states like Uttar Pradesh but have also begun to research for
alternatives to lithiumion batteries, which are mostly imported from China. EV major Tesla is expected to launch
its India operations early next year, signalling the market's attractiveness. In due course of time, with more
money owing into research, India could emerge as the number one manufacturer of electric vehicles like cars,
bikes, and tractors, said Nitin Gadkari, Minister of Road Transport and Highways, at the Indian Express Idea
Exchange programme. These are welcome developments and signal the long-term attractiveness of the EV
market segment in India. The arguments in favour of a shift towards electric vehicles are straightforward,
arising largely as a response to the challenges of pollution and energy security. India has 21 of the world's 30
cities with the worst air pollution as per data from IQAir AirVisual's 2019 World Air Quality Report. And much of
the pollution load can be traced to vehicular emissions. As such, the adoption of electric vehicles will reduce
overall emissions and also help meet the targets under the Paris agreement. Such a shift would also help lower
the country's dependence on oil imports. A NITI Aayog report had earlier pegged the savings in the oil import bill
at Rs 1.2 lakh crore Considering such a shift will lead to enormous socio-economic and environmental bene ts,
a prudent approach would be to channel investment towards facilitating the shift of various modes of public
transport, two-wheelers and taxis to electric vehicles. Covering these modes of transport itself would include
nearly 80 per cent of vehicles on Indian roads as per the NITI Aayog study.

Q.122 [30465398]
Which of the following weakens the author's argument in the given passage?

a Switching over to electric vehicles will decrease India's dependence on oil imports.

b Carbon emission is a big problem in India.

c Air quality in many Indian cities is extremely bad.

d Switching over to electric vehicles will not decrease India's dependence on oil imports.

https://www.aspiration.ai/LAW/sis/Solution.jsp?qsetId=JAmrn2p8H1A=&qsetName=LST Mock 27 2021 (CLAT) 193/223


1/17/2021 Mock Analysis

Solution:
 Answer key/Solution
Correct Answer : d
Your Answer : d
Option (d) weakens the author's argument in the given passage because it is said
in the passage that a shift to electric vehicles would also help lower the country's dependence on oil imports.
Options (a), (b) and (c) would strengthen the author's argument.
FeedBack Bookmark

Directions for questions 106 to 130: Each set of questions in this section is based on the reasoning and
arguments set out in the preceding passage. Please answer each question on the basis of what is stated or
implied in the corresponding passage. Do not rely on any information or facts other than the ones supplied to
you. In some instances, more than one option may be the answer to the question; in such a case, please choose
the option that most accurately and comprehensively answers the question.

Passage – 4

The electric vehicles market in India appears to be gaining traction. Companies have not only started to
manufacture electric scooters and bikes in states like Uttar Pradesh but have also begun to research for
alternatives to lithiumion batteries, which are mostly imported from China. EV major Tesla is expected to launch
its India operations early next year, signalling the market's attractiveness. In due course of time, with more
money owing into research, India could emerge as the number one manufacturer of electric vehicles like cars,
bikes, and tractors, said Nitin Gadkari, Minister of Road Transport and Highways, at the Indian Express Idea
Exchange programme. These are welcome developments and signal the long-term attractiveness of the EV
market segment in India. The arguments in favour of a shift towards electric vehicles are straightforward,
arising largely as a response to the challenges of pollution and energy security. India has 21 of the world's 30
cities with the worst air pollution as per data from IQAir AirVisual's 2019 World Air Quality Report. And much of
the pollution load can be traced to vehicular emissions. As such, the adoption of electric vehicles will reduce
overall emissions and also help meet the targets under the Paris agreement. Such a shift would also help lower
the country's dependence on oil imports. A NITI Aayog report had earlier pegged the savings in the oil import bill
at Rs 1.2 lakh crore Considering such a shift will lead to enormous socio-economic and environmental bene ts,
a prudent approach would be to channel investment towards facilitating the shift of various modes of public
transport, two-wheelers and taxis to electric vehicles. Covering these modes of transport itself would include
nearly 80 per cent of vehicles on Indian roads as per the NITI Aayog study.

Q.123 [30465398]
Which of the following can be inferred from the passage above?

a Tesla will compete with domestic manufacturers of electric vehicles in India.

b India is not likely to be a potential market for electric vehicles.

c The move to gradually switch over from vehicles that run on fossil fuel to electric vehicles has begun in
India.

https://www.aspiration.ai/LAW/sis/Solution.jsp?qsetId=JAmrn2p8H1A=&qsetName=LST Mock 27 2021 (CLAT) 194/223


1/17/2021 Mock Analysis

d Tesla has not launched any of its business in India yet.

Solution:
 Answer key/Solution
Correct Answer : c
Your Answer : c
Option (c) can be inferred from "The electric vehicles market in India appears to
be gaining traction." Hence, option (c) is the answer. Although it is said that electric vehicles are now being
manufactured in Uttar Pradesh, it cannot be ascertained that the companies manufacturing electric vehicles in
UP are domestic companies. So, option (a) cannot be inferred. Option (b) is contradictory to the information
mentioned in the given passage. Option (d) cannot be inferred from the given passage.
FeedBack Bookmark

Directions for questions 106 to 130: Each set of questions in this section is based on the reasoning and
arguments set out in the preceding passage. Please answer each question on the basis of what is stated or
implied in the corresponding passage. Do not rely on any information or facts other than the ones supplied to
you. In some instances, more than one option may be the answer to the question; in such a case, please choose
the option that most accurately and comprehensively answers the question.

Passage – 4

The electric vehicles market in India appears to be gaining traction. Companies have not only started to
manufacture electric scooters and bikes in states like Uttar Pradesh but have also begun to research for
alternatives to lithiumion batteries, which are mostly imported from China. EV major Tesla is expected to launch
its India operations early next year, signalling the market's attractiveness. In due course of time, with more
money owing into research, India could emerge as the number one manufacturer of electric vehicles like cars,
bikes, and tractors, said Nitin Gadkari, Minister of Road Transport and Highways, at the Indian Express Idea
Exchange programme. These are welcome developments and signal the long-term attractiveness of the EV
market segment in India. The arguments in favour of a shift towards electric vehicles are straightforward,
arising largely as a response to the challenges of pollution and energy security. India has 21 of the world's 30
cities with the worst air pollution as per data from IQAir AirVisual's 2019 World Air Quality Report. And much of
the pollution load can be traced to vehicular emissions. As such, the adoption of electric vehicles will reduce
overall emissions and also help meet the targets under the Paris agreement. Such a shift would also help lower
the country's dependence on oil imports. A NITI Aayog report had earlier pegged the savings in the oil import bill
at Rs 1.2 lakh crore Considering such a shift will lead to enormous socio-economic and environmental bene ts,
a prudent approach would be to channel investment towards facilitating the shift of various modes of public
transport, two-wheelers and taxis to electric vehicles. Covering these modes of transport itself would include
nearly 80 per cent of vehicles on Indian roads as per the NITI Aayog study.

Q.124 [30465398]
It can be inferred from the given passage that:

a Not many foreign companies will be given permission to manufacture electric vehicles in India.

https://www.aspiration.ai/LAW/sis/Solution.jsp?qsetId=JAmrn2p8H1A=&qsetName=LST Mock 27 2021 (CLAT) 195/223


1/17/2021 Mock Analysis

b EV major Tesla is the only foreign company to be allowed to market its electric vehicle in India.

c India imports lithium-ion batteries from other countries.

d India will attract several manufacturers of electric vehicles from abroad.

Solution:
 Answer key/Solution
Correct Answer : c
Your Answer : c
Option (c) can be inferred from "…lithium-ion batteries … are mostly imported
from China." The quoted line suggests that India imports lithium-ion batteries not only from China. Options (a)
and (b) cannot be inferred from the given passage. It is said that EV major Tesla is expected to launch its India
operations early next year, however, it cannot be inferred that India will attract several manufacturers of
electric vehicles from abroad. So, option (d) is incorrect.
FeedBack Bookmark

Directions for questions 106 to 130: Each set of questions in this section is based on the reasoning and
arguments set out in the preceding passage. Please answer each question on the basis of what is stated or
implied in the corresponding passage. Do not rely on any information or facts other than the ones supplied to
you. In some instances, more than one option may be the answer to the question; in such a case, please choose
the option that most accurately and comprehensively answers the question.

Passage – 4

The electric vehicles market in India appears to be gaining traction. Companies have not only started to
manufacture electric scooters and bikes in states like Uttar Pradesh but have also begun to research for
alternatives to lithiumion batteries, which are mostly imported from China. EV major Tesla is expected to launch
its India operations early next year, signalling the market's attractiveness. In due course of time, with more
money owing into research, India could emerge as the number one manufacturer of electric vehicles like cars,
bikes, and tractors, said Nitin Gadkari, Minister of Road Transport and Highways, at the Indian Express Idea
Exchange programme. These are welcome developments and signal the long-term attractiveness of the EV
market segment in India. The arguments in favour of a shift towards electric vehicles are straightforward,
arising largely as a response to the challenges of pollution and energy security. India has 21 of the world's 30
cities with the worst air pollution as per data from IQAir AirVisual's 2019 World Air Quality Report. And much of
the pollution load can be traced to vehicular emissions. As such, the adoption of electric vehicles will reduce
overall emissions and also help meet the targets under the Paris agreement. Such a shift would also help lower
the country's dependence on oil imports. A NITI Aayog report had earlier pegged the savings in the oil import bill
at Rs 1.2 lakh crore Considering such a shift will lead to enormous socio-economic and environmental bene ts,
a prudent approach would be to channel investment towards facilitating the shift of various modes of public
transport, two-wheelers and taxis to electric vehicles. Covering these modes of transport itself would include
nearly 80 per cent of vehicles on Indian roads as per the NITI Aayog study.

https://www.aspiration.ai/LAW/sis/Solution.jsp?qsetId=JAmrn2p8H1A=&qsetName=LST Mock 27 2021 (CLAT) 196/223


1/17/2021 Mock Analysis

Q.125 [30465398]
Which of the following cannot be inferred from the given passage?

a Indian government is serious about manufacturing of electric vehicles in India.

b India will compete with China in manufacturing lithium-ion batteries.

c Currently India is unable to meet the targets of Paris Agreement.

d Public transport, two-wheelers and taxis comprise more than 70 percent of the vehicles on the Indian roads.

Solution:
 Answer key/Solution
Correct Answer : b
Your Answer : c
Option (a) can be inferred from "In due course of time, with more money owing
into research, India could emerge as the number one manufacturer of electric vehicles like cars, bikes, and
tractors, said Nitin Gadkari, Minister of Road Transport and Highways, at the Indian Express Idea Exchange
programme…" It is said in the given passage that India exports lithium-ion batteries from China and it is also
said that companies have begun to research for alternatives to lithium-ion batteries, however, it cannot be
inferred from the given passage that India will compete with China in manufacturing lithium-ion batteries. So,
option (b) is the correct answer. Option (c) can be inferred from "As such, the adoption of electric vehicles will
reduce overall emissions and also help meet the targets under the Paris agreement." Option (d) can be inferred
from "…various modes of public transport, two-wheelers and taxis to electric vehicles. Covering these modes
of transport itself would include nearly 80 per cent of vehicles on Indian roads…"
FeedBack Bookmark

https://www.aspiration.ai/LAW/sis/Solution.jsp?qsetId=JAmrn2p8H1A=&qsetName=LST Mock 27 2021 (CLAT) 197/223


1/17/2021 Mock Analysis

Directions for questions 106 to 130: Each set of questions in this section is based on the reasoning and
arguments set out in the preceding passage. Please answer each question on the basis of what is stated or
implied in the corresponding passage. Do not rely on any information or facts other than the ones supplied to
you. In some instances, more than one option may be the answer to the question; in such a case, please choose
the option that most accurately and comprehensively answers the question.

Passage – 5

2020 has been no ordinary year - the Chinese have ensured that the world transits from the physical to a digital
domain. The novel coronavirus has pushed people and businesses to "work from home". Internet providers are
the busiest - connecting people. Militaries have been pushed to the borders, treaties and agreements are being
signed, and a record number of military deals have happened. This year has witnessed the most unprecedented
intensi cation of global military con icts since the Gulf War. Nations are showing their military intent. AI
applications have been at display in warfare, with drone killing machines being advertised. There is no option
left but to get the 5G technology now. The Chinese seem to have cast a die to capture the world virtually.

The good news is that India is likely to survive the onslaught of the Chinese 5G invasion if it accelerates the
launch of Indian 5G. India is working on technologies that would enable it to launch Indigenous 5G that would
run IOT platforms for civilian as well as military applications. The banning of Chinese apps and blocking of
hardware supply chains would be the correct counteroffensive to protect the business and security interests of
the country. The problem is India being poor in "implementation".

Our scientists and industry can bring technology at a winning speed, but where India starts losing out is in slow
adoption, getting entangled in policy processes and the crosshairs of the bureaucracy. The current dispensation
in Delhi realises the importance of winning this war.

India must get its timing right. The implementation of 5G, though a bit delayed, can make India a good
alternative to China. But agreements like RCEP and China's other debt strategies will remain a larger threat to
the world.

Q.126 [30465398]
Which of the following summarizes the main idea behind the passage?

a India is working on technologies that will facilitate the launch of its domestic 5G.

b China is a real threat to the world.

c India needs to launch 5G quickly if it wants to stay ahead of China in the 5G game.

d The world has moved from physical to digital domain.

https://www.aspiration.ai/LAW/sis/Solution.jsp?qsetId=JAmrn2p8H1A=&qsetName=LST Mock 27 2021 (CLAT) 198/223


1/17/2021 Mock Analysis

Solution:
 Answer key/Solution
Correct Answer : c
Options (a), (b) and (d) are true according to the given passage, however, they are
the aspects of the passage. Only option (c) summarizes the main idea of the
given passage. Refer to "The current dispensation in Delhi realises the importance of winning this war." Hence,
option (c) is the answer.

FeedBack Bookmark

Directions for questions 106 to 130: Each set of questions in this section is based on the reasoning and
arguments set out in the preceding passage. Please answer each question on the basis of what is stated or
implied in the corresponding passage. Do not rely on any information or facts other than the ones supplied to
you. In some instances, more than one option may be the answer to the question; in such a case, please choose
the option that most accurately and comprehensively answers the question.

Passage – 5

2020 has been no ordinary year - the Chinese have ensured that the world transits from the physical to a digital
domain. The novel coronavirus has pushed people and businesses to "work from home". Internet providers are
the busiest - connecting people. Militaries have been pushed to the borders, treaties and agreements are being
signed, and a record number of military deals have happened. This year has witnessed the most unprecedented
intensi cation of global military con icts since the Gulf War. Nations are showing their military intent. AI
applications have been at display in warfare, with drone killing machines being advertised. There is no option
left but to get the 5G technology now. The Chinese seem to have cast a die to capture the world virtually.

The good news is that India is likely to survive the onslaught of the Chinese 5G invasion if it accelerates the
launch of Indian 5G. India is working on technologies that would enable it to launch Indigenous 5G that would
run IOT platforms for civilian as well as military applications. The banning of Chinese apps and blocking of
hardware supply chains would be the correct counteroffensive to protect the business and security interests of
the country. The problem is India being poor in "implementation".

Our scientists and industry can bring technology at a winning speed, but where India starts losing out is in slow
adoption, getting entangled in policy processes and the crosshairs of the bureaucracy. The current dispensation
in Delhi realises the importance of winning this war.

India must get its timing right. The implementation of 5G, though a bit delayed, can make India a good
alternative to China. But agreements like RCEP and China's other debt strategies will remain a larger threat to
the world.

Q.127 [30465398]
Which of the following can be inferred from the passage above?

a India has accelerated the process for launching 5G.

b Red tapism in India slows down the launch of 5G.

https://www.aspiration.ai/LAW/sis/Solution.jsp?qsetId=JAmrn2p8H1A=&qsetName=LST Mock 27 2021 (CLAT) 199/223


1/17/2021 Mock Analysis

c The Chinese 5G is banned in India.

d In India, 5G will be rst used for military purpose.

Solution:
 Answer key/Solution
Correct Answer : b
Your Answer : b
Refer to "…but where India starts losing out is in slow adoption, getting entangled
in policy processes and the crosshairs of the bureaucracy." Hence, option (b) is the answer. Options (a), (c) and
(d) cannot be inferred from the given passage.

FeedBack Bookmark

Directions for questions 106 to 130: Each set of questions in this section is based on the reasoning and
arguments set out in the preceding passage. Please answer each question on the basis of what is stated or
implied in the corresponding passage. Do not rely on any information or facts other than the ones supplied to
you. In some instances, more than one option may be the answer to the question; in such a case, please choose
the option that most accurately and comprehensively answers the question.

Passage – 5

2020 has been no ordinary year - the Chinese have ensured that the world transits from the physical to a digital
domain. The novel coronavirus has pushed people and businesses to "work from home". Internet providers are
the busiest - connecting people. Militaries have been pushed to the borders, treaties and agreements are being
signed, and a record number of military deals have happened. This year has witnessed the most unprecedented
intensi cation of global military con icts since the Gulf War. Nations are showing their military intent. AI
applications have been at display in warfare, with drone killing machines being advertised. There is no option
left but to get the 5G technology now. The Chinese seem to have cast a die to capture the world virtually.

The good news is that India is likely to survive the onslaught of the Chinese 5G invasion if it accelerates the
launch of Indian 5G. India is working on technologies that would enable it to launch Indigenous 5G that would
run IOT platforms for civilian as well as military applications. The banning of Chinese apps and blocking of
hardware supply chains would be the correct counteroffensive to protect the business and security interests of
the country. The problem is India being poor in "implementation".

Our scientists and industry can bring technology at a winning speed, but where India starts losing out is in slow
adoption, getting entangled in policy processes and the crosshairs of the bureaucracy. The current dispensation
in Delhi realises the importance of winning this war.

India must get its timing right. The implementation of 5G, though a bit delayed, can make India a good
alternative to China. But agreements like RCEP and China's other debt strategies will remain a larger threat to
the world.

https://www.aspiration.ai/LAW/sis/Solution.jsp?qsetId=JAmrn2p8H1A=&qsetName=LST Mock 27 2021 (CLAT) 200/223


1/17/2021 Mock Analysis

Q.128 [30465398]
Which of the following strengthens the author's argument?

a China appears to be bent on dominating the world virtually.

b China does not envision itself as a world leader.

c China does not seem to have a plan to spread its 5G to the world.

d China's 5G will be used only in China.

Solution:
 Answer key/Solution
Correct Answer : a
Your Answer : a
Refer to "…the Chinese have ensured that the world transits from the physical to a
digital domain…" and "The Chinese seem to have cast a die to capture the world virtually." Thus, option (a)
strengthens the author's argument. Other options would weaken the author's argument.
FeedBack Bookmark

https://www.aspiration.ai/LAW/sis/Solution.jsp?qsetId=JAmrn2p8H1A=&qsetName=LST Mock 27 2021 (CLAT) 201/223


1/17/2021 Mock Analysis

Directions for questions 106 to 130: Each set of questions in this section is based on the reasoning and
arguments set out in the preceding passage. Please answer each question on the basis of what is stated or
implied in the corresponding passage. Do not rely on any information or facts other than the ones supplied to
you. In some instances, more than one option may be the answer to the question; in such a case, please choose
the option that most accurately and comprehensively answers the question.

Passage – 5

2020 has been no ordinary year - the Chinese have ensured that the world transits from the physical to a digital
domain. The novel coronavirus has pushed people and businesses to "work from home". Internet providers are
the busiest - connecting people. Militaries have been pushed to the borders, treaties and agreements are being
signed, and a record number of military deals have happened. This year has witnessed the most unprecedented
intensi cation of global military con icts since the Gulf War. Nations are showing their military intent. AI
applications have been at display in warfare, with drone killing machines being advertised. There is no option
left but to get the 5G technology now. The Chinese seem to have cast a die to capture the world virtually.

The good news is that India is likely to survive the onslaught of the Chinese 5G invasion if it accelerates the
launch of Indian 5G. India is working on technologies that would enable it to launch Indigenous 5G that would
run IOT platforms for civilian as well as military applications. The banning of Chinese apps and blocking of
hardware supply chains would be the correct counteroffensive to protect the business and security interests of
the country. The problem is India being poor in "implementation".

Our scientists and industry can bring technology at a winning speed, but where India starts losing out is in slow
adoption, getting entangled in policy processes and the crosshairs of the bureaucracy. The current dispensation
in Delhi realises the importance of winning this war.

India must get its timing right. The implementation of 5G, though a bit delayed, can make India a good
alternative to China. But agreements like RCEP and China's other debt strategies will remain a larger threat to
the world.

Q.129 [30465398]
Which of the following cannot be inferred from the given passage?

a China has a plan to spread 5G network that will encompass the world.

b Work from home has become a new working norm in 2020.

c India possesses a more sophisticated technology to launch 5G than the Chinese technology.

d India is conscious of the fact that the 5G war is a high-stake war for India.

https://www.aspiration.ai/LAW/sis/Solution.jsp?qsetId=JAmrn2p8H1A=&qsetName=LST Mock 27 2021 (CLAT) 202/223


1/17/2021 Mock Analysis

Solution:
 Answer key/Solution
Correct Answer : c
Your Answer : c
Option (a) can be inferred from "There is no option left but to get the 5G
technology now. The Chinese seem to have cast a die to capture the world virtually…" and "…the onslaught of
the Chinese 5G invasion…" Option (b) can be inferred from "2020 has been no ordinary year …. coronavirus has
pushed people and businesses to "work from home". Option (d) can be inferred from "The current dispensation
in Delhi realises the importance of winning this war." Although it is said that Indian scientists and industry can
bring technology at a winning speed, it cannot be inferred that India possesses a more sophisticated
technology to launch 5G than the Chinese technology. Moreover, it is said that India is working on
technologies that would enable it to launch Indigenous 5G. Hence, option (c) is the answer.

FeedBack Bookmark

Directions for questions 106 to 130: Each set of questions in this section is based on the reasoning and
arguments set out in the preceding passage. Please answer each question on the basis of what is stated or
implied in the corresponding passage. Do not rely on any information or facts other than the ones supplied to
you. In some instances, more than one option may be the answer to the question; in such a case, please choose
the option that most accurately and comprehensively answers the question.

Passage – 5

2020 has been no ordinary year - the Chinese have ensured that the world transits from the physical to a digital
domain. The novel coronavirus has pushed people and businesses to "work from home". Internet providers are
the busiest - connecting people. Militaries have been pushed to the borders, treaties and agreements are being
signed, and a record number of military deals have happened. This year has witnessed the most unprecedented
intensi cation of global military con icts since the Gulf War. Nations are showing their military intent. AI
applications have been at display in warfare, with drone killing machines being advertised. There is no option
left but to get the 5G technology now. The Chinese seem to have cast a die to capture the world virtually.

The good news is that India is likely to survive the onslaught of the Chinese 5G invasion if it accelerates the
launch of Indian 5G. India is working on technologies that would enable it to launch Indigenous 5G that would
run IOT platforms for civilian as well as military applications. The banning of Chinese apps and blocking of
hardware supply chains would be the correct counteroffensive to protect the business and security interests of
the country. The problem is India being poor in "implementation".

Our scientists and industry can bring technology at a winning speed, but where India starts losing out is in slow
adoption, getting entangled in policy processes and the crosshairs of the bureaucracy. The current dispensation
in Delhi realises the importance of winning this war.

India must get its timing right. The implementation of 5G, though a bit delayed, can make India a good
alternative to China. But agreements like RCEP and China's other debt strategies will remain a larger threat to
the world.

Q.130 [30465398]
It can be inferred from the given passage that:

https://www.aspiration.ai/LAW/sis/Solution.jsp?qsetId=JAmrn2p8H1A=&qsetName=LST Mock 27 2021 (CLAT) 203/223


1/17/2021 Mock Analysis

a India is still weighing the pros and cons of 5G.

b 5G requires a lot of energy for its operation.

c 5G may prove hazardous to the environment.

d 5G networks are not available in India yet.

Solution:
 Answer key/Solution
Correct Answer : d
Your Answer : d
Option (d) can be inferred from "India must get its timing right. The
implementation of 5G, though a bit delayed, can make India a good alternative to China."
FeedBack Bookmark

Directions for questions 131 to 133: Following is an array of questions to test your reasoning ability in different
situations. Answer each of them according to the question asked in each of them respectively:

Q.131 [30465398]
There's no denying that sleep is an important part of our health and we're not getting enough of it. Studies have
linked sleep deprivation to higher risks of stroke and lower productivity. Americans know that they need more
sleepy time and for years many employers - like Ben & Jerry's, Zappos, Nike and the Hu ngton Post - have
allowed their employees to take a snooze while at the o ce, according to the National Sleep Foundation. These
rms are also investing in hi-tech products - like specialized "Energy Pods", which are ergonomic chairs
equipped with privacy visors, timers and speakers - so that their employees can snooze in comfort.

With which of the following would the author of the above paragraph most likely agree?

a Quality nap time in o ce is looked at in a positive manner by some employers.

b If the employees sleep more, they will be more likely to feel more loyal to the company.

c Americans employees have more comfort in their o ces now.

d Sleep deprivation leads to many fatal diseases.

Solution:
 Answer key/Solution
Correct Answer : a
The author is talking about employers giving more facilities to their employees to
take snooze during o ce hours. Only option (a) goes with the main idea of the
paragraph. The other options are either extreme or irrelevant.
FeedBack Bookmark

https://www.aspiration.ai/LAW/sis/Solution.jsp?qsetId=JAmrn2p8H1A=&qsetName=LST Mock 27 2021 (CLAT) 204/223


1/17/2021 Mock Analysis

Directions for questions 131 to 133: Following is an array of questions to test your reasoning ability in different
situations. Answer each of them according to the question asked in each of them respectively:

Q.132 [30465398]
It is a matter of grave concern that retired army o cials act as lobbyists for large arms dealers. It is a clear cut
case of ‘con ict of interests’. So, the government is mulling over a ban on such activities for ve years after
retirement for high level army o cials. However, according to majority of such o cials, such a ban will affect
their ability to earn a livelihood. The government pension is not just adequate and the o cials will, as a result,
suffer nancially for at least ve years.

Which of the following is an assumption implicit in the conclusion derived by the affected army o cials?

a The government must not interfere in the livelihood of any citizen.

b Majority of the lobbyists of armed contractors are high level political o cials.

c Currently, all retired army o cials can act as lobbyists.

d High level army o cials don’t have any other option of earning a decent livelihood except being a lobbyist.

Solution:
 Answer key/Solution
Correct Answer : d
Your Answer : c
Option (a) - It is not an assumption as it is not part of the speaker's conclusion.
This is, in fact, a far-fetched conclusion. Option (b) - This is irrelevant to the given context. Option (c) - This is
surely relevant. But even if we negate the statement, the conclusion that 'the ban will affect the livelihood of
some retired army o cials' will not be affected. Option (d) - If we negate this, it will affect the conclusion of
the army o cials. So, it is the correct answer.
FeedBack Bookmark

Directions for questions 131 to 133: Following is an array of questions to test your reasoning ability in different
situations. Answer each of them according to the question asked in each of them respectively:

Q.133 [30465398]
Self-love is a tricky issue, and the right amount of it has always depended on perspective. I have healthy
selfesteem; you’re a bit full of yourself; he’s a total narcissist. But in a world where you can buy a stick to hold
your phone at the approved distance to take a photograph of yourself, has it all gone a bit too far? And if so, how
did that happen?

Which of the following is the most valid inference based on the above argument?

a ‘Sel es’ have lowered the self-esteem of the current world population.

https://www.aspiration.ai/LAW/sis/Solution.jsp?qsetId=JAmrn2p8H1A=&qsetName=LST Mock 27 2021 (CLAT) 205/223


1/17/2021 Mock Analysis

b Self-love and its measurements are subjective.

c Self-love is bene cial as long as one doesn’t become obsessed with the idea of perfection.

d The world has become more and more weird in its pursuit of self-love.

Solution:
 Answer key/Solution
Correct Answer : b
Your Answer : b
This is an easy conclusion based question. The author calls the concept of ‘self-
love’ a tricky one and its interpretation may vary. So, option (b) best summarizes the author’s main point.
Option (a) is too extreme as the author has not talked about the world population in a de nitive sense. Option
(d) is extreme for the same reason. Option (c) talks about ‘perfection’ which is alien to the context. So, option
(b) is the correct choice.
FeedBack Bookmark

Q.134 [30465398]
Tapendra walks out of the entrance of his o ce building, which faces towards East. At rst he turns right and
walks for 25 m, then he again turns right and walks straight for 45 meters, then turns to the right and walks
straight for 50 meters; then he turns towards left and walks for 35 meters. Finally, he turns back and walks for
80 m. How far and in which direction does Tapendra have to walk to reach the entrance of his o ce building?

a 35 m, South

b 45 m, East

c 25 m, South

d 25 m, West

https://www.aspiration.ai/LAW/sis/Solution.jsp?qsetId=JAmrn2p8H1A=&qsetName=LST Mock 27 2021 (CLAT) 206/223


1/17/2021 Mock Analysis

Solution:
 Answer key/Solution
Correct Answer : c
Your Answer : b

FeedBack Bookmark

Q.135 [30465398]
P, Q, R, S, T, U, V and W are eight members of a family in which there are three married couples, two sons and
two daughters. T, a lawyer is married to S, a writer whose only child, R, is a student. The grandmother of U is
married to P the banker. P's son, W is a scientist, who is married to V , an actor. There is one teacher and one
male doctor in the family. S is the sister of W.

Which of the following is not TRUE about the family?

a S, V and Q are mothers.

b U and W are sons.

c The lawyer is a daughter-in-law.

d The student's cousin is a doctor.

https://www.aspiration.ai/LAW/sis/Solution.jsp?qsetId=JAmrn2p8H1A=&qsetName=LST Mock 27 2021 (CLAT) 207/223


1/17/2021 Mock Analysis

Solution:
 Answer key/Solution
Correct Answer : c

FeedBack Bookmark

Sec 5
Directions for questions 136 to 140: Answer the questions on the basis of the information given below.

The bar graph given below shows the number of passengers (in lakh) arriving and departing from the country
through the national airlines during the period 2004-05 to 2013-14.

The line graph shows the freight (in 104 kg) carried by the airlines during the period 2004-05 to 2013-14.

https://www.aspiration.ai/LAW/sis/Solution.jsp?qsetId=JAmrn2p8H1A=&qsetName=LST Mock 27 2021 (CLAT) 208/223


1/17/2021 Mock Analysis

Q.136 [30465398]
What was the difference between the total number of passengers arriving and departing from the country to
international destinations from 2005-06 to 2009-10?

a 12 lakh

b 10 lakh

c 15 lakh

d 22 lakh

Solution:
 Answer key/Solution
Correct Answer : b

FeedBack Bookmark

Directions for questions 136 to 140: Answer the questions on the basis of the information given below.

The bar graph given below shows the number of passengers (in lakh) arriving and departing from the country
through the national airlines during the period 2004-05 to 2013-14.

The line graph shows the freight (in 104 kg) carried by the airlines during the period 2004-05 to 2013-14.

https://www.aspiration.ai/LAW/sis/Solution.jsp?qsetId=JAmrn2p8H1A=&qsetName=LST Mock 27 2021 (CLAT) 209/223


1/17/2021 Mock Analysis

Q.137 [30465398]
What was the total number of international passengers carried by the airlines from 2010-11 to 2013-14?

a 4.42 crore

b 5.48 crore

c 5.58 crore

d 6.46 crore

Solution:
 Answer key/Solution
Correct Answer : c

FeedBack Bookmark

Directions for questions 136 to 140: Answer the questions on the basis of the information given below.

The bar graph given below shows the number of passengers (in lakh) arriving and departing from the country
through the national airlines during the period 2004-05 to 2013-14.

The line graph shows the freight (in 104 kg) carried by the airlines during the period 2004-05 to 2013-14.

https://www.aspiration.ai/LAW/sis/Solution.jsp?qsetId=JAmrn2p8H1A=&qsetName=LST Mock 27 2021 (CLAT) 210/223


1/17/2021 Mock Analysis

Q.138 [30465398]
What was the approximate percentage increase in freight carried by the airlines from 2007-08 to 2012-13?

a 70%

b 60%

c 75%

d 85%

Solution:
 Answer key/Solution
Correct Answer : a

FeedBack Bookmark

Directions for questions 136 to 140: Answer the questions on the basis of the information given below.

The bar graph given below shows the number of passengers (in lakh) arriving and departing from the country
through the national airlines during the period 2004-05 to 2013-14.

The line graph shows the freight (in 104 kg) carried by the airlines during the period 2004-05 to 2013-14.

https://www.aspiration.ai/LAW/sis/Solution.jsp?qsetId=JAmrn2p8H1A=&qsetName=LST Mock 27 2021 (CLAT) 211/223


1/17/2021 Mock Analysis

Q.139 [30465398]
What was the average freight carried per year by the airlines from 2007-08 to 2010-11?

a 12.525 × 104 kg

b 14.725 × 104 kg

c 18.025 × 104 kg

d 16.825 × 104 kg

Solution:
 Answer key/Solution
Correct Answer : d

FeedBack Bookmark

https://www.aspiration.ai/LAW/sis/Solution.jsp?qsetId=JAmrn2p8H1A=&qsetName=LST Mock 27 2021 (CLAT) 212/223


1/17/2021 Mock Analysis

Directions for questions 136 to 140: Answer the questions on the basis of the information given below.

The bar graph given below shows the number of passengers (in lakh) arriving and departing from the country
through the national airlines during the period 2004-05 to 2013-14.

The line graph shows the freight (in 104 kg) carried by the airlines during the period 2004-05 to 2013-14.

Q.140 [30465398]
What was the maximum number of years for which the airlines witnessed a continuous increase in the freight
carried?

a Four

b Five

c Six

d Seven

Solution:
 Answer key/Solution
Correct Answer : b
It is clear from the graph that the freight carried witnessed a continuous increase
from 2005-06 to 2010-11. Hence, the number of years is ve.
FeedBack Bookmark

https://www.aspiration.ai/LAW/sis/Solution.jsp?qsetId=JAmrn2p8H1A=&qsetName=LST Mock 27 2021 (CLAT) 213/223


1/17/2021 Mock Analysis

Directions for questions 141 to 145: Answer the questions on the basis of the information given below.

The two tables given below show the total production of milk by six farms and the ratio of cow milk is to buffalo
milk in each farm.

Q.141 [30465398]
The production of Cow milk by Farm D in 2019 is approximately what per cent of its total production in 2016?

a 88.4%

b 78.2%

c 91.6%

d 67.5%

Solution:
 Answer key/Solution
Correct Answer : c
Your Answer : c

FeedBack Bookmark

https://www.aspiration.ai/LAW/sis/Solution.jsp?qsetId=JAmrn2p8H1A=&qsetName=LST Mock 27 2021 (CLAT) 214/223


1/17/2021 Mock Analysis

Directions for questions 141 to 145: Answer the questions on the basis of the information given below.

The two tables given below show the total production of milk by six farms and the ratio of cow milk is to buffalo
milk in each farm.

Q.142 [30465398]
During 2017 which of the given farms produced more Buffalo milk than Farm B?

a D

b E

c F

d Both D and E

Solution:
 Answer key/Solution
Correct Answer : d
Your Answer : d

FeedBack Bookmark

https://www.aspiration.ai/LAW/sis/Solution.jsp?qsetId=JAmrn2p8H1A=&qsetName=LST Mock 27 2021 (CLAT) 215/223


1/17/2021 Mock Analysis

Directions for questions 141 to 145: Answer the questions on the basis of the information given below.

The two tables given below show the total production of milk by six farms and the ratio of cow milk is to buffalo
milk in each farm.

Q.143 [30465398]
What was the difference between the total production of Cow milk by Farm C and Farm A during the above
mentioned time period?

a 2255 litres

b 2565 litres

c 2045 litres

d 2785 litres

Solution:
 Answer key/Solution
Correct Answer : b
Your Answer : b

FeedBack Bookmark

https://www.aspiration.ai/LAW/sis/Solution.jsp?qsetId=JAmrn2p8H1A=&qsetName=LST Mock 27 2021 (CLAT) 216/223


1/17/2021 Mock Analysis

Directions for questions 141 to 145: Answer the questions on the basis of the information given below.

The two tables given below show the total production of milk by six farms and the ratio of cow milk is to buffalo
milk in each farm.

Q.144 [30465398]
The quantity of buffalo milk produced by Farm F during 2019 was what percentage more or less than the buffalo
milk produced by Farm E during the same time?

a 10%

b 15%

c 12%

d 8%

Solution:
 Answer key/Solution
Correct Answer : a
Your Answer : a

FeedBack Bookmark

https://www.aspiration.ai/LAW/sis/Solution.jsp?qsetId=JAmrn2p8H1A=&qsetName=LST Mock 27 2021 (CLAT) 217/223


1/17/2021 Mock Analysis

Directions for questions 141 to 145: Answer the questions on the basis of the information given below.

The two tables given below show the total production of milk by six farms and the ratio of cow milk is to buffalo
milk in each farm.

Q.145 [30465398]
What was the average production of milk by these six farms during the year 2018?

a 5775 litres

b 6473 litres

c 6975 litres

d 8317 litres

Solution:
 Answer key/Solution
Correct Answer : c
Your Answer : c

FeedBack Bookmark

https://www.aspiration.ai/LAW/sis/Solution.jsp?qsetId=JAmrn2p8H1A=&qsetName=LST Mock 27 2021 (CLAT) 218/223


1/17/2021 Mock Analysis

Directions for questions 146 to 150: Answer the questions on the basis of the information given below.

In the given gure PS had length 42 m and it is the diameter of a circular ground having centre O. The lengths
PQ, QR and RS are equal. The semicircular area with diameter QS has a pond with depth 1.5 m whereas the
shaded region PQS has a lawn. The sector POT has owering plants whereas the triangular area SOT has a
vegetable garden. The shaded area outside chord TS is used to make compost from the garden wastages.

Q.146 [30465398]
What is the volume of water in the pond if it is lled up to two-third the depth?

a 325 m3

b 216 m3

c 308 m3

d 418 m3

Solution:
 Answer key/Solution
Correct Answer : c

FeedBack Bookmark

https://www.aspiration.ai/LAW/sis/Solution.jsp?qsetId=JAmrn2p8H1A=&qsetName=LST Mock 27 2021 (CLAT) 219/223


1/17/2021 Mock Analysis

Directions for questions 146 to 150: Answer the questions on the basis of the information given below.

In the given gure PS had length 42 m and it is the diameter of a circular ground having centre O. The lengths
PQ, QR and RS are equal. The semicircular area with diameter QS has a pond with depth 1.5 m whereas the
shaded region PQS has a lawn. The sector POT has owering plants whereas the triangular area SOT has a
vegetable garden. The shaded area outside chord TS is used to make compost from the garden wastages.

Q.147 [30465398]
What is the area covered by lawn?

a 425 m2

b 245 m2

c 355 m2

d 385 m2

Solution:
 Answer key/Solution
Correct Answer : d

FeedBack Bookmark

https://www.aspiration.ai/LAW/sis/Solution.jsp?qsetId=JAmrn2p8H1A=&qsetName=LST Mock 27 2021 (CLAT) 220/223


1/17/2021 Mock Analysis

Directions for questions 146 to 150: Answer the questions on the basis of the information given below.

In the given gure PS had length 42 m and it is the diameter of a circular ground having centre O. The lengths
PQ, QR and RS are equal. The semicircular area with diameter QS has a pond with depth 1.5 m whereas the
shaded region PQS has a lawn. The sector POT has owering plants whereas the triangular area SOT has a
vegetable garden. The shaded area outside chord TS is used to make compost from the garden wastages.

Q.148 [30465398]
If the owering plants yield 400 g of owers per square metre every day, then what will be the revenue generated
in a month by selling the owers at the rate of Rs.150 per kg?(Take number of days in a month = 30)

a Rs.6,23,700

b Rs.6,03,510

c Rs.3,23,700

d Rs.4,27,300

Solution:
 Answer key/Solution
Correct Answer : a

FeedBack Bookmark

https://www.aspiration.ai/LAW/sis/Solution.jsp?qsetId=JAmrn2p8H1A=&qsetName=LST Mock 27 2021 (CLAT) 221/223


1/17/2021 Mock Analysis

Directions for questions 146 to 150: Answer the questions on the basis of the information given below.

In the given gure PS had length 42 m and it is the diameter of a circular ground having centre O. The lengths
PQ, QR and RS are equal. The semicircular area with diameter QS has a pond with depth 1.5 m whereas the
shaded region PQS has a lawn. The sector POT has owering plants whereas the triangular area SOT has a
vegetable garden. The shaded area outside chord TS is used to make compost from the garden wastages.

Q.149 [30465398]
If there are tomatoes planted in the vegetable garden and the yield per square metre is 2 kg everyday, then what
is the revenue generated in a week by selling tomatoes at the rate of Rs.30 per kg?

a Rs.95,750

b Rs.79,590

c Rs.88,650

d Rs.92,610

Solution:
 Answer key/Solution
Correct Answer : d

FeedBack Bookmark

https://www.aspiration.ai/LAW/sis/Solution.jsp?qsetId=JAmrn2p8H1A=&qsetName=LST Mock 27 2021 (CLAT) 222/223


1/17/2021 Mock Analysis

Directions for questions 146 to 150: Answer the questions on the basis of the information given below.

In the given gure PS had length 42 m and it is the diameter of a circular ground having centre O. The lengths
PQ, QR and RS are equal. The semicircular area with diameter QS has a pond with depth 1.5 m whereas the
shaded region PQS has a lawn. The sector POT has owering plants whereas the triangular area SOT has a
vegetable garden. The shaded area outside chord TS is used to make compost from the garden wastages.

Q.150 [30465398]
What is the measure of the area used for making compost?

a 106 m2

b 221 m2

c 126 m2

d 201 m2

Solution:
 Answer key/Solution
Correct Answer : c

FeedBack Bookmark

https://www.aspiration.ai/LAW/sis/Solution.jsp?qsetId=JAmrn2p8H1A=&qsetName=LST Mock 27 2021 (CLAT) 223/223

You might also like